[obm-l] Re: [obm-l] Re: [obm-l] Sequência das médias ponderadas

2020-08-26 Por tôpico Artur Costa Steiner
Em qua, 26 de ago de 2020 20:19, Claudio Buffara 
escreveu:

> Acho que isso tá mal formulado.
> Por exemplo,quanto é s_3?
>

De modo geral, s_n = (Soma(k =1, n) p_k a_k))/(Soma(k =1, n) p_k)

Artur

>
> On Tue, Aug 25, 2020 at 3:49 PM Artur Costa Steiner <
> artur.costa.stei...@gmail.com> wrote:
>
>> Isso me foi dado como verdadeiro, mas ainda não cheguei a uma conclusão.
>>
>> Sejam (a_ n) uma sequência de reais positivos e (s_n) a sequência das
>> médias ponderadas de (a_n,) com relação aos pesos positivos (p_n).
>> Suponhamos que lim p_n = p, 0 < p < oo, e que a sequência das médias
>> aritméticas de (a_n) convirja para o real a. Então, s_n --> a.
>>
>> Abraços
>> Artur
>>
>> --
>> Esta mensagem foi verificada pelo sistema de antivírus e
>> acredita-se estar livre de perigo.
>
>
> --
> Esta mensagem foi verificada pelo sistema de antivírus e
> acredita-se estar livre de perigo.

-- 
Esta mensagem foi verificada pelo sistema de antiv�rus e
 acredita-se estar livre de perigo.



[obm-l] Re: [obm-l] Sequência das médias ponderadas

2020-08-26 Por tôpico Claudio Buffara
Acho que isso tá mal formulado.
Por exemplo,quanto é s_3?

On Tue, Aug 25, 2020 at 3:49 PM Artur Costa Steiner <
artur.costa.stei...@gmail.com> wrote:

> Isso me foi dado como verdadeiro, mas ainda não cheguei a uma conclusão.
>
> Sejam (a_ n) uma sequência de reais positivos e (s_n) a sequência das
> médias ponderadas de (a_n,) com relação aos pesos positivos (p_n).
> Suponhamos que lim p_n = p, 0 < p < oo, e que a sequência das médias
> aritméticas de (a_n) convirja para o real a. Então, s_n --> a.
>
> Abraços
> Artur
>
> --
> Esta mensagem foi verificada pelo sistema de antivírus e
> acredita-se estar livre de perigo.

-- 
Esta mensagem foi verificada pelo sistema de antiv�rus e
 acredita-se estar livre de perigo.



[obm-l] Re: [obm-l] Re: [obm-l] Re: [obm-l] Re: [obm-l] Re: [obm-l] Geometria plana com desigualdade de médias?

2020-08-26 Por tôpico Pedro José
Boa noite!
Anderson,
achei legal a sua visão. Mas não consegui evoluir com nada.
Todavia, fiquei com uma dúvida. Como x+y é um dos ângulos do triângulo
temos a restrição 0 escreveu:

> Em qui., 20 de ago. de 2020 às 22:03, Anderson Torres
>  escreveu:
> >
> > Em ter., 18 de ago. de 2020 às 19:51, Pedro José 
> escreveu:
> > >
> > > Boa noite!
> > > Cláudio,
> > > não consegui nada geométrico.
> > > O máximo que atingi foi:
> > > a/ha + b/hb + c/hc= [cotg(A1) +cotg (A2)]  + [cotg(B1) +cotg (B2)] +
> co[tg(C1) +cotg (C2)] com A1 + A2 = A; B1 + B2 + B e C1 + C2 = C.
> > > Para ser mínimo cada termo entre colchetes deve ser mínimo, o que
> ocorre quando A1 = A2; B1 = B2 e C1 = C2. Logo P seria o encontro das
> bissetrizes e logo I.
> > > Onde: A1= PAB e A2=PAC; B1=PBA e B2=PBC; C1=PCA e C2=PCB.
> >
> > Acho que daqui poderia sair uma interpretação mais escamoteada.
> > Afinal, trigonometria é uma espécie de "ponto de contato" entre a
> > geometria analítica e a sintética, entre a nuvem de desenhos e a de
> > números.
> >
> > Acredito que a solução aqui seria arranjar uma interpretação
> > geométrica desses colchetes de co-tangentes. Acredito que possamos
> > apelar para Ptolomeu em algum momento ou para um macete de
> > semelhanças, pois as projeções de um ponto sobre duas retas criam um
> > quadrilátero cíclico.
>
> Acrescentando mais coisas: se queremos minimizar cot(x) +cot(y) com
> x+y fixo, isto é equivalente a minimizar tan(90-x)+tan(90-y) com
> 90-x+90-y fixo. Ou como maximizar tan(x) + tan(y) com x+y fixo.
>
> Geometricamente, tangente é cateto oposto dividido por cateto
> adjacente. Logo uma soma de tangentes com catetos adjacentes iguais
> equivale a uma soma de catetos opostos! Assim sendo, nosso problema
> pode ser pensado da seguinte forma:
>
> Dados um ponto A e uma reta d fixos, temos que construir duas retas x
> e y, com ângulo 'alfa' entre elas, ambas passando por A e tais que a
> distância entre os pontos X e Y, que elas geram ao intersectar d, seja
> mínima.
>
> Daí fica fácil argumentar que a altura por A também tem que ser a
> bissetriz por A.
>
> No fundo do fundo é uma forma de geometrizar a solução trigonométrica.
> A trigonometria se torna apenas um atalho.
>
> Vou formalizar isso mais tarde, com desenhos e tudo.
>
>
>
> >
> > Isso até me lembra o famoso artigo do Shine sobre geometria cearense
> > VS geometria paulista:
> > https://cyshine.webs.com/geometria-2005.pdf
> >
> >
> > >
> > > Saudações,
> > > PJMS
> > >
> > > Em ter., 18 de ago. de 2020 às 11:34, Claudio Buffara <
> claudio.buff...@gmail.com> escreveu:
> > >>
> > >> Será que tem uma demonstração mais geométrica e menos algébrica
> disso? E que torne o resultado mais intuitivo?
> > >> É razoável que o ponto P não esteja muito próximo de qualquer dos
> lados, pois neste caso, se P se aproximasse do lado a, por exemplo, a/h_a
> cresceria e a expressão se afastaria do valor mínimo.
> > >> Mas, com lados não necessariamente congruentes, não é óbvio, a
> priori, que P deva ser equidistante dos três.
> > >> De fato, seria razoável esperar que P estivesse mais próximo do maior
> lado e conjecturar, por exemplo, que o P que minimiza a expressão é tal que
> a/h_a = b/h_b = c/h_c.
> > >> O fato de P ser o incentro não me parece a conjectura mais evidente
> neste caso.
> > >>
> > >>
> > >> On Sun, Aug 16, 2020 at 10:11 AM Matheus Secco <
> matheusse...@gmail.com> wrote:
> > >>>
> > >>> Olá, Vanderlei.
> > >>> Por Cauchy-Schwarz, temos
> > >>>
> > >>> (a/ha + b/hb + c/hc) * (a*ha + b*hb + c*hc) >= (a+b+c)^2.  (#)
> > >>>
> > >>> Como (a*ha + b*hb + c*hc) = 2S, onde S é a área de ABC, segue que a
> expressão a/ha + b/hb + c/hc é pelo menos 2p^2/S, onde p é o semi-perimetro.
> > >>>
> > >>> Por outro lado, a igualdade em (#) ocorre se, e somente se, ha = hb
> = hc, ou seja, quando P é o incentro do triângulo
> > >>>
> > >>> Abraços,
> > >>> Matheus
> > >>>
> > >>> Em dom, 16 de ago de 2020 08:59, Professor Vanderlei Nemitz <
> vanderma...@gmail.com> escreveu:
> > 
> >  Bom dia!
> > 
> >  Tentei utilizar alguma desigualdade de médias aqui, mas não tive
> êxito. Alguém ajuda?
> >  Muito agradecido!
> > 
> >  Seja P um ponto no interior de um triângulo e sejam ha, hb e hc as
> distâncias de P aos lados a, b e c, respectivamente. Mostre que o valor
> mínimo de (a/ha) + (b/hb) + (c/hc) ocorre quando P é o incentivo do
> triângulo ABC.
> > 
> >  --
> >  Esta mensagem foi verificada pelo sistema de antivírus e
> >  acredita-se estar livre de perigo.
> > >>>
> > >>>
> > >>> --
> > >>> Esta mensagem foi verificada pelo sistema de antivírus e
> > >>> acredita-se estar livre de perigo.
> > >>
> > >>
> > >> --
> > >> Esta mensagem foi verificada pelo sistema de antivírus e
> > >> acredita-se estar livre de perigo.
> > >
> > >
> > > --
> > > Esta mensagem foi verificada pelo sistema de antivírus e
> > > acredita-se estar livre de perigo.
>
> --
> Esta mensagem foi verificada pelo sistema de antivírus e
>  

[obm-l] Re: [obm-l] Re: [obm-l] Sequência das médias ponderadas

2020-08-25 Por tôpico Artur Costa Steiner
Em ter, 25 de ago de 2020 19:51, Esdras Muniz 
escreveu:

> Basta ter que as soma dos pesos vai pro infinito. Isso é um exercício do
> livro de análise real do Elon.
>

Mas acho que isso não prova o que foi pedido. O fato de a soma dos pesos
divergir implica que

liminf a_n <= liminf s_n <= limsup s_n <= limsup a_n. Assim, se lim a_n =
a, então m s_n = a. Mas não é isso que foi pedido.

Artur

>
> Em ter, 25 de ago de 2020 15:49, Artur Costa Steiner <
> artur.costa.stei...@gmail.com> escreveu:
>
>> Isso me foi dado como verdadeiro, mas ainda não cheguei a uma conclusão.
>>
>> Sejam (a_ n) uma sequência de reais positivos e (s_n) a sequência das
>> médias ponderadas de (a_n,) com relação aos pesos positivos (p_n).
>> Suponhamos que lim p_n = p, 0 < p < oo, e que a sequência das médias
>> aritméticas de (a_n) convirja para o real a. Então, s_n --> a.
>>
>> Abraços
>> Artur
>>
>> --
>> Esta mensagem foi verificada pelo sistema de antivírus e
>> acredita-se estar livre de perigo.
>
>
> --
> Esta mensagem foi verificada pelo sistema de antivírus e
> acredita-se estar livre de perigo.v

-- 
Esta mensagem foi verificada pelo sistema de antiv�rus e
 acredita-se estar livre de perigo.



[obm-l] Re: [obm-l] Sequência das médias ponderadas

2020-08-25 Por tôpico Esdras Muniz
Basta ter que as soma dos pesos vai pro infinito. Isso é um exercício do
livro de análise real do Elon.

Em ter, 25 de ago de 2020 15:49, Artur Costa Steiner <
artur.costa.stei...@gmail.com> escreveu:

> Isso me foi dado como verdadeiro, mas ainda não cheguei a uma conclusão.
>
> Sejam (a_ n) uma sequência de reais positivos e (s_n) a sequência das
> médias ponderadas de (a_n,) com relação aos pesos positivos (p_n).
> Suponhamos que lim p_n = p, 0 < p < oo, e que a sequência das médias
> aritméticas de (a_n) convirja para o real a. Então, s_n --> a.
>
> Abraços
> Artur
>
> --
> Esta mensagem foi verificada pelo sistema de antivírus e
> acredita-se estar livre de perigo.

-- 
Esta mensagem foi verificada pelo sistema de antiv�rus e
 acredita-se estar livre de perigo.



Re: [obm-l] Re: [obm-l] Re: [obm-l] Re: [obm-l] Elipse e lugar g eométrico

2020-08-25 Por tôpico qedtexte

Tenho um arquivo com uma figura mostrando as
elipses. Posso mandar no privado pra quem quiser.

Lus
--
Esta mensagem foi verificada pelo sistema de antiv�rus e
acredita-se estar livre de perigo.



[obm-l] Re: [obm-l] Re: [obm-l] Re: [obm-l] Elipse e lugar geométrico

2020-08-24 Por tôpico Claudio Buffara
Tem um artigo do (saudoso) Morgado na RPM sobre este assunto. Está aqui:
http://www.rpm.org.br/cdrpm/43/5.htm

[]s,
Claudio.

On Sat, Aug 22, 2020 at 9:14 PM Professor Vanderlei Nemitz <
vanderma...@gmail.com> wrote:

> Demorei para responder, mas queria dizer que foi muito boa sua resolução,
> como sempre, Ralph!
> Eu desconhecia o fato de as coordenadas do incentro serem dadas daquela
> forma.
>
> Muitíssimo obrigado!
>
> Vanderlei
>
>
> 
>  Livre
> de vírus. www.avast.com
> .
> <#m_3828508563874758992_DAB4FAD8-2DD7-40BB-A1B8-4E2AA1F9FDF2>
>
> Em qui., 20 de ago. de 2020 às 00:37, Ralph Costa Teixeira <
> ralp...@gmail.com> escreveu:
>
>> As coordenadas do incentro sao a media ponderada das coordenadas dos
>> vertices, usando os lados como pesos. Ou seja, se escrevo P=(5cost,4sint),
>> F1=(-3,0), F2=(3,0) e Incentro=(x,y):
>>
>> x = (30cost + (-3)b + 3c) / 16
>> y = (24sint + 0 + 0) / 16
>>
>> onde b=d(P,F2) e c=d(P,F1). Note que b+c=eixo maior = 10.
>>
>> Mais especificamente:
>>
>> b^2=d(P,F2)^2=(5cost-3)^2+(4sint)^2=9(cost)^2-30cost+25=(3cost-5)^2, ou
>> seja, b=5-3cost, portanto c=5+3cost.
>>
>> Jogando na formula de x e y:
>>
>> x= 3cost ; y=3sint/2. Outra elipse, a saber, (x^2)/9+(y^2)/(9/4)=1
>> (talvez tirando os pontos onde tudo degenera, para ser chato).
>>
>> Abraço, Ralph.
>>
>>
>>
>> Hmm Assim:
>>
>> On Wed, Aug 19, 2020 at 11:58 PM Professor Vanderlei Nemitz <
>> vanderma...@gmail.com> wrote:
>>
>>> Oi!
>>> Venho com mais uma envolvendo incentro.
>>>
>>> *O ponto P pertence a uma elipse de focos F1 e F2 e de equação (x^2)/25
>>> + (y^2)/16 = 1. Determine o lugar geométrico do incentro do triângulo
>>> PF1F2.*
>>>
>>> Muito obrigado!
>>>
>>>
>>> 
>>>  Livre
>>> de vírus. www.avast.com
>>> .
>>>
>>> <#m_3828508563874758992_m_6041639077674691514_m_2344932968934913062_DAB4FAD8-2DD7-40BB-A1B8-4E2AA1F9FDF2>
>>>
>>> --
>>> Esta mensagem foi verificada pelo sistema de antivírus e
>>> acredita-se estar livre de perigo.
>>
>>
>> --
>> Esta mensagem foi verificada pelo sistema de antivírus e
>> acredita-se estar livre de perigo.
>
>
> --
> Esta mensagem foi verificada pelo sistema de antivírus e
> acredita-se estar livre de perigo.

-- 
Esta mensagem foi verificada pelo sistema de antiv�rus e
 acredita-se estar livre de perigo.



Re: [obm-l] Re: [obm-l] Re: [obm-l] Elipse e lugar geométrico

2020-08-23 Por tôpico qedtexte
Sauda,c~oes, 


Demorei para responder, mas queria dizer que foi muito boa sua 
resoluo, como sempre, Ralph!
Eu desconhecia o fato de as coordenadas do incentro serem dadas daquela 
forma.
Eu no lembrava mais mas a demonstraao aparece na RPM 43, por 
exemplo.
Artigo do Morgado.

Resolvendo o problema de construir um tringulo dados a,b+c,d_b, com 
BD_b = d_b
bissetriz interna, clculos simblicos mostraram que se A percorre a elipse E_1 de focos 
B,C, centro O_1e eixo maior PQ=b+c, ento D_b percorre uma outra elipse E_2 tal que:


It looks that foot D of B-bisector is onellipse confocal C and major axis BX,where X 
dividesinternally CQ in thesame ratio as B externally.

Ou seja, os focos de E_2 so F,C com eixo maior BX, tal que B,X 
so
conjugados harmnicos do segmento CQ.

Os eixos menores de E_1 e E_2 so paralelos. Fazendo uma "figura", 
vem:

P B   F O_1  
 O_2  CX  Q

BC=a; O_1=M_a, PQ=b+c; O_2 centro de E_2.

D_c deve percorrer uma elipse tambm. Assim, fazendo I=BD_b/\CD_c acho 
que
d pra mostrar que o lugar geomtrico de I  outra elipse.

Como o centro do crculo \phi=(B,d_b) est num dos eixos da elipse, o 
problema
tem uma construo com rgua e compasso.

Abraos,
Lus









--
Esta mensagem foi verificada pelo sistema de antiv�rus e
acredita-se estar livre de perigo.



[obm-l] Re: [obm-l] Re: [obm-l] Resto da divisão de um polinômio

2020-08-23 Por tôpico Artur Costa Steiner
Outra solução:

As raízes de x^2 + x + 1 são r1 = cis 2pi/3 e r2= cis 4pi/3, as raízes
cúbicas de 1 exceto 1.. Sendo D o quociente e ax + b o resto da divisão.
temos que

*x^30 - x^28 + 7x^12 = D(x) ( x^2 + x + 1) + ax + b*

*Como 30 e 12 são múltiplos de 3, r1^30 = r1^12 = 1. E r1^28 = r1 . r1^27 =
r1. Assim, fazendo x = r1, vem*

*1 - r1 + 7 = 8 - r1 = 8 - (-1/2 + raiz(3)/2 i) = 8,5 - raiz(3)/2 i = -a/2
+ a raiz(3)/2 i + b.*

*Assim, a = -1, b - a/2 = b + 1/2 = 8,5 , b = 8. O resto é -x + 8.*

*Fazendo x = r2, chegamos ao mesmo resultado.*

*Mas acho que  solução do Matheus é mais elegante.  A minha se simplificou
porque 30 e 12 são múltiplos de 3 e 28 = 1 + 27, 27 também múltiplo de 3*


*Artur *
Em sáb, 22 de ago de 2020 21:38, Matheus Secco 
escreveu:

> Neste caso específico, você pode usar congruência de polinômios (que é bem
> similar à congruência para números inteiros) e isso é facilitado pelo fato
> de x^3 - 1 = (x - 1)(x^2+x+1).
>
> Com essa observação, podemos escrever x^3 == 1 (mod x^2+x+1). Com isso,
> x^30 = (x^3)^10 == 1 (mod x^2+x+1), x^28 = (x^3)^9 * x == x (mod x^2+x+1) e
> 7x^12 = 7(x^3)^4 == 7 (mod x^2+x+1).
>
> Assim x^30 - x^28 + 7x^12 == 1 - x + 7 == 8 - x (mod x^2+x+1) e como o
> grau de (8-x) é menor que o grau de (x^2+x+1), o resto é 8 - x.
>
> Abraços,
>
> Matheus.
>
> On Sat, Aug 22, 2020 at 9:19 PM Professor Vanderlei Nemitz <
> vanderma...@gmail.com> wrote:
>
>> Oi!
>>
>> Existe algum fato específico que ajude a determinar o resto da divisão de
>> um polinômio de grau elevado por outro, ou depende do caso?
>>
>> Por exemplo, como encontrar o seguinte resto, sem excessivos cálculos?
>> Muito obrigado!
>>
>> *Determine o resto da divisão do polinômio x^30 - x^28 + 7x^12 por x^2 +
>> x + 1?*
>>
>>
>> 
>>  Livre
>> de vírus. www.avast.com
>> .
>>
>> <#m_-615878603476649313_m_2794148347715460488_DAB4FAD8-2DD7-40BB-A1B8-4E2AA1F9FDF2>
>>
>> --
>> Esta mensagem foi verificada pelo sistema de antivírus e
>> acredita-se estar livre de perigo.
>
>
> --
> Esta mensagem foi verificada pelo sistema de antivírus e
> acredita-se estar livre de perigo.

-- 
Esta mensagem foi verificada pelo sistema de antiv�rus e
 acredita-se estar livre de perigo.



[obm-l] Re: [obm-l] Resto da divisão de um polinômio

2020-08-22 Por tôpico Esdras Muniz
Vc pode dizer que x^2=-(x+1) e abrir as contas.

Em sáb, 22 de ago de 2020 21:19, Professor Vanderlei Nemitz <
vanderma...@gmail.com> escreveu:

> Oi!
>
> Existe algum fato específico que ajude a determinar o resto da divisão de
> um polinômio de grau elevado por outro, ou depende do caso?
>
> Por exemplo, como encontrar o seguinte resto, sem excessivos cálculos?
> Muito obrigado!
>
> *Determine o resto da divisão do polinômio x^30 - x^28 + 7x^12 por x^2 +
> x + 1?*
>
>
> 
>  Livre
> de vírus. www.avast.com
> .
> <#m_695281535589841859_DAB4FAD8-2DD7-40BB-A1B8-4E2AA1F9FDF2>
>
> --
> Esta mensagem foi verificada pelo sistema de antivírus e
> acredita-se estar livre de perigo.

-- 
Esta mensagem foi verificada pelo sistema de antiv�rus e
 acredita-se estar livre de perigo.



[obm-l] Re: [obm-l] Resto da divisão de um polinômio

2020-08-22 Por tôpico Matheus Secco
Neste caso específico, você pode usar congruência de polinômios (que é bem
similar à congruência para números inteiros) e isso é facilitado pelo fato
de x^3 - 1 = (x - 1)(x^2+x+1).

Com essa observação, podemos escrever x^3 == 1 (mod x^2+x+1). Com isso,
x^30 = (x^3)^10 == 1 (mod x^2+x+1), x^28 = (x^3)^9 * x == x (mod x^2+x+1) e
7x^12 = 7(x^3)^4 == 7 (mod x^2+x+1).

Assim x^30 - x^28 + 7x^12 == 1 - x + 7 == 8 - x (mod x^2+x+1) e como o grau
de (8-x) é menor que o grau de (x^2+x+1), o resto é 8 - x.

Abraços,

Matheus.

On Sat, Aug 22, 2020 at 9:19 PM Professor Vanderlei Nemitz <
vanderma...@gmail.com> wrote:

> Oi!
>
> Existe algum fato específico que ajude a determinar o resto da divisão de
> um polinômio de grau elevado por outro, ou depende do caso?
>
> Por exemplo, como encontrar o seguinte resto, sem excessivos cálculos?
> Muito obrigado!
>
> *Determine o resto da divisão do polinômio x^30 - x^28 + 7x^12 por x^2 +
> x + 1?*
>
>
> 
>  Livre
> de vírus. www.avast.com
> .
> <#m_2794148347715460488_DAB4FAD8-2DD7-40BB-A1B8-4E2AA1F9FDF2>
>
> --
> Esta mensagem foi verificada pelo sistema de antivírus e
> acredita-se estar livre de perigo.

-- 
Esta mensagem foi verificada pelo sistema de antiv�rus e
 acredita-se estar livre de perigo.



[obm-l] Re: [obm-l] Re: [obm-l] Elipse e lugar geométrico

2020-08-22 Por tôpico Professor Vanderlei Nemitz
Demorei para responder, mas queria dizer que foi muito boa sua resolução,
como sempre, Ralph!
Eu desconhecia o fato de as coordenadas do incentro serem dadas daquela
forma.

Muitíssimo obrigado!

Vanderlei


Livre
de vírus. www.avast.com
.
<#DAB4FAD8-2DD7-40BB-A1B8-4E2AA1F9FDF2>

Em qui., 20 de ago. de 2020 às 00:37, Ralph Costa Teixeira <
ralp...@gmail.com> escreveu:

> As coordenadas do incentro sao a media ponderada das coordenadas dos
> vertices, usando os lados como pesos. Ou seja, se escrevo P=(5cost,4sint),
> F1=(-3,0), F2=(3,0) e Incentro=(x,y):
>
> x = (30cost + (-3)b + 3c) / 16
> y = (24sint + 0 + 0) / 16
>
> onde b=d(P,F2) e c=d(P,F1). Note que b+c=eixo maior = 10.
>
> Mais especificamente:
>
> b^2=d(P,F2)^2=(5cost-3)^2+(4sint)^2=9(cost)^2-30cost+25=(3cost-5)^2, ou
> seja, b=5-3cost, portanto c=5+3cost.
>
> Jogando na formula de x e y:
>
> x= 3cost ; y=3sint/2. Outra elipse, a saber, (x^2)/9+(y^2)/(9/4)=1 (talvez
> tirando os pontos onde tudo degenera, para ser chato).
>
> Abraço, Ralph.
>
>
>
> Hmm Assim:
>
> On Wed, Aug 19, 2020 at 11:58 PM Professor Vanderlei Nemitz <
> vanderma...@gmail.com> wrote:
>
>> Oi!
>> Venho com mais uma envolvendo incentro.
>>
>> *O ponto P pertence a uma elipse de focos F1 e F2 e de equação (x^2)/25 +
>> (y^2)/16 = 1. Determine o lugar geométrico do incentro do triângulo PF1F2.*
>>
>> Muito obrigado!
>>
>>
>> 
>>  Livre
>> de vírus. www.avast.com
>> .
>>
>> <#m_6041639077674691514_m_2344932968934913062_DAB4FAD8-2DD7-40BB-A1B8-4E2AA1F9FDF2>
>>
>> --
>> Esta mensagem foi verificada pelo sistema de antivírus e
>> acredita-se estar livre de perigo.
>
>
> --
> Esta mensagem foi verificada pelo sistema de antivírus e
> acredita-se estar livre de perigo.

-- 
Esta mensagem foi verificada pelo sistema de antiv�rus e
 acredita-se estar livre de perigo.



[obm-l] Re: [obm-l] Re: [obm-l] Re: [obm-l] Re: [obm-l] Geometria plana com desigualdade de médias?

2020-08-20 Por tôpico Anderson Torres
Em qui., 20 de ago. de 2020 às 22:03, Anderson Torres
 escreveu:
>
> Em ter., 18 de ago. de 2020 às 19:51, Pedro José  
> escreveu:
> >
> > Boa noite!
> > Cláudio,
> > não consegui nada geométrico.
> > O máximo que atingi foi:
> > a/ha + b/hb + c/hc= [cotg(A1) +cotg (A2)]  + [cotg(B1) +cotg (B2)] + 
> > co[tg(C1) +cotg (C2)] com A1 + A2 = A; B1 + B2 + B e C1 + C2 = C.
> > Para ser mínimo cada termo entre colchetes deve ser mínimo, o que ocorre 
> > quando A1 = A2; B1 = B2 e C1 = C2. Logo P seria o encontro das bissetrizes 
> > e logo I.
> > Onde: A1= PAB e A2=PAC; B1=PBA e B2=PBC; C1=PCA e C2=PCB.
>
> Acho que daqui poderia sair uma interpretação mais escamoteada.
> Afinal, trigonometria é uma espécie de "ponto de contato" entre a
> geometria analítica e a sintética, entre a nuvem de desenhos e a de
> números.
>
> Acredito que a solução aqui seria arranjar uma interpretação
> geométrica desses colchetes de co-tangentes. Acredito que possamos
> apelar para Ptolomeu em algum momento ou para um macete de
> semelhanças, pois as projeções de um ponto sobre duas retas criam um
> quadrilátero cíclico.

Acrescentando mais coisas: se queremos minimizar cot(x) +cot(y) com
x+y fixo, isto é equivalente a minimizar tan(90-x)+tan(90-y) com
90-x+90-y fixo. Ou como maximizar tan(x) + tan(y) com x+y fixo.

Geometricamente, tangente é cateto oposto dividido por cateto
adjacente. Logo uma soma de tangentes com catetos adjacentes iguais
equivale a uma soma de catetos opostos! Assim sendo, nosso problema
pode ser pensado da seguinte forma:

Dados um ponto A e uma reta d fixos, temos que construir duas retas x
e y, com ângulo 'alfa' entre elas, ambas passando por A e tais que a
distância entre os pontos X e Y, que elas geram ao intersectar d, seja
mínima.

Daí fica fácil argumentar que a altura por A também tem que ser a
bissetriz por A.

No fundo do fundo é uma forma de geometrizar a solução trigonométrica.
A trigonometria se torna apenas um atalho.

Vou formalizar isso mais tarde, com desenhos e tudo.



>
> Isso até me lembra o famoso artigo do Shine sobre geometria cearense
> VS geometria paulista:
> https://cyshine.webs.com/geometria-2005.pdf
>
>
> >
> > Saudações,
> > PJMS
> >
> > Em ter., 18 de ago. de 2020 às 11:34, Claudio Buffara 
> >  escreveu:
> >>
> >> Será que tem uma demonstração mais geométrica e menos algébrica disso? E 
> >> que torne o resultado mais intuitivo?
> >> É razoável que o ponto P não esteja muito próximo de qualquer dos lados, 
> >> pois neste caso, se P se aproximasse do lado a, por exemplo, a/h_a 
> >> cresceria e a expressão se afastaria do valor mínimo.
> >> Mas, com lados não necessariamente congruentes, não é óbvio, a priori, que 
> >> P deva ser equidistante dos três.
> >> De fato, seria razoável esperar que P estivesse mais próximo do maior lado 
> >> e conjecturar, por exemplo, que o P que minimiza a expressão é tal que 
> >> a/h_a = b/h_b = c/h_c.
> >> O fato de P ser o incentro não me parece a conjectura mais evidente neste 
> >> caso.
> >>
> >>
> >> On Sun, Aug 16, 2020 at 10:11 AM Matheus Secco  
> >> wrote:
> >>>
> >>> Olá, Vanderlei.
> >>> Por Cauchy-Schwarz, temos
> >>>
> >>> (a/ha + b/hb + c/hc) * (a*ha + b*hb + c*hc) >= (a+b+c)^2.  (#)
> >>>
> >>> Como (a*ha + b*hb + c*hc) = 2S, onde S é a área de ABC, segue que a 
> >>> expressão a/ha + b/hb + c/hc é pelo menos 2p^2/S, onde p é o 
> >>> semi-perimetro.
> >>>
> >>> Por outro lado, a igualdade em (#) ocorre se, e somente se, ha = hb = hc, 
> >>> ou seja, quando P é o incentro do triângulo
> >>>
> >>> Abraços,
> >>> Matheus
> >>>
> >>> Em dom, 16 de ago de 2020 08:59, Professor Vanderlei Nemitz 
> >>>  escreveu:
> 
>  Bom dia!
> 
>  Tentei utilizar alguma desigualdade de médias aqui, mas não tive êxito. 
>  Alguém ajuda?
>  Muito agradecido!
> 
>  Seja P um ponto no interior de um triângulo e sejam ha, hb e hc as 
>  distâncias de P aos lados a, b e c, respectivamente. Mostre que o valor 
>  mínimo de (a/ha) + (b/hb) + (c/hc) ocorre quando P é o incentivo do 
>  triângulo ABC.
> 
>  --
>  Esta mensagem foi verificada pelo sistema de antivírus e
>  acredita-se estar livre de perigo.
> >>>
> >>>
> >>> --
> >>> Esta mensagem foi verificada pelo sistema de antivírus e
> >>> acredita-se estar livre de perigo.
> >>
> >>
> >> --
> >> Esta mensagem foi verificada pelo sistema de antivírus e
> >> acredita-se estar livre de perigo.
> >
> >
> > --
> > Esta mensagem foi verificada pelo sistema de antivírus e
> > acredita-se estar livre de perigo.

-- 
Esta mensagem foi verificada pelo sistema de antiv�rus e
 acredita-se estar livre de perigo.


=
Instru��es para entrar na lista, sair da lista e usar a lista em
http://www.mat.puc-rio.br/~obmlistas/obm-l.html
=


[obm-l] Re: [obm-l] Re: [obm-l] Re: [obm-l] Re: [obm-l] Geometria plana com desigualdade de médias?

2020-08-20 Por tôpico Anderson Torres
Em ter., 18 de ago. de 2020 às 19:51, Pedro José  escreveu:
>
> Boa noite!
> Cláudio,
> não consegui nada geométrico.
> O máximo que atingi foi:
> a/ha + b/hb + c/hc= [cotg(A1) +cotg (A2)]  + [cotg(B1) +cotg (B2)] + 
> co[tg(C1) +cotg (C2)] com A1 + A2 = A; B1 + B2 + B e C1 + C2 = C.
> Para ser mínimo cada termo entre colchetes deve ser mínimo, o que ocorre 
> quando A1 = A2; B1 = B2 e C1 = C2. Logo P seria o encontro das bissetrizes e 
> logo I.
> Onde: A1= PAB e A2=PAC; B1=PBA e B2=PBC; C1=PCA e C2=PCB.

Acho que daqui poderia sair uma interpretação mais escamoteada.
Afinal, trigonometria é uma espécie de "ponto de contato" entre a
geometria analítica e a sintética, entre a nuvem de desenhos e a de
números.

Acredito que a solução aqui seria arranjar uma interpretação
geométrica desses colchetes de co-tangentes. Acredito que possamos
apelar para Ptolomeu em algum momento ou para um macete de
semelhanças, pois as projeções de um ponto sobre duas retas criam um
quadrilátero cíclico.

Isso até me lembra o famoso artigo do Shine sobre geometria cearense
VS geometria paulista:
https://cyshine.webs.com/geometria-2005.pdf


>
> Saudações,
> PJMS
>
> Em ter., 18 de ago. de 2020 às 11:34, Claudio Buffara 
>  escreveu:
>>
>> Será que tem uma demonstração mais geométrica e menos algébrica disso? E que 
>> torne o resultado mais intuitivo?
>> É razoável que o ponto P não esteja muito próximo de qualquer dos lados, 
>> pois neste caso, se P se aproximasse do lado a, por exemplo, a/h_a cresceria 
>> e a expressão se afastaria do valor mínimo.
>> Mas, com lados não necessariamente congruentes, não é óbvio, a priori, que P 
>> deva ser equidistante dos três.
>> De fato, seria razoável esperar que P estivesse mais próximo do maior lado e 
>> conjecturar, por exemplo, que o P que minimiza a expressão é tal que a/h_a = 
>> b/h_b = c/h_c.
>> O fato de P ser o incentro não me parece a conjectura mais evidente neste 
>> caso.
>>
>>
>> On Sun, Aug 16, 2020 at 10:11 AM Matheus Secco  
>> wrote:
>>>
>>> Olá, Vanderlei.
>>> Por Cauchy-Schwarz, temos
>>>
>>> (a/ha + b/hb + c/hc) * (a*ha + b*hb + c*hc) >= (a+b+c)^2.  (#)
>>>
>>> Como (a*ha + b*hb + c*hc) = 2S, onde S é a área de ABC, segue que a 
>>> expressão a/ha + b/hb + c/hc é pelo menos 2p^2/S, onde p é o semi-perimetro.
>>>
>>> Por outro lado, a igualdade em (#) ocorre se, e somente se, ha = hb = hc, 
>>> ou seja, quando P é o incentro do triângulo
>>>
>>> Abraços,
>>> Matheus
>>>
>>> Em dom, 16 de ago de 2020 08:59, Professor Vanderlei Nemitz 
>>>  escreveu:

 Bom dia!

 Tentei utilizar alguma desigualdade de médias aqui, mas não tive êxito. 
 Alguém ajuda?
 Muito agradecido!

 Seja P um ponto no interior de um triângulo e sejam ha, hb e hc as 
 distâncias de P aos lados a, b e c, respectivamente. Mostre que o valor 
 mínimo de (a/ha) + (b/hb) + (c/hc) ocorre quando P é o incentivo do 
 triângulo ABC.

 --
 Esta mensagem foi verificada pelo sistema de antivírus e
 acredita-se estar livre de perigo.
>>>
>>>
>>> --
>>> Esta mensagem foi verificada pelo sistema de antivírus e
>>> acredita-se estar livre de perigo.
>>
>>
>> --
>> Esta mensagem foi verificada pelo sistema de antivírus e
>> acredita-se estar livre de perigo.
>
>
> --
> Esta mensagem foi verificada pelo sistema de antivírus e
> acredita-se estar livre de perigo.

-- 
Esta mensagem foi verificada pelo sistema de antiv�rus e
 acredita-se estar livre de perigo.


=
Instru��es para entrar na lista, sair da lista e usar a lista em
http://www.mat.puc-rio.br/~obmlistas/obm-l.html
=


[obm-l] Re: [obm-l] Álgebra

2020-08-20 Por tôpico Anderson Torres
Em sáb., 15 de ago. de 2020 às 17:57, marcone augusto araújo borges
 escreveu:
>
> Determinar todos os pares ordenados (x,y) de número racionais que são as 
> soluções da equação x^2019 + y^2019 = x^2020 + y^2020
> Desde já agradeço.

Hum, estou achando isso meio confuso.

Se x e y forem iguais, temos 2x^2019=2x^2020, x=1 ou x=0. Vamos então
supor x e y diferentes.

Podemos pegar estas duas frações como (m/d)^2019 + (n/d)^2019 =
(m/d)^2020 + (n/d)^2020 em que d é o menor valor possível.

Assim, multiplica tudo por d^2020, temos d*(m^2019 + n^2019) = (m^2020
+ n^2020).

Em outras palavras, queremos encontrar todos os ternos (m,n,d) de
inteiros tais que (m^2020 + n^2020)/(m^2019 + n^2019)=d é inteiro.

Para isso, vamos tentar procurar os primos p tais que p é divisor de
(m^2019 + n^2019) e também é divisor de (m^2020 + n^2020).
Podemos supor que este primo p exista, dado que m e n são pelo menos
1, logo (m^2019 + n^2019) é pelo menos 2, logo é fatorável.

Note que se m e n forem múltiplos de p, escrevendo m=pm' e n=dn',
obtemos d=p * (m'^2020 + n'^2020)/(m'^2019 + n'^2019). Assim sendo, d
também será múltiplo de p. Assim, obtemos uma nova solução (m',n,d')

Assim, podemos de cara excluir os casos em que m e n são múltiplos de
um mesmo primo p.

Enfim.

Se p é divisor de p é divisor de (m^2019 + n^2019) e também é divisor
de (m^2020 + n^2020), então também é divisor de (m+n)(m^2019 +
n^2019)-(m^2020 + n^2020) = m*n^2019+n*m^2019=mn(m^2018+n^2018).

Temos então três hipóteses, que não são necessariamente mutuamente exclusivas:

p é divisor de m
p é divisor de n
p é divisor de (m^2018+n^2018)

Se p é divisor de m, então também é divisor de n, e vice-versa, dado
que p é divisor da soma dos dois. Assim sendo, podemos de cara
eliminar os dois primeiros casos.

Logo, p é divisor de (m^2018+n^2018).

Acredito que, repetindo esse raciocínio mais umas 2018 vezes, chegamos
em que p deve ser divisor de m+n e de m^2+n^2.

Ou, novamente, de (m+n)(m+n)-(m^2+n^2)=2mn

Logo, p é divisor de 2. Ou seja, p é igual a 2. Ou seja, o único primo
possível nisto tudo é 2. Ou seja, (m^2+n^2)=2^k*(m+n), com m e n ambos
ímpares e primos entre si.

Assim, podemos escrever m=a+b e n=a-b para inteiros a e b. Assim,
(m^2+n^2)/(m+n)=((a+b)^2+(a-b)^2)/(a+b+a-b) = 2(a^2+b^2)/(2a)=a+b^2/a,
o que implica que a é divisor de b^2.

Mas a e b são primos entre si, pois se tivessem fatores em comum,
estes fatores apareceriam em m e n.

Logo, a=1. Mas a é maior que b, pois m=a-b é maior que 0. Logo, a=1 e
b=0. Assim sendo, m=n, e sendo primos entre si, daria m=n=1.

É isso mesmo? Só tem x=y=0 e x=y=1 como soluções?

>
> --
> Esta mensagem foi verificada pelo sistema de antivírus e
> acredita-se estar livre de perigo.

-- 
Esta mensagem foi verificada pelo sistema de antiv�rus e
 acredita-se estar livre de perigo.


=
Instru��es para entrar na lista, sair da lista e usar a lista em
http://www.mat.puc-rio.br/~obmlistas/obm-l.html
=


[obm-l] Re: [obm-l] Re: [obm-l] Re: [obm-l] Re: [obm-l] Re: [obm-l] Re: [obm-l] Re: [obm-l] polinômio irredutível

2020-08-20 Por tôpico Anderson Torres
Em seg., 17 de ago. de 2020 às 12:14, Claudio Buffara
 escreveu:
>
> Eu acho que o Eisenstein inventou este critério pra polinômios da forma x^n + 
> a ou, mais geralmente, pra polinômios ciclotômicos.
> Daí funciona bem.
>
> On Mon, Aug 17, 2020 at 11:02 AM Esdras Muniz  
> wrote:
>>
>> E se p=3, e p divide N^2+9, então p^2 divide N^2+9.
>>
>> Então o critério de Eisenstein realmente não é tão abrangente. Será que tem 
>> algum outro critério que cubra casos em que o de Eisenstein não cubra?
>>
>> Em seg, 17 de ago de 2020 09:46, Claudio Buffara  
>> escreveu:
>>>
>>> Boa! Se p <> 3 mas p divide 3N e 3N^2, então p divide N ==> p não divide 
>>> N^3 + 9.
>>>
>>> On Sun, Aug 16, 2020 at 10:51 PM Esdras Muniz  
>>> wrote:

 Tenta com x^3+9.

 Em dom, 16 de ago de 2020 15:24, Claudio Buffara 
  escreveu:
>
> f(x) em Z[x], bem entendido...
>
>
> On Sun, Aug 16, 2020 at 3:08 PM Claudio Buffara 
>  wrote:
>>
>> Que tal essa aqui?
>> Prove ou disprove que, dado um polinômio f(x), irredutível sobre Q, 
>> existe um inteiro N tal que a irredutibilidade de f pode ser provada 
>> pelo critério de Eisenstein aplicado a f(x+N).

Isso me parece uma daquelas questões ultra capciosas sobre "prove ou
disprove que existe um algoritmo que..."

Inclusive imagino que esta seja uma questão indecidível neste caso particular...

>>
>> On Sun, Aug 16, 2020 at 2:31 PM Matheus Secco  
>> wrote:
>>>
>>> O melhor jeito é pensar na contrapositiva (supondo que você esteja 
>>> falando sobre irredutibilidade em Z[x] ou até em Q[x]): se f(x) fatora 
>>> como g(x)*h(x), então f(x+a) fatora como g(x+a) *h(x+a) e é claro que 
>>> uma vez que g(x) e h(x) têm coeficientes inteiros, então g(x+a) e 
>>> h(x+a) também têm. A recíproca é essencialmente idêntica.
>>>
>>> Abraços
>>>
>>> Em dom, 16 de ago de 2020 14:11, Luís Lopes  
>>> escreveu:

 Sauda,c~oes,

 Como provar que um polinômio f(x) tendo como coeficientes números 
 inteiros
 é irredutível se e somente se f(x+a) é irredutível para algum  
 inteiro ?

 Luís




 --
 Esta mensagem foi verificada pelo sistema de antivírus e
 acredita-se estar livre de perigo.
>>>
>>>
>>> --
>>> Esta mensagem foi verificada pelo sistema de antivírus e
>>> acredita-se estar livre de perigo.
>
>
> --
> Esta mensagem foi verificada pelo sistema de antivírus e
> acredita-se estar livre de perigo.


 --
 Esta mensagem foi verificada pelo sistema de antivírus e
 acredita-se estar livre de perigo.
>>>
>>>
>>> --
>>> Esta mensagem foi verificada pelo sistema de antivírus e
>>> acredita-se estar livre de perigo.
>>
>>
>> --
>> Esta mensagem foi verificada pelo sistema de antivírus e
>> acredita-se estar livre de perigo.
>
>
> --
> Esta mensagem foi verificada pelo sistema de antivírus e
> acredita-se estar livre de perigo.

-- 
Esta mensagem foi verificada pelo sistema de antiv�rus e
 acredita-se estar livre de perigo.


=
Instru��es para entrar na lista, sair da lista e usar a lista em
http://www.mat.puc-rio.br/~obmlistas/obm-l.html
=


[obm-l] Re: [obm-l] Elipse e lugar geométrico

2020-08-19 Por tôpico Ralph Costa Teixeira
As coordenadas do incentro sao a media ponderada das coordenadas dos
vertices, usando os lados como pesos. Ou seja, se escrevo P=(5cost,4sint),
F1=(-3,0), F2=(3,0) e Incentro=(x,y):

x = (30cost + (-3)b + 3c) / 16
y = (24sint + 0 + 0) / 16

onde b=d(P,F2) e c=d(P,F1). Note que b+c=eixo maior = 10.

Mais especificamente:

b^2=d(P,F2)^2=(5cost-3)^2+(4sint)^2=9(cost)^2-30cost+25=(3cost-5)^2, ou
seja, b=5-3cost, portanto c=5+3cost.

Jogando na formula de x e y:

x= 3cost ; y=3sint/2. Outra elipse, a saber, (x^2)/9+(y^2)/(9/4)=1 (talvez
tirando os pontos onde tudo degenera, para ser chato).

Abraço, Ralph.



Hmm Assim:

On Wed, Aug 19, 2020 at 11:58 PM Professor Vanderlei Nemitz <
vanderma...@gmail.com> wrote:

> Oi!
> Venho com mais uma envolvendo incentro.
>
> *O ponto P pertence a uma elipse de focos F1 e F2 e de equação (x^2)/25 +
> (y^2)/16 = 1. Determine o lugar geométrico do incentro do triângulo PF1F2.*
>
> Muito obrigado!
>
>
> 
>  Livre
> de vírus. www.avast.com
> .
> <#m_2344932968934913062_DAB4FAD8-2DD7-40BB-A1B8-4E2AA1F9FDF2>
>
> --
> Esta mensagem foi verificada pelo sistema de antivírus e
> acredita-se estar livre de perigo.

-- 
Esta mensagem foi verificada pelo sistema de antiv�rus e
 acredita-se estar livre de perigo.



[obm-l] Re: [obm-l] Re: [obm-l] Re: [obm-l] Re: [obm-l] Geometria plana com desigualdade de médias?

2020-08-18 Por tôpico Claudio Buffara
Realmente, não era isso que eu estava procurando...  mas valeu! É outra
solução.


On Tue, Aug 18, 2020 at 7:51 PM Pedro José  wrote:

> Boa noite!
> Cláudio,
> não consegui nada geométrico.
> O máximo que atingi foi:
> a/ha + b/hb + c/hc= [cotg(A1) +cotg (A2)]  + [cotg(B1) +cotg (B2)] +
> co[tg(C1) +cotg (C2)] com A1 + A2 = A; B1 + B2 + B e C1 + C2 = C.
> Para ser mínimo cada termo entre colchetes deve ser mínimo, o que ocorre
> quando A1 = A2; B1 = B2 e C1 = C2. Logo P seria o encontro das bissetrizes
> e logo I.
> Onde: A1= PAB e A2=PAC; B1=PBA e B2=PBC; C1=PCA e C2=PCB.
>
> Saudações,
> PJMS
>
> Em ter., 18 de ago. de 2020 às 11:34, Claudio Buffara <
> claudio.buff...@gmail.com> escreveu:
>
>> Será que tem uma demonstração mais geométrica e menos algébrica disso? E
>> que torne o resultado mais intuitivo?
>> É razoável que o ponto P não esteja muito próximo de qualquer dos lados,
>> pois neste caso, se P se aproximasse do lado a, por exemplo,
>> a/h_a cresceria e a expressão se afastaria do valor mínimo.
>> Mas, com lados não necessariamente congruentes, não é óbvio, a priori,
>> que P deva ser equidistante dos três.
>> De fato, seria razoável esperar que P estivesse mais próximo do maior
>> lado e conjecturar, por exemplo, que o P que minimiza a expressão é tal que
>> a/h_a = b/h_b = c/h_c.
>> O fato de P ser o incentro não me parece a conjectura mais evidente neste
>> caso.
>>
>>
>> On Sun, Aug 16, 2020 at 10:11 AM Matheus Secco 
>> wrote:
>>
>>> Olá, Vanderlei.
>>> Por Cauchy-Schwarz, temos
>>>
>>> (a/ha + b/hb + c/hc) * (a*ha + b*hb + c*hc) >= (a+b+c)^2.  (#)
>>>
>>> Como (a*ha + b*hb + c*hc) = 2S, onde S é a área de ABC, segue que a
>>> expressão a/ha + b/hb + c/hc é pelo menos 2p^2/S, onde p é o
>>> semi-perimetro.
>>>
>>> Por outro lado, a igualdade em (#) ocorre se, e somente se, ha = hb =
>>> hc, ou seja, quando P é o incentro do triângulo
>>>
>>> Abraços,
>>> Matheus
>>>
>>> Em dom, 16 de ago de 2020 08:59, Professor Vanderlei Nemitz <
>>> vanderma...@gmail.com> escreveu:
>>>
 Bom dia!

 Tentei utilizar alguma desigualdade de médias aqui, mas não tive êxito.
 Alguém ajuda?
 Muito agradecido!

 Seja P um ponto no interior de um triângulo e sejam ha, hb e hc as
 distâncias de P aos lados a, b e c, respectivamente. Mostre que o valor
 mínimo de (a/ha) + (b/hb) + (c/hc) ocorre quando P é o incentivo do
 triângulo ABC.

 --
 Esta mensagem foi verificada pelo sistema de antivírus e
 acredita-se estar livre de perigo.
>>>
>>>
>>> --
>>> Esta mensagem foi verificada pelo sistema de antivírus e
>>> acredita-se estar livre de perigo.
>>
>>
>> --
>> Esta mensagem foi verificada pelo sistema de antivírus e
>> acredita-se estar livre de perigo.
>
>
> --
> Esta mensagem foi verificada pelo sistema de antivírus e
> acredita-se estar livre de perigo.

-- 
Esta mensagem foi verificada pelo sistema de antiv�rus e
 acredita-se estar livre de perigo.



[obm-l] Re: [obm-l] Re: [obm-l] Re: [obm-l] Geometria plana com desigualdade de médias?

2020-08-18 Por tôpico Pedro José
Boa noite!
Cláudio,
não consegui nada geométrico.
O máximo que atingi foi:
a/ha + b/hb + c/hc= [cotg(A1) +cotg (A2)]  + [cotg(B1) +cotg (B2)] +
co[tg(C1) +cotg (C2)] com A1 + A2 = A; B1 + B2 + B e C1 + C2 = C.
Para ser mínimo cada termo entre colchetes deve ser mínimo, o que ocorre
quando A1 = A2; B1 = B2 e C1 = C2. Logo P seria o encontro das bissetrizes
e logo I.
Onde: A1= PAB e A2=PAC; B1=PBA e B2=PBC; C1=PCA e C2=PCB.

Saudações,
PJMS

Em ter., 18 de ago. de 2020 às 11:34, Claudio Buffara <
claudio.buff...@gmail.com> escreveu:

> Será que tem uma demonstração mais geométrica e menos algébrica disso? E
> que torne o resultado mais intuitivo?
> É razoável que o ponto P não esteja muito próximo de qualquer dos lados,
> pois neste caso, se P se aproximasse do lado a, por exemplo,
> a/h_a cresceria e a expressão se afastaria do valor mínimo.
> Mas, com lados não necessariamente congruentes, não é óbvio, a priori, que
> P deva ser equidistante dos três.
> De fato, seria razoável esperar que P estivesse mais próximo do maior lado
> e conjecturar, por exemplo, que o P que minimiza a expressão é tal que
> a/h_a = b/h_b = c/h_c.
> O fato de P ser o incentro não me parece a conjectura mais evidente neste
> caso.
>
>
> On Sun, Aug 16, 2020 at 10:11 AM Matheus Secco 
> wrote:
>
>> Olá, Vanderlei.
>> Por Cauchy-Schwarz, temos
>>
>> (a/ha + b/hb + c/hc) * (a*ha + b*hb + c*hc) >= (a+b+c)^2.  (#)
>>
>> Como (a*ha + b*hb + c*hc) = 2S, onde S é a área de ABC, segue que a
>> expressão a/ha + b/hb + c/hc é pelo menos 2p^2/S, onde p é o
>> semi-perimetro.
>>
>> Por outro lado, a igualdade em (#) ocorre se, e somente se, ha = hb = hc,
>> ou seja, quando P é o incentro do triângulo
>>
>> Abraços,
>> Matheus
>>
>> Em dom, 16 de ago de 2020 08:59, Professor Vanderlei Nemitz <
>> vanderma...@gmail.com> escreveu:
>>
>>> Bom dia!
>>>
>>> Tentei utilizar alguma desigualdade de médias aqui, mas não tive êxito.
>>> Alguém ajuda?
>>> Muito agradecido!
>>>
>>> Seja P um ponto no interior de um triângulo e sejam ha, hb e hc as
>>> distâncias de P aos lados a, b e c, respectivamente. Mostre que o valor
>>> mínimo de (a/ha) + (b/hb) + (c/hc) ocorre quando P é o incentivo do
>>> triângulo ABC.
>>>
>>> --
>>> Esta mensagem foi verificada pelo sistema de antivírus e
>>> acredita-se estar livre de perigo.
>>
>>
>> --
>> Esta mensagem foi verificada pelo sistema de antivírus e
>> acredita-se estar livre de perigo.
>
>
> --
> Esta mensagem foi verificada pelo sistema de antivírus e
> acredita-se estar livre de perigo.

-- 
Esta mensagem foi verificada pelo sistema de antiv�rus e
 acredita-se estar livre de perigo.



[obm-l] Re: [obm-l] Re: [obm-l] Geometria plana com desigualdade de médias?

2020-08-18 Por tôpico Claudio Buffara
Será que tem uma demonstração mais geométrica e menos algébrica disso? E
que torne o resultado mais intuitivo?
É razoável que o ponto P não esteja muito próximo de qualquer dos lados,
pois neste caso, se P se aproximasse do lado a, por exemplo,
a/h_a cresceria e a expressão se afastaria do valor mínimo.
Mas, com lados não necessariamente congruentes, não é óbvio, a priori, que
P deva ser equidistante dos três.
De fato, seria razoável esperar que P estivesse mais próximo do maior lado
e conjecturar, por exemplo, que o P que minimiza a expressão é tal que
a/h_a = b/h_b = c/h_c.
O fato de P ser o incentro não me parece a conjectura mais evidente neste
caso.


On Sun, Aug 16, 2020 at 10:11 AM Matheus Secco 
wrote:

> Olá, Vanderlei.
> Por Cauchy-Schwarz, temos
>
> (a/ha + b/hb + c/hc) * (a*ha + b*hb + c*hc) >= (a+b+c)^2.  (#)
>
> Como (a*ha + b*hb + c*hc) = 2S, onde S é a área de ABC, segue que a
> expressão a/ha + b/hb + c/hc é pelo menos 2p^2/S, onde p é o
> semi-perimetro.
>
> Por outro lado, a igualdade em (#) ocorre se, e somente se, ha = hb = hc,
> ou seja, quando P é o incentro do triângulo
>
> Abraços,
> Matheus
>
> Em dom, 16 de ago de 2020 08:59, Professor Vanderlei Nemitz <
> vanderma...@gmail.com> escreveu:
>
>> Bom dia!
>>
>> Tentei utilizar alguma desigualdade de médias aqui, mas não tive êxito.
>> Alguém ajuda?
>> Muito agradecido!
>>
>> Seja P um ponto no interior de um triângulo e sejam ha, hb e hc as
>> distâncias de P aos lados a, b e c, respectivamente. Mostre que o valor
>> mínimo de (a/ha) + (b/hb) + (c/hc) ocorre quando P é o incentivo do
>> triângulo ABC.
>>
>> --
>> Esta mensagem foi verificada pelo sistema de antivírus e
>> acredita-se estar livre de perigo.
>
>
> --
> Esta mensagem foi verificada pelo sistema de antivírus e
> acredita-se estar livre de perigo.

-- 
Esta mensagem foi verificada pelo sistema de antiv�rus e
 acredita-se estar livre de perigo.



Re: [obm-l] Re: [obm-l] Re: [obm-l] Re: [obm-l] Re: [obm-l] Re: [obm-l] Re: [obm-l] polinômio irredutível

2020-08-17 Por tôpico qedtexte

Sauda,c~oes,

Legal o estudo dox^3+9.

Sobre oEisenstein generalizado (teorema 3 em

http://yufeizhao.com/olympiad/intpoly.pdf;), tenho duas 
dvidas:





Theorem 3(Extended Eisenstein).Letf(x) =anxn+an1xn1++a1x+a0be a polynomial with integer coefficients 
such thatp|aifor 0i  k,pﰀ|/akandp2ﰀ|/a0. Thenf(x) has an irreducible factor of degree greater thank.


Quando k=n obtm-se o critrio tradicional.

i) quais as condies para os outros coeficientes a_(k+1), a_(k+2), 
. , a_n ?
p pode dividi-los ou no ?

ii) o grau do fator irredutvel   k ou = k ?

Lus














--
Esta mensagem foi verificada pelo sistema de antiv�rus e
acredita-se estar livre de perigo.



[obm-l] Re: [obm-l] Re: [obm-l] Re: [obm-l] Re: [obm-l] Re: [obm-l] Re: [obm-l] polinômio irredutível

2020-08-17 Por tôpico Claudio Buffara
Eu acho que o Eisenstein inventou este critério pra polinômios da forma
x^n + a ou, mais geralmente, pra polinômios ciclotômicos.
Daí funciona bem.

On Mon, Aug 17, 2020 at 11:02 AM Esdras Muniz 
wrote:

> E se p=3, e p divide N^2+9, então p^2 divide N^2+9.
>
> Então o critério de Eisenstein realmente não é tão abrangente. Será que
> tem algum outro critério que cubra casos em que o de Eisenstein não cubra?
>
> Em seg, 17 de ago de 2020 09:46, Claudio Buffara <
> claudio.buff...@gmail.com> escreveu:
>
>> Boa! Se p <> 3 mas p divide 3N e 3N^2, então p divide N ==> p não divide
>> N^3 + 9.
>>
>> On Sun, Aug 16, 2020 at 10:51 PM Esdras Muniz 
>> wrote:
>>
>>> Tenta com x^3+9.
>>>
>>> Em dom, 16 de ago de 2020 15:24, Claudio Buffara <
>>> claudio.buff...@gmail.com> escreveu:
>>>
 f(x) em Z[x], bem entendido...


 On Sun, Aug 16, 2020 at 3:08 PM Claudio Buffara <
 claudio.buff...@gmail.com> wrote:

> Que tal essa aqui?
> Prove ou disprove que, dado um polinômio f(x), irredutível sobre Q,
> existe um inteiro N tal que a irredutibilidade de f pode ser provada pelo
> critério de Eisenstein aplicado a f(x+N).
>
> On Sun, Aug 16, 2020 at 2:31 PM Matheus Secco 
> wrote:
>
>> O melhor jeito é pensar na contrapositiva (supondo que você esteja
>> falando sobre irredutibilidade em Z[x] ou até em Q[x]): se f(x) fatora 
>> como
>> g(x)*h(x), então f(x+a) fatora como g(x+a) *h(x+a) e é claro que uma vez
>> que g(x) e h(x) têm coeficientes inteiros, então g(x+a) e h(x+a) também
>> têm. A recíproca é essencialmente idêntica.
>>
>> Abraços
>>
>> Em dom, 16 de ago de 2020 14:11, Luís Lopes 
>> escreveu:
>>
>>> Sauda,c~oes,
>>>
>>> Como provar que um polinômio f(x) tendo como coeficientes números
>>> inteiros
>>> é irredutível se e somente se f(x+a) é irredutível para algum 
>>> inteiro ?
>>>
>>> Luís
>>>
>>>
>>>
>>>
>>> --
>>> Esta mensagem foi verificada pelo sistema de antivírus e
>>> acredita-se estar livre de perigo.
>>>
>>
>> --
>> Esta mensagem foi verificada pelo sistema de antivírus e
>> acredita-se estar livre de perigo.
>
>
 --
 Esta mensagem foi verificada pelo sistema de antivírus e
 acredita-se estar livre de perigo.
>>>
>>>
>>> --
>>> Esta mensagem foi verificada pelo sistema de antivírus e
>>> acredita-se estar livre de perigo.
>>
>>
>> --
>> Esta mensagem foi verificada pelo sistema de antivírus e
>> acredita-se estar livre de perigo.
>
>
> --
> Esta mensagem foi verificada pelo sistema de antivírus e
> acredita-se estar livre de perigo.

-- 
Esta mensagem foi verificada pelo sistema de antiv�rus e
 acredita-se estar livre de perigo.



[obm-l] Re: [obm-l] Re: [obm-l] Re: [obm-l] Re: [obm-l] Re: [obm-l] polinômio irredutível

2020-08-17 Por tôpico Esdras Muniz
E se p=3, e p divide N^2+9, então p^2 divide N^2+9.

Então o critério de Eisenstein realmente não é tão abrangente. Será que tem
algum outro critério que cubra casos em que o de Eisenstein não cubra?

Em seg, 17 de ago de 2020 09:46, Claudio Buffara 
escreveu:

> Boa! Se p <> 3 mas p divide 3N e 3N^2, então p divide N ==> p não divide
> N^3 + 9.
>
> On Sun, Aug 16, 2020 at 10:51 PM Esdras Muniz 
> wrote:
>
>> Tenta com x^3+9.
>>
>> Em dom, 16 de ago de 2020 15:24, Claudio Buffara <
>> claudio.buff...@gmail.com> escreveu:
>>
>>> f(x) em Z[x], bem entendido...
>>>
>>>
>>> On Sun, Aug 16, 2020 at 3:08 PM Claudio Buffara <
>>> claudio.buff...@gmail.com> wrote:
>>>
 Que tal essa aqui?
 Prove ou disprove que, dado um polinômio f(x), irredutível sobre Q,
 existe um inteiro N tal que a irredutibilidade de f pode ser provada pelo
 critério de Eisenstein aplicado a f(x+N).

 On Sun, Aug 16, 2020 at 2:31 PM Matheus Secco 
 wrote:

> O melhor jeito é pensar na contrapositiva (supondo que você esteja
> falando sobre irredutibilidade em Z[x] ou até em Q[x]): se f(x) fatora 
> como
> g(x)*h(x), então f(x+a) fatora como g(x+a) *h(x+a) e é claro que uma vez
> que g(x) e h(x) têm coeficientes inteiros, então g(x+a) e h(x+a) também
> têm. A recíproca é essencialmente idêntica.
>
> Abraços
>
> Em dom, 16 de ago de 2020 14:11, Luís Lopes 
> escreveu:
>
>> Sauda,c~oes,
>>
>> Como provar que um polinômio f(x) tendo como coeficientes números
>> inteiros
>> é irredutível se e somente se f(x+a) é irredutível para algum 
>> inteiro ?
>>
>> Luís
>>
>>
>>
>>
>> --
>> Esta mensagem foi verificada pelo sistema de antivírus e
>> acredita-se estar livre de perigo.
>>
>
> --
> Esta mensagem foi verificada pelo sistema de antivírus e
> acredita-se estar livre de perigo.


>>> --
>>> Esta mensagem foi verificada pelo sistema de antivírus e
>>> acredita-se estar livre de perigo.
>>
>>
>> --
>> Esta mensagem foi verificada pelo sistema de antivírus e
>> acredita-se estar livre de perigo.
>
>
> --
> Esta mensagem foi verificada pelo sistema de antivírus e
> acredita-se estar livre de perigo.

-- 
Esta mensagem foi verificada pelo sistema de antiv�rus e
 acredita-se estar livre de perigo.



[obm-l] Re: [obm-l] Re: [obm-l] Re: [obm-l] Re: [obm-l] polinômio irredutível

2020-08-17 Por tôpico Claudio Buffara
Boa! Se p <> 3 mas p divide 3N e 3N^2, então p divide N ==> p não divide
N^3 + 9.

On Sun, Aug 16, 2020 at 10:51 PM Esdras Muniz 
wrote:

> Tenta com x^3+9.
>
> Em dom, 16 de ago de 2020 15:24, Claudio Buffara <
> claudio.buff...@gmail.com> escreveu:
>
>> f(x) em Z[x], bem entendido...
>>
>>
>> On Sun, Aug 16, 2020 at 3:08 PM Claudio Buffara <
>> claudio.buff...@gmail.com> wrote:
>>
>>> Que tal essa aqui?
>>> Prove ou disprove que, dado um polinômio f(x), irredutível sobre Q,
>>> existe um inteiro N tal que a irredutibilidade de f pode ser provada pelo
>>> critério de Eisenstein aplicado a f(x+N).
>>>
>>> On Sun, Aug 16, 2020 at 2:31 PM Matheus Secco 
>>> wrote:
>>>
 O melhor jeito é pensar na contrapositiva (supondo que você esteja
 falando sobre irredutibilidade em Z[x] ou até em Q[x]): se f(x) fatora como
 g(x)*h(x), então f(x+a) fatora como g(x+a) *h(x+a) e é claro que uma vez
 que g(x) e h(x) têm coeficientes inteiros, então g(x+a) e h(x+a) também
 têm. A recíproca é essencialmente idêntica.

 Abraços

 Em dom, 16 de ago de 2020 14:11, Luís Lopes 
 escreveu:

> Sauda,c~oes,
>
> Como provar que um polinômio f(x) tendo como coeficientes números
> inteiros
> é irredutível se e somente se f(x+a) é irredutível para algum 
> inteiro ?
>
> Luís
>
>
>
>
> --
> Esta mensagem foi verificada pelo sistema de antivírus e
> acredita-se estar livre de perigo.
>

 --
 Esta mensagem foi verificada pelo sistema de antivírus e
 acredita-se estar livre de perigo.
>>>
>>>
>> --
>> Esta mensagem foi verificada pelo sistema de antivírus e
>> acredita-se estar livre de perigo.
>
>
> --
> Esta mensagem foi verificada pelo sistema de antivírus e
> acredita-se estar livre de perigo.

-- 
Esta mensagem foi verificada pelo sistema de antiv�rus e
 acredita-se estar livre de perigo.



[obm-l] Re: [obm-l] Re: [obm-l] Geometria plana com desigualdade de médias?

2020-08-17 Por tôpico Professor Vanderlei Nemitz
Muito obrigado, Matheus!
Pensei nas outras desigualdades, menos em Cauchy-Schwarz.

Muito bom!

Em dom, 16 de ago de 2020 10:11, Matheus Secco 
escreveu:

> Olá, Vanderlei.
> Por Cauchy-Schwarz, temos
>
> (a/ha + b/hb + c/hc) * (a*ha + b*hb + c*hc) >= (a+b+c)^2.  (#)
>
> Como (a*ha + b*hb + c*hc) = 2S, onde S é a área de ABC, segue que a
> expressão a/ha + b/hb + c/hc é pelo menos 2p^2/S, onde p é o
> semi-perimetro.
>
> Por outro lado, a igualdade em (#) ocorre se, e somente se, ha = hb = hc,
> ou seja, quando P é o incentro do triângulo
>
> Abraços,
> Matheus
>
> Em dom, 16 de ago de 2020 08:59, Professor Vanderlei Nemitz <
> vanderma...@gmail.com> escreveu:
>
>> Bom dia!
>>
>> Tentei utilizar alguma desigualdade de médias aqui, mas não tive êxito.
>> Alguém ajuda?
>> Muito agradecido!
>>
>> Seja P um ponto no interior de um triângulo e sejam ha, hb e hc as
>> distâncias de P aos lados a, b e c, respectivamente. Mostre que o valor
>> mínimo de (a/ha) + (b/hb) + (c/hc) ocorre quando P é o incentivo do
>> triângulo ABC.
>>
>> --
>> Esta mensagem foi verificada pelo sistema de antivírus e
>> acredita-se estar livre de perigo.
>
>
> --
> Esta mensagem foi verificada pelo sistema de antivírus e
> acredita-se estar livre de perigo.

-- 
Esta mensagem foi verificada pelo sistema de antiv�rus e
 acredita-se estar livre de perigo.



[obm-l] Re: [obm-l] Re: [obm-l] Re: [obm-l] polinômio irredutível

2020-08-16 Por tôpico Esdras Muniz
Tenta com x^3+9.

Em dom, 16 de ago de 2020 15:24, Claudio Buffara 
escreveu:

> f(x) em Z[x], bem entendido...
>
>
> On Sun, Aug 16, 2020 at 3:08 PM Claudio Buffara 
> wrote:
>
>> Que tal essa aqui?
>> Prove ou disprove que, dado um polinômio f(x), irredutível sobre Q,
>> existe um inteiro N tal que a irredutibilidade de f pode ser provada pelo
>> critério de Eisenstein aplicado a f(x+N).
>>
>> On Sun, Aug 16, 2020 at 2:31 PM Matheus Secco 
>> wrote:
>>
>>> O melhor jeito é pensar na contrapositiva (supondo que você esteja
>>> falando sobre irredutibilidade em Z[x] ou até em Q[x]): se f(x) fatora como
>>> g(x)*h(x), então f(x+a) fatora como g(x+a) *h(x+a) e é claro que uma vez
>>> que g(x) e h(x) têm coeficientes inteiros, então g(x+a) e h(x+a) também
>>> têm. A recíproca é essencialmente idêntica.
>>>
>>> Abraços
>>>
>>> Em dom, 16 de ago de 2020 14:11, Luís Lopes 
>>> escreveu:
>>>
 Sauda,c~oes,

 Como provar que um polinômio f(x) tendo como coeficientes números
 inteiros
 é irredutível se e somente se f(x+a) é irredutível para algum 
 inteiro ?

 Luís




 --
 Esta mensagem foi verificada pelo sistema de antivírus e
 acredita-se estar livre de perigo.

>>>
>>> --
>>> Esta mensagem foi verificada pelo sistema de antivírus e
>>> acredita-se estar livre de perigo.
>>
>>
> --
> Esta mensagem foi verificada pelo sistema de antivírus e
> acredita-se estar livre de perigo.

-- 
Esta mensagem foi verificada pelo sistema de antiv�rus e
 acredita-se estar livre de perigo.



Re: [obm-l] Re: [obm-l] Re: [obm-l] polinômio irredutível

2020-08-16 Por tôpico qedtexte

Sauda,c~oes, oi Cludio,

Que tal essa aqui?
Prove ou disprove que, dado um polinmio f(x), irredutvel sobre Q, existe um inteiro N tal que a 
irredutibilidade de f pode ser provada pelo critrio de Eisenstein aplicado a f(x+N).

Vou esperar a resposta. Pelo exemplo do site

https://mathworld.wolfram.com/EisensteinsIrreducibilityCriterion.html

a gente pode achar que  verdade. O bom seria que esse N e o a da outra mensagem tivessem 
uma faixa de busca. Por tentativa e erro fica difcil.


De qualquer jeito, no tenho conhecimento/experincia nenhuma nessa 
rea.
S acho legal.

Abraos,
Lus


--
Esta mensagem foi verificada pelo sistema de antiv�rus e
acredita-se estar livre de perigo.



Re: [obm-l] Re: [obm-l] Re: [obm-l] polinômio irredutível

2020-08-16 Por tôpico qedtexte

Sauda,c~oes novamente,

Obrigado pelas respostas.

As hipteses so as que vocs falaram: tudo em Z[x].

Na verdade tudo comeou com o problema de saber se f(x)=x^4 + x^3 + 4x + 1 
 irredutvel em Z[x].
Testando a=-1, f(x-1)=x^4 - 3x^3 + 3x^2 + 3x - 3 e agora por Eisenstein com p=3, f(x) 
 irredutvel.

Mas antes precisa do Lema

"Um polinmio f(x) em Z[x]  irredutvel em Z[x] se e somente se f(x+a) 
 irredutvel para algum a
inteiro."


O melhor jeito  pensar na contrapositiva (supondo que voc esteja falando sobre irredutibilidade 
em Z[x] ou at em Q[x]):

Boa, no pensei.


se f(x) fatora como g(x)*h(x), ento f(x+a) fatora como g(x+a)*h(x+a) 
Isso  bvio ? Precisa provar ? Vale dizer que se f(u(x))fatora como g(u(x))*h(u(x)), 
ento f(u(x+a)) fatora como g(u(x+a))*h(u(x+a)) ?


e  claro que uma vez que g(x) e h(x) tm coeficientes inteiros, ento g(x+a) e 
h(x+a) tambm tm. A recproca  essencialmente 
idntica.
Ok.


Abraos,
Lus



--
Esta mensagem foi verificada pelo sistema de antiv�rus e
acredita-se estar livre de perigo.



[obm-l] Re: [obm-l] Re: [obm-l] polinômio irredutível

2020-08-16 Por tôpico Claudio Buffara
f(x) em Z[x], bem entendido...


On Sun, Aug 16, 2020 at 3:08 PM Claudio Buffara 
wrote:

> Que tal essa aqui?
> Prove ou disprove que, dado um polinômio f(x), irredutível sobre Q, existe
> um inteiro N tal que a irredutibilidade de f pode ser provada pelo critério
> de Eisenstein aplicado a f(x+N).
>
> On Sun, Aug 16, 2020 at 2:31 PM Matheus Secco 
> wrote:
>
>> O melhor jeito é pensar na contrapositiva (supondo que você esteja
>> falando sobre irredutibilidade em Z[x] ou até em Q[x]): se f(x) fatora como
>> g(x)*h(x), então f(x+a) fatora como g(x+a) *h(x+a) e é claro que uma vez
>> que g(x) e h(x) têm coeficientes inteiros, então g(x+a) e h(x+a) também
>> têm. A recíproca é essencialmente idêntica.
>>
>> Abraços
>>
>> Em dom, 16 de ago de 2020 14:11, Luís Lopes 
>> escreveu:
>>
>>> Sauda,c~oes,
>>>
>>> Como provar que um polinômio f(x) tendo como coeficientes números
>>> inteiros
>>> é irredutível se e somente se f(x+a) é irredutível para algum 
>>> inteiro ?
>>>
>>> Luís
>>>
>>>
>>>
>>>
>>> --
>>> Esta mensagem foi verificada pelo sistema de antivírus e
>>> acredita-se estar livre de perigo.
>>>
>>
>> --
>> Esta mensagem foi verificada pelo sistema de antivírus e
>> acredita-se estar livre de perigo.
>
>

-- 
Esta mensagem foi verificada pelo sistema de antiv�rus e
 acredita-se estar livre de perigo.



[obm-l] Re: [obm-l] Re: [obm-l] polinômio irredutível

2020-08-16 Por tôpico Claudio Buffara
Que tal essa aqui?
Prove ou disprove que, dado um polinômio f(x), irredutível sobre Q, existe
um inteiro N tal que a irredutibilidade de f pode ser provada pelo critério
de Eisenstein aplicado a f(x+N).

On Sun, Aug 16, 2020 at 2:31 PM Matheus Secco 
wrote:

> O melhor jeito é pensar na contrapositiva (supondo que você esteja falando
> sobre irredutibilidade em Z[x] ou até em Q[x]): se f(x) fatora como
> g(x)*h(x), então f(x+a) fatora como g(x+a) *h(x+a) e é claro que uma vez
> que g(x) e h(x) têm coeficientes inteiros, então g(x+a) e h(x+a) também
> têm. A recíproca é essencialmente idêntica.
>
> Abraços
>
> Em dom, 16 de ago de 2020 14:11, Luís Lopes 
> escreveu:
>
>> Sauda,c~oes,
>>
>> Como provar que um polinômio f(x) tendo como coeficientes números
>> inteiros
>> é irredutível se e somente se f(x+a) é irredutível para algum  inteiro
>> ?
>>
>> Luís
>>
>>
>>
>>
>> --
>> Esta mensagem foi verificada pelo sistema de antivírus e
>> acredita-se estar livre de perigo.
>>
>
> --
> Esta mensagem foi verificada pelo sistema de antivírus e
> acredita-se estar livre de perigo.

-- 
Esta mensagem foi verificada pelo sistema de antiv�rus e
 acredita-se estar livre de perigo.



[obm-l] Re: [obm-l] polinômio irredutível

2020-08-16 Por tôpico Matheus Secco
O melhor jeito é pensar na contrapositiva (supondo que você esteja falando
sobre irredutibilidade em Z[x] ou até em Q[x]): se f(x) fatora como
g(x)*h(x), então f(x+a) fatora como g(x+a) *h(x+a) e é claro que uma vez
que g(x) e h(x) têm coeficientes inteiros, então g(x+a) e h(x+a) também
têm. A recíproca é essencialmente idêntica.

Abraços

Em dom, 16 de ago de 2020 14:11, Luís Lopes 
escreveu:

> Sauda,c~oes,
>
> Como provar que um polinômio f(x) tendo como coeficientes números inteiros
> é irredutível se e somente se f(x+a) é irredutível para algum  inteiro
> ?
>
> Luís
>
>
>
>
> --
> Esta mensagem foi verificada pelo sistema de antivírus e
> acredita-se estar livre de perigo.
>

-- 
Esta mensagem foi verificada pelo sistema de antiv�rus e
 acredita-se estar livre de perigo.



[obm-l] Re: [obm-l] Geometria plana com desigualdade de médias?

2020-08-16 Por tôpico Matheus Secco
Olá, Vanderlei.
Por Cauchy-Schwarz, temos

(a/ha + b/hb + c/hc) * (a*ha + b*hb + c*hc) >= (a+b+c)^2.  (#)

Como (a*ha + b*hb + c*hc) = 2S, onde S é a área de ABC, segue que a
expressão a/ha + b/hb + c/hc é pelo menos 2p^2/S, onde p é o
semi-perimetro.

Por outro lado, a igualdade em (#) ocorre se, e somente se, ha = hb = hc,
ou seja, quando P é o incentro do triângulo

Abraços,
Matheus

Em dom, 16 de ago de 2020 08:59, Professor Vanderlei Nemitz <
vanderma...@gmail.com> escreveu:

> Bom dia!
>
> Tentei utilizar alguma desigualdade de médias aqui, mas não tive êxito.
> Alguém ajuda?
> Muito agradecido!
>
> Seja P um ponto no interior de um triângulo e sejam ha, hb e hc as
> distâncias de P aos lados a, b e c, respectivamente. Mostre que o valor
> mínimo de (a/ha) + (b/hb) + (c/hc) ocorre quando P é o incentivo do
> triângulo ABC.
>
> --
> Esta mensagem foi verificada pelo sistema de antivírus e
> acredita-se estar livre de perigo.

-- 
Esta mensagem foi verificada pelo sistema de antiv�rus e
 acredita-se estar livre de perigo.



[obm-l] Re: [obm-l] Re: [obm-l] Re: [obm-l] Mostrar que está função não existe

2020-08-11 Por tôpico Esdras Muniz
Depois de ver essa solicitação genial, fiquei com vergonha de mandar a
minha.

Em ter, 11 de ago de 2020 01:37, Ralph Costa Teixeira 
escreveu:

> Acho que isso caiu numa IMO que eu fiz Ah, achei, 1987. Aqui tem uma
> resposta bem legal:
>
>
> https://math.stackexchange.com/questions/325504/imo-1987-function-such-that-ffn-n1987
>
>
> On Tue, Aug 11, 2020 at 12:50 AM  wrote:
>
>> É, fatou dizer que k é ímpar
>>
>> Artur
>>
>> Em 10 de ago de 2020 22:33, Ralph Costa Teixeira 
>> escreveu:
>>
>> K inteiro... ímpar? Porque tomando f(n)=n+k/2...
>>
>> On Mon, Aug 10, 2020, 22:05 Artur Costa Steiner <
>> artur.costa.stei...@gmail.com> wrote:
>>
>> Me pareceu que isto era simples, mas segui um caminho errado e ainda não
>> cheguei lá.
>>
>> Mostre que não existe f:N --> N, N os naturais com o 0, tal que f(f(n)) =
>> n + k, k > 0 inteiro.
>>
>> Obrigado
>>
>> Artur
>>
>> --
>> Esta mensagem foi verificada pelo sistema de antivírus e
>> acredita-se estar livre de perigo.
>>
>>
>> --
>> Esta mensagem foi verificada pelo sistema de antiv�rus e
>> acredita-se estar livre de perigo.
>>
>>
>>
>> --
>> Esta mensagem foi verificada pelo sistema de antivírus e
>> acredita-se estar livre de perigo.
>
>
> --
> Esta mensagem foi verificada pelo sistema de antivírus e
> acredita-se estar livre de perigo.

-- 
Esta mensagem foi verificada pelo sistema de antiv�rus e
 acredita-se estar livre de perigo.



Re: [obm-l] Re: [obm-l] Mostrar que está função não existe

2020-08-11 Por tôpico Artur_steiner
É, fatou dizer que k é ímparArturEm 10 de ago de 2020 22:33, Ralph Costa Teixeira  escreveu:K inteiro... ímpar? Porque tomando f(n)=n+k/2...On Mon, Aug 10, 2020, 22:05 Artur Costa Steiner  wrote:Me pareceu que isto era simples, mas segui um caminho errado e ainda não cheguei lá. Mostre que não existe f:N --> N, N os naturais com o 0, tal que f(f(n)) = n + k, k > 0 inteiro. Obrigado Artur
--
Esta mensagem foi verificada pelo sistema de antivírus e 
 acredita-se estar livre de perigo.


--
Esta mensagem foi verificada pelo sistema de antiv�rus e 
 acredita-se estar livre de perigo.

--
Esta mensagem foi verificada pelo sistema de antivírus e 
 acredita-se estar livre de perigo.




[obm-l] Re: [obm-l] Re: [obm-l] Mostrar que está função não existe

2020-08-10 Por tôpico Ralph Costa Teixeira
Acho que isso caiu numa IMO que eu fiz Ah, achei, 1987. Aqui tem uma
resposta bem legal:

https://math.stackexchange.com/questions/325504/imo-1987-function-such-that-ffn-n1987


On Tue, Aug 11, 2020 at 12:50 AM  wrote:

> É, fatou dizer que k é ímpar
>
> Artur
>
> Em 10 de ago de 2020 22:33, Ralph Costa Teixeira 
> escreveu:
>
> K inteiro... ímpar? Porque tomando f(n)=n+k/2...
>
> On Mon, Aug 10, 2020, 22:05 Artur Costa Steiner <
> artur.costa.stei...@gmail.com> wrote:
>
> Me pareceu que isto era simples, mas segui um caminho errado e ainda não
> cheguei lá.
>
> Mostre que não existe f:N --> N, N os naturais com o 0, tal que f(f(n)) =
> n + k, k > 0 inteiro.
>
> Obrigado
>
> Artur
>
> --
> Esta mensagem foi verificada pelo sistema de antivírus e
> acredita-se estar livre de perigo.
>
>
> --
> Esta mensagem foi verificada pelo sistema de antiv�rus e
> acredita-se estar livre de perigo.
>
>
>
> --
> Esta mensagem foi verificada pelo sistema de antivírus e
> acredita-se estar livre de perigo.

-- 
Esta mensagem foi verificada pelo sistema de antiv�rus e
 acredita-se estar livre de perigo.



[obm-l] Re: [obm-l] Mostrar que está função não existe

2020-08-10 Por tôpico Ralph Costa Teixeira
K inteiro... ímpar? Porque tomando f(n)=n+k/2...

On Mon, Aug 10, 2020, 22:05 Artur Costa Steiner <
artur.costa.stei...@gmail.com> wrote:

> Me pareceu que isto era simples, mas segui um caminho errado e ainda não
> cheguei lá.
>
> Mostre que não existe f:N --> N, N os naturais com o 0, tal que f(f(n)) =
> n + k, k > 0 inteiro.
>
> Obrigado
>
> Artur
>
> --
> Esta mensagem foi verificada pelo sistema de antivírus e
> acredita-se estar livre de perigo.

-- 
Esta mensagem foi verificada pelo sistema de antiv�rus e
 acredita-se estar livre de perigo.



Re: [obm-l] Re: [obm-l] Re: [obm-l] polinômio minimal

2020-08-09 Por tôpico qedtexte
Sauda,c~oes, oi Joo Pedro, 

Certo. Mas se a gente no souber que  minimal ? 


Lus


--
Esta mensagem foi verificada pelo sistema de antiv�rus e
acredita-se estar livre de perigo.



Re: [obm-l] Re: [obm-l] polinômio minimal

2020-08-09 Por tôpico qedtexte

Sauda,c~oes, oi Joo Pedro,

Obrigado por responder.

Tinha feito isso. Deu

x^8 - 12 x^6 + 32 x^4 - 72 x^2 + 4

expand (x + 1)^8 - 12 (x + 1)^6 + 32 (x + 1)^4 - 72 (x + 1)^2 + 4

x^8 + 8 x^7 + 16 x^6 - 16 x^5 - 78 x^4 - 56 x^3 - 32 x^2 - 80 x - 47

E oCritrio de Eisensteinno se aplica.

E para x=x+2, deu

x^8 + 16 x^7 + 100 x^6 + 304 x^5 + 432 x^4 + 128 x^3 - 392 x^2 - 544 x - 284

Quase. Falha no a_0=284.

Lus




--
Esta mensagem foi verificada pelo sistema de antiv�rus e
acredita-se estar livre de perigo.



[obm-l] Re: [obm-l] Re: [obm-l] polinômio minimal

2020-08-09 Por tôpico João Pedro de Abreu Marciano
Verdade...
Seja p = x^8 - 12 x^6 + 32 x^4 - 72 x^2 + 4 um polinômio minimal de α,
então não pode haver polinômio de grau menor que 8 com α sendo raiz.
Suponha que p não é irredutível. Logo, existem g,h tais que  p = g*h, com
0
escreveu:

> Sauda,c~oes, oi João Pedro,
>
> Obrigado por responder.
>
> Tinha feito isso. Deu
>
> 
>
> expand  (x + 1)^8 - 12 (x + 1)^6 + 32 (x + 1)^4 - 72 (x + 1)^2 + 4
>
> x^8 + 8 x^7 + 16 x^6 - 16 x^5 - 78 x^4 - 56 x^3 - 32 x^2 - 80 x - 47
>
> E o Critério de Eisenstein não se aplica.
>
> E para x=x+2, deu
>
> x^8 + 16 x^7 + 100 x^6 + 304 x^5 + 432 x^4 + 128 x^3 - 392 x^2 - 544 x -
> 284
>
> Quase. Falha no a_0=284.
>
> Luís
>
>
>

-- 
Esta mensagem foi verificada pelo sistema de antiv�rus e
 acredita-se estar livre de perigo.



Re: [obm-l] teste

2020-08-08 Por tôpico Carlos Victor
 

Há muito tempo que os meus emails enviados também estão assim e não sei
o motivo. 

Carlos Victor 

PS : este email não sei se chegará aos companheiros da lista 

Em 08/08/2020 17:39, Luís Lopes escreveu: 

> Recebo as mensagens normalmente. Mas não tenho confirmação de 
> chegada ao grupo das que envio. E não aparecem nos arquivos também. 
> 
> Mandei uma há umas 5 horas intitulada polinômio minimal. 
> Chegou? Alguém recebeu ? 
> 
> Luís 
> 
> -- 
> Esta mensagem foi verificada pelo sistema de antivírus e 
> acredita-se estar livre de perigo.

 
-- 
Esta mensagem foi verificada pelo sistema de antiv�rus e
 acredita-se estar livre de perigo.



[obm-l] Re: [obm-l] polinômio minimal

2020-08-08 Por tôpico João Pedro de Abreu Marciano
Boa noite!

Tente aplicar o Critério de Eisenstein com p=3 e substituindo x por x+1.

Att.
João Pedro.

Em sáb., 8 de ago. de 2020 às 17:14, 
escreveu:

> Sauda,c~oes,
>
> O polinômio 
> é o polinômio minimal de α = sqrt(2) + sqrt(1+sqrt(3)).
>
> Como provar que ele é irredutível em Q[x] ?
>
> Luís
>
> --
> Esta mensagem foi verificada pelo sistema de antivírus e
> acredita-se estar livre de perigo.

-- 
Esta mensagem foi verificada pelo sistema de antiv�rus e
 acredita-se estar livre de perigo.



Re: [obm-l] teste

2020-08-08 Por tôpico Luiz Kv
chegou

Em sáb, 8 de ago de 2020 18:20, Carlos Victor 
escreveu:

> Há muito tempo que os meus emails enviados também estão assim e não sei o
> motivo.
>
> Carlos Victor
>
> PS : este email não sei se chegará aos companheiros da lista
>
> Em 08/08/2020 17:39, Luís Lopes escreveu:
>
> Recebo as mensagens normalmente. Mas não tenho confirmação de
> chegada ao grupo das que envio. E não aparecem nos arquivos também.
>
> Mandei uma há umas 5 horas intitulada polinômio minimal.
> Chegou? Alguém recebeu ?
>
> Luís
>
>
> --
> Esta mensagem foi verificada pelo sistema de antivírus e
> acredita-se estar livre de perigo.
>
>
>
> --
> Esta mensagem foi verificada pelo sistema de antivírus e
> acredita-se estar livre de perigo.
>

-- 
Esta mensagem foi verificada pelo sistema de antiv�rus e
 acredita-se estar livre de perigo.



RE: [obm-l] teste

2020-08-08 Por tôpico Luís Lopes
Sauda,c~oes,

Oi Carlos Victor,

Obrigado por responder. Acho que recebi sua mensagem pois vc a enviou
tanto para a lista quanto para mim diretamente. E do mesmo jeito, acho que
você vai receber minha resposta somente porque estou escrevendo
diretamente pra você, com cópia pros dois endereços da obm.puc.

Isso já acontece comigo há muito, muito tempo. No começo quando fazia reply,
agora quando mando uma mensagem nova. Às vezes mando as mensagens por
um outro email, e essas eu vejo chegarem no email que mandei. Mas não chegam
nesse daqui. E muito certamente não chegam na lista também.

Se por acaso alguém da lista receber esta mensagem e puder entrar em contato
com o administrador, ficaria muito grato.

Obrigado.

Luís





De: Carlos Victor 
Enviado: sábado, 8 de agosto de 2020 17:01
Para: obm-l@mat.puc-rio.br 
Cc: owner-ob...@mat.puc-rio.br ; Luís Lopes 

Assunto: Re: [obm-l] teste


Há muito tempo que os meus emails enviados também estão assim e não sei o 
motivo.

Carlos Victor

PS : este email não sei se chegará aos companheiros da lista

Em 08/08/2020 17:39, Luís Lopes escreveu:

Recebo as mensagens normalmente. Mas não tenho confirmação de
chegada ao grupo das que envio. E não aparecem nos arquivos também.

Mandei uma há umas 5 horas intitulada polinômio minimal.
Chegou? Alguém recebeu ?

Luís


--
Esta mensagem foi verificada pelo sistema de antivírus e
acredita-se estar livre de perigo.


-- 
Esta mensagem foi verificada pelo sistema de antivírus e
 acredita-se estar livre de perigo.



[obm-l] Re: [obm-l] Re: [obm-l] Re: [obm-l] Álgebra

2020-08-05 Por tôpico Esdras Muniz
Acho que dá -2. Usa que (x+y)^2=xy e (x/y)^3=1.

Em qua, 5 de ago de 2020 20:07, Anderson Torres <
torres.anderson...@gmail.com> escreveu:

> Em ter., 14 de jul. de 2020 às 23:39, Pacini Bores
>  escreveu:
> >
> > A expressão pedida ao quadrado é igual a 4, sem usar complexos.
> >
> > Pacini
> >
> > Em 14/07/2020 21:50, marcone augusto araújo borges escreveu:
> >
> > Se x^2 +xy + y^2  = 0, com x,y <>0
> > Determinar (x/(x+y))^2019 + (y/(x+y))^2019, sem usar números complexos.
>
> Bem, é meio óbvio que x!=y e x!=-y, senão daria 0.
>
> Podemos supor sem perda de generalidade que x+y=1 (basta dividir x e y
> pela soma)
>
> Assim, temos x+y=1 e x^2+2xy+y^2=1, portanto xy=1.
>
> Assim x e y são zeros do polinômio P(x)=x^2-x+1, e x^2019+y^2019 seria
> calculável mediante uma recorrência.
>
>
> >
> > --
> > Esta mensagem foi verificada pelo sistema de antivírus e
> > acredita-se estar livre de perigo.
> >
> >
> >
> > --
> > Esta mensagem foi verificada pelo sistema de antivírus e
> > acredita-se estar livre de perigo.
>
> --
> Esta mensagem foi verificada pelo sistema de antivírus e
>  acredita-se estar livre de perigo.
>
>
> =
> Instru�ões para entrar na lista, sair da lista e usar a lista em
> http://www.mat.puc-rio.br/~obmlistas/obm-l.html
> =
>

-- 
Esta mensagem foi verificada pelo sistema de antiv�rus e
 acredita-se estar livre de perigo.



[obm-l] Re: [obm-l] Re: [obm-l] Álgebra

2020-08-05 Por tôpico Anderson Torres
Em ter., 14 de jul. de 2020 às 23:39, Pacini Bores
 escreveu:
>
> A expressão pedida ao quadrado é igual a 4, sem usar complexos.
>
> Pacini
>
> Em 14/07/2020 21:50, marcone augusto araújo borges escreveu:
>
> Se x^2 +xy + y^2  = 0, com x,y <>0
> Determinar (x/(x+y))^2019 + (y/(x+y))^2019, sem usar números complexos.

Bem, é meio óbvio que x!=y e x!=-y, senão daria 0.

Podemos supor sem perda de generalidade que x+y=1 (basta dividir x e y
pela soma)

Assim, temos x+y=1 e x^2+2xy+y^2=1, portanto xy=1.

Assim x e y são zeros do polinômio P(x)=x^2-x+1, e x^2019+y^2019 seria
calculável mediante uma recorrência.


>
> --
> Esta mensagem foi verificada pelo sistema de antivírus e
> acredita-se estar livre de perigo.
>
>
>
> --
> Esta mensagem foi verificada pelo sistema de antivírus e
> acredita-se estar livre de perigo.

-- 
Esta mensagem foi verificada pelo sistema de antiv�rus e
 acredita-se estar livre de perigo.


=
Instru��es para entrar na lista, sair da lista e usar a lista em
http://www.mat.puc-rio.br/~obmlistas/obm-l.html
=


[obm-l] Re: [obm-l] Funções complexas - número de zeros em C

2020-07-31 Por tôpico Artur Costa Steiner
A minha conclusão de que f e g não têm zeros em C sse f = g está
equivocada. É verdade que se f e g não têm zeros então f = g. Mas a
recíproca não é verdadeira

>
>> Artur
>>
>

-- 
Esta mensagem foi verificada pelo sistema de antiv�rus e
 acredita-se estar livre de perigo.



[obm-l] Re: [obm-l] Funções complexas - número de zeros em C

2020-07-30 Por tôpico Artur Costa Steiner
Naquele meu outro post, houve um equívoco no enunciado do T. de Rouché.

> A desigualdade
>
> |f(z) - g(z)| < |f(z)| + |g(z)|
>
> tem que valer apenas no traço W* da curva.
>
> Artur
>

-- 
Esta mensagem foi verificada pelo sistema de antiv�rus e
 acredita-se estar livre de perigo.



[obm-l] Re: [obm-l] Re: [obm-l] Funções complexas - número de zeros em C

2020-07-30 Por tôpico Artur Costa Steiner
Em qui, 30 de jul de 2020 16:22, Artur Costa Steiner 
escreveu:

> Também me ocorreu isso, mas depois pensei em me basear no T. de Rouché (o
> da Análise Complexa, não o da Álgebra Linear). Em sua forma geral, este
> teorema diz:
>
> Se V um aberto do plano e W  uma curva suave e fechada em V tal que Ind(W,
> z) = 0 ou 1 para z em V/W* (o traço de W) e = 0 para z em C/V. Se f e g são
> funções holomorfas em V tais que
>
> |f(z) - g(z)| < |f(z)| + |g(z)|
>
> para z em V/W*, então f e g têm o mesmo número de zeros no conjunto I = {z
> em V/W* : Ind(W,z) = 1-}
>
> Este teorema é particularmente interessante quando W é uma curva bonita em
> que se identifica claramente a área delimitada por W. Neste ,caso I é essa
> área. Caso típico do círculo de centro a e raio r,  quando então I é o
> disco aberto D(a, r).
>
> No caso, vemos que, para que o limite dado faça sentido,  para |z|
> suficientemente grande g não pode se anular.  Logo, o conjunto Zg dos zeros
> de g é limitado. Se Zg for infinito, terá um ponto de acumulação em C, o
> domínio da inteira g. Pelas propriedades das funções holomorfas, g será
> entãoidenticamente nula, conflitando assim com o limite dado.  Logo, Zg é
> finito, tendo n>= 0 elementos (contando multiplicidades).
>
> Como lim |z| --> oo f(z)/g(z) = 1, então lim |z| --> oo f(z)/g(z) - 1 = lim
> |z| --> oo [f(z) - g(z)]/g(z) = 0. Segue-se que lim |z| --> oo |f(z) -
> g(z|]/|g(z)) = 0, havendo portanto r0 > 0 tal que |z| > r0 => |f(z) -
> g(z)|/|g(z)| < 1 e, portanto,
>
> |f(z) - g(z)| < ||g(z)| (1)
>
> Assim, para r > r0, (1) vale no círculo de centro na origem e raio r,
> periferia de D(0 r). Pelo T de Rouché  com V = C, f e g têm neste disco o
> mesmo número de zeros , ou seja, n zeros.
>
> Como isso vale pra todo r > r0, concluímos que f e f têm n zeros em C.
>
> Se f for um polinômio de grau n > 0 e g(z) = c_n z^n, c_n o coeficiente
> líder de f, temos uma prova do T. Fundamental da Álgebra que mostra que
> polinômios de grau n > 0 têm n zeros (ou raízes).
>
> Neste problema, cheguei a duas conclusões me parecem corretas:
>
> f e g não terão zeros em C esse f = g, g uma função que não se anule
>
> Se lim |z| --> oo g(z) = oo, então f e g são polinômios de mesmo grau
> positivo.
>
> Abs
>
> Artur
>
>
> Em qui, 30 de jul de 2020 13:43, Claudio Buffara <
> claudio.buff...@gmail.com> escreveu:
>
>> Será que fazendo w = 1/z  e  w -> 0 ajuda?
>>
>> On Thu, Jul 30, 2020 at 7:24 AM Artur Costa Steiner <
>> artur.costa.stei...@gmail.com> wrote:
>>
>>> Sejam f e g funções inteiras tais que lim |z| ---> oo f(z)/g(z) = 1.
>>> Mostre que f e g tem um número finito de zeros em C e que o número de zeros
>>> de f é igual ao número de zeros de  g.
>>>
>>> Abs
>>>
>>> Artur
>>>
>>> --
>>> Esta mensagem foi verificada pelo sistema de antivírus e
>>> acredita-se estar livre de perigo.
>>
>>
>> --
>> Esta mensagem foi verificada pelo sistema de antivírus e
>> acredita-se estar livre de perigo.
>
>

-- 
Esta mensagem foi verificada pelo sistema de antiv�rus e
 acredita-se estar livre de perigo.



[obm-l] Re: [obm-l] Re: [obm-l] Funções complexas - número de zeros em C

2020-07-30 Por tôpico Artur Costa Steiner
Também me ocorreu isso, mas depois pensei em me basear no T. de Rouché (o
da Análise Complexa, não o da Álgebra Linear). Em sua forma geral, este
teorema diz:

Se V um aberto do plano e W  uma curva suave e fechada em V tal que Ind(W,
z) = 0 ou 1 para z em V/W* (o traço de W) e = 0 para z em C/V. Se f e g são
funções holomorfas em V tais que

|f(z) - g(z)| < |f(z)| + |g(z)|

para z em V/W*, então f e g têm o mesmo número de zeros no conjunto I = {z
em V/W* : Ind(W,z) = 1-}

Este teorema é particularmente interessante quando W é uma curva bonita em
que se identifica claramente a área delimitada por W. Neste ,caso I é essa
área. Caso típico do círculo de centro a e raio r,  quando então I é o
disco aberto D(a, r).

No caso, vemos que, para que o limite dado faça sentido,  para |z|
suficientemente grande g não pode se anular.  Logo, o conjunto Zg dos zeros
de g é limitado. Se Zg for infinito, terá um ponto de acumulação em C, o
domínio da inteira g. Pelas propriedades das funções holomorfas, g será
entãoidenticamente nula, conflitando assim com o limite dado.  Logo, Zg é
finito, tendo n>= 0 elementos (contando multiplicidades).

Como lim |z| --> oo f(z)/g(z) = 1, então lim |z| --> oo f(z)/g(z) - 1 = lim
|z| --> oo [f(z) - g(z)]/g(z) = 0. Segue-se que lim |z| --> oo |f(z) -
g(z|]/|g(z)) = 0, havendo portanto r0 > 0 tal que |z| > r0 => |f(z) -
g(z)|/|g(z)| < 1 e, portanto,

|f(z) - g(z)| < ||g(z)| (1)

Assim, para r > r0, (1) vale no círculo de centro na origem e raio r,
periferia de D(0 r). Pelo T de Rouché  com V = C, f e g têm neste disco o
mesmo número de zeros , ou seja, n zeros.

Como isso vale pra todo r > r0, concluímos que f e f têm n zeros em C.

Se f for um polinômio de grau n > 0 e g(z) = c_n z^n, c_n o coeficiente
líder de f, temos uma prova do T. Fundamental da Álgebra que mostra que
polinômios de grau n > 0 têm n zeros (ou raízes).

Neste problema, cheguei a duas conclusões me parecem corretas:

f e g não terão zeros em C esse f = g, g uma função que não se anule

Se lim |z| --> oo g(z) = oo, então f e g são polinômios de mesmo grau
positivo.

Abs

Artur


Em qui, 30 de jul de 2020 13:43, Claudio Buffara 
escreveu:

> Será que fazendo w = 1/z  e  w -> 0 ajuda?
>
> On Thu, Jul 30, 2020 at 7:24 AM Artur Costa Steiner <
> artur.costa.stei...@gmail.com> wrote:
>
>> Sejam f e g funções inteiras tais que lim |z| ---> oo f(z)/g(z) = 1.
>> Mostre que f e g tem um número finito de zeros em C e que o número de zeros
>> de f é igual ao número de zeros de  g.
>>
>> Abs
>>
>> Artur
>>
>> --
>> Esta mensagem foi verificada pelo sistema de antivírus e
>> acredita-se estar livre de perigo.
>
>
> --
> Esta mensagem foi verificada pelo sistema de antivírus e
> acredita-se estar livre de perigo.

-- 
Esta mensagem foi verificada pelo sistema de antiv�rus e
 acredita-se estar livre de perigo.



[obm-l] Re: [obm-l] Funções complexas - número de zeros em C

2020-07-30 Por tôpico Claudio Buffara
Será que fazendo w = 1/z  e  w -> 0 ajuda?

On Thu, Jul 30, 2020 at 7:24 AM Artur Costa Steiner <
artur.costa.stei...@gmail.com> wrote:

> Sejam f e g funções inteiras tais que lim |z| ---> oo f(z)/g(z) = 1.
> Mostre que f e g tem um número finito de zeros em C e que o número de zeros
> de f é igual ao número de zeros de  g.
>
> Abs
>
> Artur
>
> --
> Esta mensagem foi verificada pelo sistema de antivírus e
> acredita-se estar livre de perigo.

-- 
Esta mensagem foi verificada pelo sistema de antiv�rus e
 acredita-se estar livre de perigo.



Re: [obm-l] Desafio de probabilidade

2020-07-25 Por tôpico Claudio Buffara
É isso mesmo. Tem que sair 3 vezes o MESMO NÚMERO e não 3 vezes a MESMA
PARIDADE.

[]s,
Claudio.

On Sat, Jul 25, 2020 at 3:53 PM Ralph Costa Teixeira 
wrote:

> Oi, Claudio
>
> Eu também pensei em trocar o dado por uma moeda, mas se entendi bem o
> enunciado, não podemos! O problema eh que, se o dado der 2,4,6,2,4,6,1,1,1,
> quem ganha eh Umberto; trocando pela moeda, vemos par,par,par e vamos dar o
> trofeu para o Ze Roberto... Muda o jogo!
>
> On Sat, Jul 25, 2020 at 3:24 PM Claudio Buffara 
> wrote:
>
>> Pra facilitar, podemos substituir o dado por uma moeda, com cara = par =
>> 0 e coroa = ímpar = 1, já que o que importa é apenas a paridade do número
>> na face superior do dado lançado e, neste caso, P(par) = P(ímpar) = 1/2.
>>
>> Como 3 caras seguidas ou 3 coroas seguidas encerra o jogo, basta
>> considerar os dois últimos lançamentos.
>>
>> Suponha que dois lançamentos seguidos tenham sido 1 e 0 (cara e coroa).
>> Após sair o 0, digamos que a probabilidade de ZR vencer seja p.
>>
>> Se o terceiro lançamento for 0, a probabilidade de ZR vencer aumentará
>> para q  (p e q são incógnitas a serem determinadas), e q é justamente a
>> probabilidade desejada, já que é a probabilidade de ZR vencer dado que os
>> dois últimos lançamentos foram 0 e 0.
>> Se o quarto lançamento for 0, ZR vence. Mas se for 1, sua probabilidade
>> de vencer cai para 1-p pois, neste caso, por simetria, Umberto passa a ter
>> probabilidade p de vencer, em virtude dos dois últimos lançamentos terem
>> sido 0 e 1.
>> Ou seja, q = (1/2)*(1 + (1-p))  <==>  p + 2q = 2.
>>
>> Se o terceiro lançamento for 1, a probabilidade de ZR vencer cai para 1-p.
>> Neste caso, podemos escrever p = (1/2)*(q + (1-p))  <==>  3p - q = 1.
>>
>> Resolvendo este sistema, achamos p = 4/7 e q = 5/7.
>>
>> Na verdade, isso tudo fica mais fácil de ver se você fizer uma árvore.
>>
>> []s,
>> Claudio.
>>
>> On Sat, Jul 25, 2020 at 2:03 PM Professor Vanderlei Nemitz <
>> vanderma...@gmail.com> wrote:
>>
>>> Então meu raciocínio foi muito errado, pois pensei assim:
>>> Seja p a probabilidade de Zé Roberto vender. Podemos considerar que o
>>> jogo "começa" com Zé Roberto precisando obter um 6 para vencer.
>>> Assim, a probabilidade de Humberto vencer é:
>>> q = (3/6).(1/6).p, ou seja, p = 12q
>>> Assim, p = 12/13 e q = 1/13
>>>
>>> Prezado Cláudio, você pode explicar sua resolução?
>>>
>>> Muito obrigado!
>>>
>>>
>>>
>>>
>>>
>>>
>>> Em sáb., 25 de jul. de 2020 às 13:43, Claudio Buffara <
>>> claudio.buff...@gmail.com> escreveu:
>>>
 Eu achei 5/7.

 On Sat, Jul 25, 2020 at 7:28 AM Professor Vanderlei Nemitz <
 vanderma...@gmail.com> wrote:

> Bom dia!
> O problema a seguir encontra-se em uma prova de desafios da PUC-RJ,
> muito boas!!!
> Acho que são organizadas pelo professor Nicolau Saldanha.
> Encontrei uma resposta bem alta, mais de 90%. Será que está correto?
> Muito obrigado!
>
> Zé Roberto e Umberto gostam de jogar par ou ímpar; Zé Roberto sempre
> pede par e Umberto sempre pede íımpar. Eles gostam de inventar novas
> maneiras de jogar. A última maneira que eles inventaram usa um dado comum,
> com seis faces numeradas de 1 a 6. Eles jogam o dado várias vezes até que
> um número saia três vezes seguidas; Zé Roberto ganha se este número for
> par, Umberto ganha se for ímpar. Sábado de manhã o dado teve os 
> resultados:
> 5, 3, 4, 2, 6, 1, 1, 3, 1, 4, 2, 3, 5, 6, 3, 4, 5, 4, 4, 4 e neste ponto 
> Zé
> Roberto se declarou vitorioso. Sábado de tarde o dado teve os resultados:
> 6, 1, 4, 2, 3, 5, 6, 6; neste momento o jogo foi interrompido pela queda 
> de
> um meteorito. Quando a situação se acalmou, eles concordaram em continuar
> do ponto em que estavam. Qual é a probabilidade de que Zé Roberto seja o
> vencedor?
>
> --
> Esta mensagem foi verificada pelo sistema de antivírus e
> acredita-se estar livre de perigo.


 --
 Esta mensagem foi verificada pelo sistema de antivírus e
 acredita-se estar livre de perigo.
>>>
>>>
>>> --
>>> Esta mensagem foi verificada pelo sistema de antivírus e
>>> acredita-se estar livre de perigo.
>>
>>
>> --
>> Esta mensagem foi verificada pelo sistema de antivírus e
>> acredita-se estar livre de perigo.
>
>
> --
> Esta mensagem foi verificada pelo sistema de antivírus e
> acredita-se estar livre de perigo.

-- 
Esta mensagem foi verificada pelo sistema de antiv�rus e
 acredita-se estar livre de perigo.



Re: [obm-l] Desafio de probabilidade

2020-07-25 Por tôpico Claudio Buffara
Por favor desconsiderem.
Reli o enunciado e vi que errei.
Pro ZR ganhar, tem que sair o mesmo número par 3 vezes seguidas.
E minha solução é para o caso (bem mais fácil!) em que ele ganha se saírem
3 números pares seguidos.

[]s,
Claudio.


On Sat, Jul 25, 2020 at 3:08 PM Claudio Buffara 
wrote:

> Pra facilitar, podemos substituir o dado por uma moeda, com cara = par = 0
> e coroa = ímpar = 1, já que o que importa é apenas a paridade do número na
> face superior do dado lançado e, neste caso, P(par) = P(ímpar) = 1/2.
>
> Como 3 caras seguidas ou 3 coroas seguidas encerra o jogo, basta
> considerar os dois últimos lançamentos.
>
> Suponha que dois lançamentos seguidos tenham sido 1 e 0 (cara e coroa).
> Após sair o 0, digamos que a probabilidade de ZR vencer seja p.
>
> Se o terceiro lançamento for 0, a probabilidade de ZR vencer aumentará
> para q  (p e q são incógnitas a serem determinadas), e q é justamente a
> probabilidade desejada, já que é a probabilidade de ZR vencer dado que os
> dois últimos lançamentos foram 0 e 0.
> Se o quarto lançamento for 0, ZR vence. Mas se for 1, sua probabilidade de
> vencer cai para 1-p pois, neste caso, por simetria, Umberto passa a ter
> probabilidade p de vencer, em virtude dos dois últimos lançamentos terem
> sido 0 e 1.
> Ou seja, q = (1/2)*(1 + (1-p))  <==>  p + 2q = 2.
>
> Se o terceiro lançamento for 1, a probabilidade de ZR vencer cai para 1-p.
> Neste caso, podemos escrever p = (1/2)*(q + (1-p))  <==>  3p - q = 1.
>
> Resolvendo este sistema, achamos p = 4/7 e q = 5/7.
>
> Na verdade, isso tudo fica mais fácil de ver se você fizer uma árvore.
>
> []s,
> Claudio.
>
> On Sat, Jul 25, 2020 at 2:03 PM Professor Vanderlei Nemitz <
> vanderma...@gmail.com> wrote:
>
>> Então meu raciocínio foi muito errado, pois pensei assim:
>> Seja p a probabilidade de Zé Roberto vender. Podemos considerar que o
>> jogo "começa" com Zé Roberto precisando obter um 6 para vencer.
>> Assim, a probabilidade de Humberto vencer é:
>> q = (3/6).(1/6).p, ou seja, p = 12q
>> Assim, p = 12/13 e q = 1/13
>>
>> Prezado Cláudio, você pode explicar sua resolução?
>>
>> Muito obrigado!
>>
>>
>>
>>
>>
>>
>> Em sáb., 25 de jul. de 2020 às 13:43, Claudio Buffara <
>> claudio.buff...@gmail.com> escreveu:
>>
>>> Eu achei 5/7.
>>>
>>> On Sat, Jul 25, 2020 at 7:28 AM Professor Vanderlei Nemitz <
>>> vanderma...@gmail.com> wrote:
>>>
 Bom dia!
 O problema a seguir encontra-se em uma prova de desafios da PUC-RJ,
 muito boas!!!
 Acho que são organizadas pelo professor Nicolau Saldanha.
 Encontrei uma resposta bem alta, mais de 90%. Será que está correto?
 Muito obrigado!

 Zé Roberto e Umberto gostam de jogar par ou ímpar; Zé Roberto sempre
 pede par e Umberto sempre pede íımpar. Eles gostam de inventar novas
 maneiras de jogar. A última maneira que eles inventaram usa um dado comum,
 com seis faces numeradas de 1 a 6. Eles jogam o dado várias vezes até que
 um número saia três vezes seguidas; Zé Roberto ganha se este número for
 par, Umberto ganha se for ímpar. Sábado de manhã o dado teve os resultados:
 5, 3, 4, 2, 6, 1, 1, 3, 1, 4, 2, 3, 5, 6, 3, 4, 5, 4, 4, 4 e neste ponto Zé
 Roberto se declarou vitorioso. Sábado de tarde o dado teve os resultados:
 6, 1, 4, 2, 3, 5, 6, 6; neste momento o jogo foi interrompido pela queda de
 um meteorito. Quando a situação se acalmou, eles concordaram em continuar
 do ponto em que estavam. Qual é a probabilidade de que Zé Roberto seja o
 vencedor?

 --
 Esta mensagem foi verificada pelo sistema de antivírus e
 acredita-se estar livre de perigo.
>>>
>>>
>>> --
>>> Esta mensagem foi verificada pelo sistema de antivírus e
>>> acredita-se estar livre de perigo.
>>
>>
>> --
>> Esta mensagem foi verificada pelo sistema de antivírus e
>> acredita-se estar livre de perigo.
>
>

-- 
Esta mensagem foi verificada pelo sistema de antiv�rus e
 acredita-se estar livre de perigo.



Re: [obm-l] Desafio de probabilidade

2020-07-25 Por tôpico Ralph Costa Teixeira
Argh: tem um errinho de digitação... Era p=7b *MENOS* 3...

Mas o resto continua valendo, achei p=25/43 e b=22/43... que condiz com
minha intuição de que, partindo de um número par (que não se repetiu),
tenho uma pequena vantagem (b=22/43 é ligeiramente maior que 1/2).

On Sat, Jul 25, 2020 at 3:37 PM Ralph Costa Teixeira 
wrote:

> Oi, Vanderlei.
>
> Para facilitar a notação, eu serei Zé Roberto. :D
>
> Intuitivamente: como você desconfiou, p não pode ser isso tudo. Para eu
> ganhar, tenho que rolar um 6, **ou** rolar outra coisa e "praticamente"
> começar o jogo de novo. Isto daria a estimativa:
> p = 1/6 + 5/6 . 1/2 = 7/12
> Mas esta estimativa está errada pois, o jogo "recomeçaria" a partir de 1,
> 2, 3, 4 ou 5, dando uma pequena vantagem para Umberto! Ou seja, sem fazer
> muita conta afirmo que:
> p = 1/6 + 5/6 . Prob(Eu ganhar a partir de um 1,2,3,4,5 simples) < 1/6 +
> 5/12 = 7/12.
>
> ---///---
> Se entendi o que você pensou: para Umberto ganhar, temos que NÃO ROLAR 6
> agora (prob=5/6), **e** depois "começar o jogo do zero" (prob=3/6). Então
> Umberto ganharia com probabilidade q=5/6.3/6=5/12 (acho que você devia usar
> 5/6; e não entendi o p extra). Era isso? Mas mesmo assim não funciona, pelo
> mesmo motivo da minha estimativa do p estar furada: não rolando 6, o jogo
> não "começaria do zero"; Umberto teria uma pequena vantagem (pois rolamos
> 1, 3 ou 5 com mais chance do que 2 ou 4). Em suma, o que conseguimos
> concluir daqui eh que q > 5/12, ou seja p<7/12, que nem acima!
>
> ---///---
> Vamos calcular p, definido como"
> p = probabilidade de eu vencer sendo que os dois últimos números foram
> iguais e pares
> E vamos inventar:
> b = probabilidade de eu vencer sendo que o último número foi par, mas o
> penúltimo foi diferente do último.
> Agora, na situação do problema (terminando em x-6-6, com x<>6), a partir
> daqui temos duas possibilidades:
> -- Se o próximo número for 6 (prob = 1/6), ganhei.
> -- Se o próximo número for 2 ou 4 (prob = 2/6), passamos para uma nova
> situação onde eu tenho probabilidade b de vencer.
> -- Se o próximo número for 1, 3 ou 5 (prob=3/6), passamos para uma nova
> situação onde eu tenho probabilidade 1-b de vencer.
>
> Assim, p = 1/6 + (2/6)b + (3/6)(1-b) = 2/3 - b/6.
>
> Por outro lado, a partir de uma situação do tipo "b" -- (digamos, para
> fixar ideias, sequência terminando em -2-4), temos as seguintes
> possibilidades:
> -- Se o próximo número for 4 (prob = 1/6), passo a ter probabilidade p de
> ganhar;
> -- Se for 2 ou 6 (prob = 2/6), passo a ter probabilidade b de ganhar.
> -- Se for 1, 3 ou 5 (prob = 3/6), passo a ter probabilidade 1-b de ganhar.
>
> Assim, b = (1/6)p + 2/6b + 3/6(1-b), ou seja, p=7b+3.
>
> Juntando as duas coisas, achei p=25/43 Hein, sério??
>
> ---///---
>  Vamos resolver de outro jeito mais "adulto", para ver o PODER DAS
> MATRIZES... :D :D:
>
> Depois de alguns lançamentos, o jogo tem 6 estados possíveis dependendo
> apenas dos 3 últimos lançamentos:
>
> E1: ...y-y-y com y par (eu ganhei! pare o jogo!)
> E2: ...x-y-y com x<>y e y par (estou quase ganhando!)
> E3: ...x-y com x<>y e y par (tenho pequena vantagem)
> E4: ...x-y com x<>y e y ímpar (tenho pequena desvantagem)
> E5: ...x-y-y com x<>y e y í mpar (estou quase perdendo!)
> E6: ...y-y-y com y ímpar (perdi! vire a mesa!)
>
> A matriz M de transição entre esses estados:
>
> 1 1/6000 0
> 00 1/600 0
> 0 2/6 2/6 3/6 3/6 0
> 0 3/6 3/6 2/6 2/6 0
> 000 1/60 0
> 0000 1/6 1
>
> Pois bem, M^k.v (onde v=e_2=[0; 1; 0; 0; 0; 0]) seria a distribuição de
> probabilidade dos estados, começando de e_2 (situação do problema), daqui a
> k jogadas. Ou seja, a gente quer determinar p onde lim(k->Inf) M^k.v = [p;
> 0; 0; 0; 0; 1-p].
>
> Diagonalizei M usando o computador (que não liga para a elegância das
> simetrias :( ), deu M = PDP^(-1) onde
>
> P=[1 1 1 1 1 0
> 0 (1/(√5-3))(√5+3) (3/2)√5-(7/2) -(1/2)i√3-((13)/2) (1/2)i√3-((13)/2) 0
> 0 -2((√5)/(√5-3)) (5/2)-(3/2)√5 (7/2)i√3+(5/2) (5/2)-(7/2)i√3 0
> 0 -2((√5)/(√5-3)) (5/2)-(3/2)√5 -(7/2)i√3-(5/2) (7/2)i√3-(5/2) 0
> 0 (1/(√5-3))(√5+3) (3/2)√5-(7/2) (1/2)i√3+((13)/2) ((13)/2)-(1/2)i√3 0
> -1 1 1 -1 -1 1]
>
> D = diag(1; 5/12- √5 /4;  5/12 +√5 /4; (-i√3-1)/12; (i√3-1)/12; 1]
>
> Q=P^(-1)= [1 ((25)/(43)) ((22)/(43)) ((21)/(43)) ((18)/(43)) 0
> 0 (1/(12))√5-(1/4) (7/(60))√5-(1/4) (7/(60))√5-(1/4) (1/(12))√5-(1/4) 0
> 0 -(1/(12))√5-(1/4) -(7/(60))√5-(1/4) -(7/(60))√5-(1/4) -(1/(12))√5-(1/4) 0
> 0 -(5/(516))i√3-(7/(172)) -((13)/(516))i√3-(1/(172))
> ((13)/(516))i√3+(1/(172)) (5/(516))i√3+(7/(172)) 0
> 0 (5/(516))i√3-(7/(172)) ((13)/(516))i√3-(1/(172))
> (1/(172))-((13)/(516))i√3 (7/(172))-(5/(516))i√3 0
> 1 1 1 1 1 1]
>
> Mas não olhe para tudo isso! Veja bem, M^k.v = P D^k P^(-1) e_2 = P D^k
> Q_2, onde Q_2 eh a segunda coluna de Q. Mas D^k vai para diag(1, 0, 0, 0,
> 0, 1) quando k->Inf, portanto ligamos apenas para Q(1,2)=25/43 e Q(1,6)=1.
> Queremos apenas a primeira coordenada de P . 

Re: [obm-l] Desafio de probabilidade

2020-07-25 Por tôpico Ralph Costa Teixeira
Oi, Claudio

Eu também pensei em trocar o dado por uma moeda, mas se entendi bem o
enunciado, não podemos! O problema eh que, se o dado der 2,4,6,2,4,6,1,1,1,
quem ganha eh Umberto; trocando pela moeda, vemos par,par,par e vamos dar o
trofeu para o Ze Roberto... Muda o jogo!

On Sat, Jul 25, 2020 at 3:24 PM Claudio Buffara 
wrote:

> Pra facilitar, podemos substituir o dado por uma moeda, com cara = par = 0
> e coroa = ímpar = 1, já que o que importa é apenas a paridade do número na
> face superior do dado lançado e, neste caso, P(par) = P(ímpar) = 1/2.
>
> Como 3 caras seguidas ou 3 coroas seguidas encerra o jogo, basta
> considerar os dois últimos lançamentos.
>
> Suponha que dois lançamentos seguidos tenham sido 1 e 0 (cara e coroa).
> Após sair o 0, digamos que a probabilidade de ZR vencer seja p.
>
> Se o terceiro lançamento for 0, a probabilidade de ZR vencer aumentará
> para q  (p e q são incógnitas a serem determinadas), e q é justamente a
> probabilidade desejada, já que é a probabilidade de ZR vencer dado que os
> dois últimos lançamentos foram 0 e 0.
> Se o quarto lançamento for 0, ZR vence. Mas se for 1, sua probabilidade de
> vencer cai para 1-p pois, neste caso, por simetria, Umberto passa a ter
> probabilidade p de vencer, em virtude dos dois últimos lançamentos terem
> sido 0 e 1.
> Ou seja, q = (1/2)*(1 + (1-p))  <==>  p + 2q = 2.
>
> Se o terceiro lançamento for 1, a probabilidade de ZR vencer cai para 1-p.
> Neste caso, podemos escrever p = (1/2)*(q + (1-p))  <==>  3p - q = 1.
>
> Resolvendo este sistema, achamos p = 4/7 e q = 5/7.
>
> Na verdade, isso tudo fica mais fácil de ver se você fizer uma árvore.
>
> []s,
> Claudio.
>
> On Sat, Jul 25, 2020 at 2:03 PM Professor Vanderlei Nemitz <
> vanderma...@gmail.com> wrote:
>
>> Então meu raciocínio foi muito errado, pois pensei assim:
>> Seja p a probabilidade de Zé Roberto vender. Podemos considerar que o
>> jogo "começa" com Zé Roberto precisando obter um 6 para vencer.
>> Assim, a probabilidade de Humberto vencer é:
>> q = (3/6).(1/6).p, ou seja, p = 12q
>> Assim, p = 12/13 e q = 1/13
>>
>> Prezado Cláudio, você pode explicar sua resolução?
>>
>> Muito obrigado!
>>
>>
>>
>>
>>
>>
>> Em sáb., 25 de jul. de 2020 às 13:43, Claudio Buffara <
>> claudio.buff...@gmail.com> escreveu:
>>
>>> Eu achei 5/7.
>>>
>>> On Sat, Jul 25, 2020 at 7:28 AM Professor Vanderlei Nemitz <
>>> vanderma...@gmail.com> wrote:
>>>
 Bom dia!
 O problema a seguir encontra-se em uma prova de desafios da PUC-RJ,
 muito boas!!!
 Acho que são organizadas pelo professor Nicolau Saldanha.
 Encontrei uma resposta bem alta, mais de 90%. Será que está correto?
 Muito obrigado!

 Zé Roberto e Umberto gostam de jogar par ou ímpar; Zé Roberto sempre
 pede par e Umberto sempre pede íımpar. Eles gostam de inventar novas
 maneiras de jogar. A última maneira que eles inventaram usa um dado comum,
 com seis faces numeradas de 1 a 6. Eles jogam o dado várias vezes até que
 um número saia três vezes seguidas; Zé Roberto ganha se este número for
 par, Umberto ganha se for ímpar. Sábado de manhã o dado teve os resultados:
 5, 3, 4, 2, 6, 1, 1, 3, 1, 4, 2, 3, 5, 6, 3, 4, 5, 4, 4, 4 e neste ponto Zé
 Roberto se declarou vitorioso. Sábado de tarde o dado teve os resultados:
 6, 1, 4, 2, 3, 5, 6, 6; neste momento o jogo foi interrompido pela queda de
 um meteorito. Quando a situação se acalmou, eles concordaram em continuar
 do ponto em que estavam. Qual é a probabilidade de que Zé Roberto seja o
 vencedor?

 --
 Esta mensagem foi verificada pelo sistema de antivírus e
 acredita-se estar livre de perigo.
>>>
>>>
>>> --
>>> Esta mensagem foi verificada pelo sistema de antivírus e
>>> acredita-se estar livre de perigo.
>>
>>
>> --
>> Esta mensagem foi verificada pelo sistema de antivírus e
>> acredita-se estar livre de perigo.
>
>
> --
> Esta mensagem foi verificada pelo sistema de antivírus e
> acredita-se estar livre de perigo.

-- 
Esta mensagem foi verificada pelo sistema de antiv�rus e
 acredita-se estar livre de perigo.



Re: [obm-l] Desafio de probabilidade

2020-07-25 Por tôpico Ralph Costa Teixeira
Oi, Vanderlei.

Para facilitar a notação, eu serei Zé Roberto. :D

Intuitivamente: como você desconfiou, p não pode ser isso tudo. Para eu
ganhar, tenho que rolar um 6, **ou** rolar outra coisa e "praticamente"
começar o jogo de novo. Isto daria a estimativa:
p = 1/6 + 5/6 . 1/2 = 7/12
Mas esta estimativa está errada pois, o jogo "recomeçaria" a partir de 1,
2, 3, 4 ou 5, dando uma pequena vantagem para Umberto! Ou seja, sem fazer
muita conta afirmo que:
p = 1/6 + 5/6 . Prob(Eu ganhar a partir de um 1,2,3,4,5 simples) < 1/6 +
5/12 = 7/12.

---///---
Se entendi o que você pensou: para Umberto ganhar, temos que NÃO ROLAR 6
agora (prob=5/6), **e** depois "começar o jogo do zero" (prob=3/6). Então
Umberto ganharia com probabilidade q=5/6.3/6=5/12 (acho que você devia usar
5/6; e não entendi o p extra). Era isso? Mas mesmo assim não funciona, pelo
mesmo motivo da minha estimativa do p estar furada: não rolando 6, o jogo
não "começaria do zero"; Umberto teria uma pequena vantagem (pois rolamos
1, 3 ou 5 com mais chance do que 2 ou 4). Em suma, o que conseguimos
concluir daqui eh que q > 5/12, ou seja p<7/12, que nem acima!

---///---
Vamos calcular p, definido como"
p = probabilidade de eu vencer sendo que os dois últimos números foram
iguais e pares
E vamos inventar:
b = probabilidade de eu vencer sendo que o último número foi par, mas o
penúltimo foi diferente do último.
Agora, na situação do problema (terminando em x-6-6, com x<>6), a partir
daqui temos duas possibilidades:
-- Se o próximo número for 6 (prob = 1/6), ganhei.
-- Se o próximo número for 2 ou 4 (prob = 2/6), passamos para uma nova
situação onde eu tenho probabilidade b de vencer.
-- Se o próximo número for 1, 3 ou 5 (prob=3/6), passamos para uma nova
situação onde eu tenho probabilidade 1-b de vencer.

Assim, p = 1/6 + (2/6)b + (3/6)(1-b) = 2/3 - b/6.

Por outro lado, a partir de uma situação do tipo "b" -- (digamos, para
fixar ideias, sequência terminando em -2-4), temos as seguintes
possibilidades:
-- Se o próximo número for 4 (prob = 1/6), passo a ter probabilidade p de
ganhar;
-- Se for 2 ou 6 (prob = 2/6), passo a ter probabilidade b de ganhar.
-- Se for 1, 3 ou 5 (prob = 3/6), passo a ter probabilidade 1-b de ganhar.

Assim, b = (1/6)p + 2/6b + 3/6(1-b), ou seja, p=7b+3.

Juntando as duas coisas, achei p=25/43 Hein, sério??

---///---
 Vamos resolver de outro jeito mais "adulto", para ver o PODER DAS
MATRIZES... :D :D:

Depois de alguns lançamentos, o jogo tem 6 estados possíveis dependendo
apenas dos 3 últimos lançamentos:

E1: ...y-y-y com y par (eu ganhei! pare o jogo!)
E2: ...x-y-y com x<>y e y par (estou quase ganhando!)
E3: ...x-y com x<>y e y par (tenho pequena vantagem)
E4: ...x-y com x<>y e y ímpar (tenho pequena desvantagem)
E5: ...x-y-y com x<>y e y í mpar (estou quase perdendo!)
E6: ...y-y-y com y ímpar (perdi! vire a mesa!)

A matriz M de transição entre esses estados:

1 1/6000 0
00 1/600 0
0 2/6 2/6 3/6 3/6 0
0 3/6 3/6 2/6 2/6 0
000 1/60 0
0000 1/6 1

Pois bem, M^k.v (onde v=e_2=[0; 1; 0; 0; 0; 0]) seria a distribuição de
probabilidade dos estados, começando de e_2 (situação do problema), daqui a
k jogadas. Ou seja, a gente quer determinar p onde lim(k->Inf) M^k.v = [p;
0; 0; 0; 0; 1-p].

Diagonalizei M usando o computador (que não liga para a elegância das
simetrias :( ), deu M = PDP^(-1) onde

P=[1 1 1 1 1 0
0 (1/(√5-3))(√5+3) (3/2)√5-(7/2) -(1/2)i√3-((13)/2) (1/2)i√3-((13)/2) 0
0 -2((√5)/(√5-3)) (5/2)-(3/2)√5 (7/2)i√3+(5/2) (5/2)-(7/2)i√3 0
0 -2((√5)/(√5-3)) (5/2)-(3/2)√5 -(7/2)i√3-(5/2) (7/2)i√3-(5/2) 0
0 (1/(√5-3))(√5+3) (3/2)√5-(7/2) (1/2)i√3+((13)/2) ((13)/2)-(1/2)i√3 0
-1 1 1 -1 -1 1]

D = diag(1; 5/12- √5 /4;  5/12 +√5 /4; (-i√3-1)/12; (i√3-1)/12; 1]

Q=P^(-1)= [1 ((25)/(43)) ((22)/(43)) ((21)/(43)) ((18)/(43)) 0
0 (1/(12))√5-(1/4) (7/(60))√5-(1/4) (7/(60))√5-(1/4) (1/(12))√5-(1/4) 0
0 -(1/(12))√5-(1/4) -(7/(60))√5-(1/4) -(7/(60))√5-(1/4) -(1/(12))√5-(1/4) 0
0 -(5/(516))i√3-(7/(172)) -((13)/(516))i√3-(1/(172))
((13)/(516))i√3+(1/(172)) (5/(516))i√3+(7/(172)) 0
0 (5/(516))i√3-(7/(172)) ((13)/(516))i√3-(1/(172))
(1/(172))-((13)/(516))i√3 (7/(172))-(5/(516))i√3 0
1 1 1 1 1 1]

Mas não olhe para tudo isso! Veja bem, M^k.v = P D^k P^(-1) e_2 = P D^k
Q_2, onde Q_2 eh a segunda coluna de Q. Mas D^k vai para diag(1, 0, 0, 0,
0, 1) quando k->Inf, portanto ligamos apenas para Q(1,2)=25/43 e Q(1,6)=1.
Queremos apenas a primeira coordenada de P . [25/43; 0; 0; 0; 0; 1], ou
seja, p = 25/43 P(1,1) + 1 P(6,1) = 25/43.

Ou seja: sério!! :D

Abraco, Ralph.


On Sat, Jul 25, 2020 at 2:03 PM Professor Vanderlei Nemitz <
vanderma...@gmail.com> wrote:

> Então meu raciocínio foi muito errado, pois pensei assim:
> Seja p a probabilidade de Zé Roberto vender. Podemos considerar que o jogo
> "começa" com Zé Roberto precisando obter um 6 para vencer.
> Assim, a probabilidade de Humberto vencer é:
> q = (3/6).(1/6).p, ou seja, p = 12q
> Assim, p = 12/13 e q = 1/13
>
> Prezado 

Re: [obm-l] Desafio de probabilidade

2020-07-25 Por tôpico Claudio Buffara
Pra facilitar, podemos substituir o dado por uma moeda, com cara = par = 0
e coroa = ímpar = 1, já que o que importa é apenas a paridade do número na
face superior do dado lançado e, neste caso, P(par) = P(ímpar) = 1/2.

Como 3 caras seguidas ou 3 coroas seguidas encerra o jogo, basta considerar
os dois últimos lançamentos.

Suponha que dois lançamentos seguidos tenham sido 1 e 0 (cara e coroa).
Após sair o 0, digamos que a probabilidade de ZR vencer seja p.

Se o terceiro lançamento for 0, a probabilidade de ZR vencer aumentará para
q  (p e q são incógnitas a serem determinadas), e q é justamente a
probabilidade desejada, já que é a probabilidade de ZR vencer dado que os
dois últimos lançamentos foram 0 e 0.
Se o quarto lançamento for 0, ZR vence. Mas se for 1, sua probabilidade de
vencer cai para 1-p pois, neste caso, por simetria, Umberto passa a ter
probabilidade p de vencer, em virtude dos dois últimos lançamentos terem
sido 0 e 1.
Ou seja, q = (1/2)*(1 + (1-p))  <==>  p + 2q = 2.

Se o terceiro lançamento for 1, a probabilidade de ZR vencer cai para 1-p.
Neste caso, podemos escrever p = (1/2)*(q + (1-p))  <==>  3p - q = 1.

Resolvendo este sistema, achamos p = 4/7 e q = 5/7.

Na verdade, isso tudo fica mais fácil de ver se você fizer uma árvore.

[]s,
Claudio.

On Sat, Jul 25, 2020 at 2:03 PM Professor Vanderlei Nemitz <
vanderma...@gmail.com> wrote:

> Então meu raciocínio foi muito errado, pois pensei assim:
> Seja p a probabilidade de Zé Roberto vender. Podemos considerar que o jogo
> "começa" com Zé Roberto precisando obter um 6 para vencer.
> Assim, a probabilidade de Humberto vencer é:
> q = (3/6).(1/6).p, ou seja, p = 12q
> Assim, p = 12/13 e q = 1/13
>
> Prezado Cláudio, você pode explicar sua resolução?
>
> Muito obrigado!
>
>
>
>
>
>
> Em sáb., 25 de jul. de 2020 às 13:43, Claudio Buffara <
> claudio.buff...@gmail.com> escreveu:
>
>> Eu achei 5/7.
>>
>> On Sat, Jul 25, 2020 at 7:28 AM Professor Vanderlei Nemitz <
>> vanderma...@gmail.com> wrote:
>>
>>> Bom dia!
>>> O problema a seguir encontra-se em uma prova de desafios da PUC-RJ,
>>> muito boas!!!
>>> Acho que são organizadas pelo professor Nicolau Saldanha.
>>> Encontrei uma resposta bem alta, mais de 90%. Será que está correto?
>>> Muito obrigado!
>>>
>>> Zé Roberto e Umberto gostam de jogar par ou ímpar; Zé Roberto sempre
>>> pede par e Umberto sempre pede íımpar. Eles gostam de inventar novas
>>> maneiras de jogar. A última maneira que eles inventaram usa um dado comum,
>>> com seis faces numeradas de 1 a 6. Eles jogam o dado várias vezes até que
>>> um número saia três vezes seguidas; Zé Roberto ganha se este número for
>>> par, Umberto ganha se for ímpar. Sábado de manhã o dado teve os resultados:
>>> 5, 3, 4, 2, 6, 1, 1, 3, 1, 4, 2, 3, 5, 6, 3, 4, 5, 4, 4, 4 e neste ponto Zé
>>> Roberto se declarou vitorioso. Sábado de tarde o dado teve os resultados:
>>> 6, 1, 4, 2, 3, 5, 6, 6; neste momento o jogo foi interrompido pela queda de
>>> um meteorito. Quando a situação se acalmou, eles concordaram em continuar
>>> do ponto em que estavam. Qual é a probabilidade de que Zé Roberto seja o
>>> vencedor?
>>>
>>> --
>>> Esta mensagem foi verificada pelo sistema de antivírus e
>>> acredita-se estar livre de perigo.
>>
>>
>> --
>> Esta mensagem foi verificada pelo sistema de antivírus e
>> acredita-se estar livre de perigo.
>
>
> --
> Esta mensagem foi verificada pelo sistema de antivírus e
> acredita-se estar livre de perigo.

-- 
Esta mensagem foi verificada pelo sistema de antiv�rus e
 acredita-se estar livre de perigo.



Re: [obm-l] Desafio de probabilidade

2020-07-25 Por tôpico Professor Vanderlei Nemitz
Então meu raciocínio foi muito errado, pois pensei assim:
Seja p a probabilidade de Zé Roberto vender. Podemos considerar que o jogo
"começa" com Zé Roberto precisando obter um 6 para vencer.
Assim, a probabilidade de Humberto vencer é:
q = (3/6).(1/6).p, ou seja, p = 12q
Assim, p = 12/13 e q = 1/13

Prezado Cláudio, você pode explicar sua resolução?

Muito obrigado!






Em sáb., 25 de jul. de 2020 às 13:43, Claudio Buffara <
claudio.buff...@gmail.com> escreveu:

> Eu achei 5/7.
>
> On Sat, Jul 25, 2020 at 7:28 AM Professor Vanderlei Nemitz <
> vanderma...@gmail.com> wrote:
>
>> Bom dia!
>> O problema a seguir encontra-se em uma prova de desafios da PUC-RJ, muito
>> boas!!!
>> Acho que são organizadas pelo professor Nicolau Saldanha.
>> Encontrei uma resposta bem alta, mais de 90%. Será que está correto?
>> Muito obrigado!
>>
>> Zé Roberto e Umberto gostam de jogar par ou ímpar; Zé Roberto sempre pede
>> par e Umberto sempre pede íımpar. Eles gostam de inventar novas maneiras de
>> jogar. A última maneira que eles inventaram usa um dado comum, com seis
>> faces numeradas de 1 a 6. Eles jogam o dado várias vezes até que um número
>> saia três vezes seguidas; Zé Roberto ganha se este número for par, Umberto
>> ganha se for ímpar. Sábado de manhã o dado teve os resultados: 5, 3, 4, 2,
>> 6, 1, 1, 3, 1, 4, 2, 3, 5, 6, 3, 4, 5, 4, 4, 4 e neste ponto Zé Roberto se
>> declarou vitorioso. Sábado de tarde o dado teve os resultados: 6, 1, 4, 2,
>> 3, 5, 6, 6; neste momento o jogo foi interrompido pela queda de um
>> meteorito. Quando a situação se acalmou, eles concordaram em continuar do
>> ponto em que estavam. Qual é a probabilidade de que Zé Roberto seja o
>> vencedor?
>>
>> --
>> Esta mensagem foi verificada pelo sistema de antivírus e
>> acredita-se estar livre de perigo.
>
>
> --
> Esta mensagem foi verificada pelo sistema de antivírus e
> acredita-se estar livre de perigo.

-- 
Esta mensagem foi verificada pelo sistema de antiv�rus e
 acredita-se estar livre de perigo.



Re: [obm-l] Desafio de probabilidade

2020-07-25 Por tôpico Claudio Buffara
Eu achei 5/7.

On Sat, Jul 25, 2020 at 7:28 AM Professor Vanderlei Nemitz <
vanderma...@gmail.com> wrote:

> Bom dia!
> O problema a seguir encontra-se em uma prova de desafios da PUC-RJ, muito
> boas!!!
> Acho que são organizadas pelo professor Nicolau Saldanha.
> Encontrei uma resposta bem alta, mais de 90%. Será que está correto?
> Muito obrigado!
>
> Zé Roberto e Umberto gostam de jogar par ou ímpar; Zé Roberto sempre pede
> par e Umberto sempre pede íımpar. Eles gostam de inventar novas maneiras de
> jogar. A última maneira que eles inventaram usa um dado comum, com seis
> faces numeradas de 1 a 6. Eles jogam o dado várias vezes até que um número
> saia três vezes seguidas; Zé Roberto ganha se este número for par, Umberto
> ganha se for ímpar. Sábado de manhã o dado teve os resultados: 5, 3, 4, 2,
> 6, 1, 1, 3, 1, 4, 2, 3, 5, 6, 3, 4, 5, 4, 4, 4 e neste ponto Zé Roberto se
> declarou vitorioso. Sábado de tarde o dado teve os resultados: 6, 1, 4, 2,
> 3, 5, 6, 6; neste momento o jogo foi interrompido pela queda de um
> meteorito. Quando a situação se acalmou, eles concordaram em continuar do
> ponto em que estavam. Qual é a probabilidade de que Zé Roberto seja o
> vencedor?
>
> --
> Esta mensagem foi verificada pelo sistema de antivírus e
> acredita-se estar livre de perigo.

-- 
Esta mensagem foi verificada pelo sistema de antiv�rus e
 acredita-se estar livre de perigo.



[obm-l] Re: [obm-l] Re: [obm-l] Re: [obm-l] Ajuda em teoria dos números

2020-07-24 Por tôpico Prof. Douglas Oliveira
Obrigado Claudio e Esdras, fatoração show


Em sex., 24 de jul. de 2020 às 11:12, Esdras Muniz <
esdrasmunizm...@gmail.com> escreveu:

> Se for solução inteira positiva, acho que só tem 3 e 4. Vc supõe spdg x
> maior ou igual a y, vê que y=1 não tem solução e x=y tb não. Daí, x>y>1.
> Fatorando a expressão, fica: (xy-8-(x-y))(xy-8+(x-y))=15. Como
> (xy-8-(x-y))>(xy-8+(x-y))>-2. Temos que ou (xy-8-(x-y))=1 e (xy-8+(x-y))=15,
> o que não tem soluções inteiras positivas, ou (xy-8-(x-y))=3 e (xy-8+(x-y))=5,
> cujas únicas soluções inteiras são x=4 e y=3.
>
> Em sex, 24 de jul de 2020 10:36, Claudio Buffara <
> claudio.buff...@gmail.com> escreveu:
>
>> Pelo que entendi, a solução é a porção dessa curva algébrica situada no
>> 1o quadrante.
>> Dá pra fazer isso no Wolfram Alpha, com o comando plot (x*y-7)^2 - x^2 -
>> y^2 = 0.
>>
>> []s,
>> Claudio.
>>
>> On Fri, Jul 24, 2020 at 9:58 AM Prof. Douglas Oliveira <
>> profdouglaso.del...@gmail.com> wrote:
>>
>>> Preciso de ajuda para encontrar todas as soluções não negativas da
>>> equação
>>> (xy-7)^2=x^2+y^2.
>>>
>>> Desde já agradeço a ajuda
>>> Douglas Oliveira
>>>
>>> --
>>> Esta mensagem foi verificada pelo sistema de antivírus e
>>> acredita-se estar livre de perigo.
>>
>>
>> --
>> Esta mensagem foi verificada pelo sistema de antivírus e
>> acredita-se estar livre de perigo.
>
>
> --
> Esta mensagem foi verificada pelo sistema de antivírus e
> acredita-se estar livre de perigo.

-- 
Esta mensagem foi verificada pelo sistema de antiv�rus e
 acredita-se estar livre de perigo.



[obm-l] Re: [obm-l] Re: [obm-l] Ajuda em teoria dos números

2020-07-24 Por tôpico Esdras Muniz
Se for solução inteira positiva, acho que só tem 3 e 4. Vc supõe spdg x
maior ou igual a y, vê que y=1 não tem solução e x=y tb não. Daí, x>y>1.
Fatorando a expressão, fica: (xy-8-(x-y))(xy-8+(x-y))=15. Como
(xy-8-(x-y))>(xy-8+(x-y))>-2.
Temos que ou (xy-8-(x-y))=1 e (xy-8+(x-y))=15, o que não tem soluções
inteiras positivas, ou (xy-8-(x-y))=3 e (xy-8+(x-y))=5, cujas únicas
soluções inteiras são x=4 e y=3.

Em sex, 24 de jul de 2020 10:36, Claudio Buffara 
escreveu:

> Pelo que entendi, a solução é a porção dessa curva algébrica situada no 1o
> quadrante.
> Dá pra fazer isso no Wolfram Alpha, com o comando plot (x*y-7)^2 - x^2 -
> y^2 = 0.
>
> []s,
> Claudio.
>
> On Fri, Jul 24, 2020 at 9:58 AM Prof. Douglas Oliveira <
> profdouglaso.del...@gmail.com> wrote:
>
>> Preciso de ajuda para encontrar todas as soluções não negativas da equação
>> (xy-7)^2=x^2+y^2.
>>
>> Desde já agradeço a ajuda
>> Douglas Oliveira
>>
>> --
>> Esta mensagem foi verificada pelo sistema de antivírus e
>> acredita-se estar livre de perigo.
>
>
> --
> Esta mensagem foi verificada pelo sistema de antivírus e
> acredita-se estar livre de perigo.

-- 
Esta mensagem foi verificada pelo sistema de antiv�rus e
 acredita-se estar livre de perigo.



[obm-l] Re: [obm-l] Ajuda em teoria dos números

2020-07-24 Por tôpico Claudio Buffara
Pelo que entendi, a solução é a porção dessa curva algébrica situada no 1o
quadrante.
Dá pra fazer isso no Wolfram Alpha, com o comando plot (x*y-7)^2 - x^2 -
y^2 = 0.

[]s,
Claudio.

On Fri, Jul 24, 2020 at 9:58 AM Prof. Douglas Oliveira <
profdouglaso.del...@gmail.com> wrote:

> Preciso de ajuda para encontrar todas as soluções não negativas da equação
> (xy-7)^2=x^2+y^2.
>
> Desde já agradeço a ajuda
> Douglas Oliveira
>
> --
> Esta mensagem foi verificada pelo sistema de antivírus e
> acredita-se estar livre de perigo.

-- 
Esta mensagem foi verificada pelo sistema de antiv�rus e
 acredita-se estar livre de perigo.



Re: [obm-l] Probabilidade

2020-07-21 Por tôpico Daniel Jelin
Que interessante! Pra mim deu isso tb, por outro caminho. Podemos ter: 0,
1, 2, 3, 4 ou 5 caras no máximo. 1 cara: podemos escolhemos 1 posição
qualquer dentre as 10; 2 caras: podemos escolher 2 posições de um total de
9, porque 1 posição entre caras deve ser garantido pra coroa; 3 caras:
escolhemos 3 posições de um total de 8, guardando 2 posições entre as
caras. 4 caras: escolhemos 4 posições de 7; 5 caras: escolhemos 5 de 6; e
pra 0 cara, claro, temos uma só opção. A chance de sair cara e coroa é a
mesma, 1/2, então temos:

(1/2)^10*(C10,1 + C9,2 + C8,3 + C7,4 + C6,5 + 1)=144/1024

On Wed, Jul 22, 2020 at 12:46 AM Ralph Costa Teixeira 
wrote:

> Vou chamar coroa de C e cara de K. Vamos criar duas funcoes:
>
> f(n)=numero de sequências de n lançamentos sem CC, terminando com K.
> g(n)=numero de sequências de n lançamentos sem CC, terminando com C.
>
> Por exemplo:
> f(1)=1 (K); g(1)=1 (C); f(2)=2 (CK, KK); g(2)=1 (KC)...
>
> Pois bem, note que f(n+1)=f(n)+g(n) -- para a sequência de n+1 elementos
> terminar com K, basta que não haja CC nos n primeiros;
> Por outro lado, g(n+1)=f(n) -- para a sequência de n+1 elementos terminar
> com C, a sequência dos n primeiros (nao pode ter CC e tem que terminar com
> K).
>
> Juntando as coisas, temos f(n+1)=f(n)+f(n-1) -- Fibonacci! O que queremos
> deve ser (f(10)+g(10)) / 2^10. Mas g(10)=f(9), então queremos f(11)/1024.
>
> Bom, melhor fazer logo no braco:
> {f(n)} = 1,2,3,5,8,13,21,34,55,89,144...
> (Fibonacci, com um ligeiro "shift" pois nao começa com 1,1,...)
>
> Portanto, acho que a resposta deve ser 144/1024. Acertei?
>
> Abraço, Ralph.
>
>
>
> On Tue, Jul 21, 2020 at 10:33 PM marcone augusto araújo borges <
> marconeborge...@hotmail.com> wrote:
>
>> Uma moeda honesta é lançada 10 vezes. Qual a probabilidade de não sair
>> duas caras consecutivas?
>> Eu achei que fosse (3/4)^9, mas fui informado que a resposta não é essa.
>>
>> --
>> Esta mensagem foi verificada pelo sistema de antivírus e
>> acredita-se estar livre de perigo.
>>
>
> --
> Esta mensagem foi verificada pelo sistema de antivírus e
> acredita-se estar livre de perigo.

-- 
Esta mensagem foi verificada pelo sistema de antiv�rus e
 acredita-se estar livre de perigo.



Re: [obm-l] Probabilidade

2020-07-21 Por tôpico Ralph Costa Teixeira
Vou chamar coroa de C e cara de K. Vamos criar duas funcoes:

f(n)=numero de sequências de n lançamentos sem CC, terminando com K.
g(n)=numero de sequências de n lançamentos sem CC, terminando com C.

Por exemplo:
f(1)=1 (K); g(1)=1 (C); f(2)=2 (CK, KK); g(2)=1 (KC)...

Pois bem, note que f(n+1)=f(n)+g(n) -- para a sequência de n+1 elementos
terminar com K, basta que não haja CC nos n primeiros;
Por outro lado, g(n+1)=f(n) -- para a sequência de n+1 elementos terminar
com C, a sequência dos n primeiros (nao pode ter CC e tem que terminar com
K).

Juntando as coisas, temos f(n+1)=f(n)+f(n-1) -- Fibonacci! O que queremos
deve ser (f(10)+g(10)) / 2^10. Mas g(10)=f(9), então queremos f(11)/1024.

Bom, melhor fazer logo no braco:
{f(n)} = 1,2,3,5,8,13,21,34,55,89,144...
(Fibonacci, com um ligeiro "shift" pois nao começa com 1,1,...)

Portanto, acho que a resposta deve ser 144/1024. Acertei?

Abraço, Ralph.



On Tue, Jul 21, 2020 at 10:33 PM marcone augusto araújo borges <
marconeborge...@hotmail.com> wrote:

> Uma moeda honesta é lançada 10 vezes. Qual a probabilidade de não sair
> duas caras consecutivas?
> Eu achei que fosse (3/4)^9, mas fui informado que a resposta não é essa.
>
> --
> Esta mensagem foi verificada pelo sistema de antivírus e
> acredita-se estar livre de perigo.
>

-- 
Esta mensagem foi verificada pelo sistema de antiv�rus e
 acredita-se estar livre de perigo.



[obm-l] Re: [obm-l] Método para participar da IOI(International Olympiad in Informatics)

2020-07-18 Por tôpico HugLeo
Olá,

Você pode usar a planilha do Mostafa:

https://docs.google.com/spreadsheets/d/1iJZWP2nS_OB3kCTjq8L6TrJJ4o-5lhxDOyTaocSYc-k/edit#gid=84654839


On Tue, Jul 7, 2020 at 11:31 AM Gustavo Bruno  wrote:

> Caro Senhor(a),
>
> Eu sei que você deve estar muito ocupado e que recebe muitos emails,
> portanto isso deve levar apenas sessenta segundos de leitura.
>
> Eu obtive 2 medalhas de bronze na OBMEP nos últimos 2 anos, ambas de nível
> 2. Trabalhei como programador freelancer para uma empresa canadense e, como
> consequência, possuo boas habilidades de programação e Inglês.
>
> Gostaria de saber qual seria o melhor método para estudar e conseguir uma
> medalha de ouro na OBM e, posteriormente, entrar na seleção para a IOI.
> Acredito que estudar atráves dos bancos de questões e, periodicamente,
> leitura de material complementar seja um bom método, mas gostaria de saber
> a sua opinião.
>
> Eu entendo se você estiver muito ocupado para responder, mas até mesmo uma
> resposta de uma ou duas linhas me faria muito feliz,
>
> Atenciosamente,
>
> Gustavo
>
> --
> Esta mensagem foi verificada pelo sistema de antivírus e
> acredita-se estar livre de perigo.

-- 
Esta mensagem foi verificada pelo sistema de antiv�rus e
 acredita-se estar livre de perigo.



[obm-l] Re: [obm-l] Álgebra

2020-07-15 Por tôpico Pacini Bores
 

A expressão pedida ao quadrado é igual a 4, sem usar complexos. 

Pacini 

Em 14/07/2020 21:50, marcone augusto araújo borges escreveu: 

> Se x^2 +xy + y^2 = 0, com x,y <>0 
> Determinar (x/(x+y))^2019 + (y/(x+y))^2019, sem usar números complexos. 
> -- 
> Esta mensagem foi verificada pelo sistema de antivírus e 
> acredita-se estar livre de perigo.

 
-- 
Esta mensagem foi verificada pelo sistema de antiv�rus e
 acredita-se estar livre de perigo.



[obm-l] Re: [obm-l] Congruência

2020-07-08 Por tôpico Anderson Torres
Em sáb., 4 de jul. de 2020 às 20:29, marcone augusto araújo borges
 escreveu:
>
> Determinar os inteiros positivos x tais que (x^5+5x2+x+1) é múltiplo de 121

Tente ver primeiro por 11. Isso já dá uma reduzida.

> --
> Esta mensagem foi verificada pelo sistema de antivírus e
> acredita-se estar livre de perigo.

-- 
Esta mensagem foi verificada pelo sistema de antiv�rus e
 acredita-se estar livre de perigo.


=
Instru��es para entrar na lista, sair da lista e usar a lista em
http://www.mat.puc-rio.br/~obmlistas/obm-l.html
=


[obm-l] Re: [obm-l] transcendência

2020-07-08 Por tôpico Carlos Gustavo Tamm de Araujo Moreira
Caro Israel,
Sim. Suponha que x e y são algebricamente dependentes sobre um corpo de
base K. Se y é algébrico, K(y)|K é uma extensão algébrica. Como x é raiz de
uma equação polinomial com coeficientes em K(y) (pois  x e y são
algebricamente dependentes), a extensão K(x,y)=K(y)(x)|K(y) é algébrica.
Portanto, como K(x,y)|K(y) e K(y)|K são extensões algébricas, K(x,y)|K
também é algébrica, e, em particular, x também é algébrico. Portanto, se x
é transcendente então y também é transcendente.
Abraços,
 Gugu


On Wed, Jul 8, 2020 at 2:04 PM Israel Meireles Chrisostomo <
israelmchrisost...@gmail.com> wrote:

> se dois números são algebricamente dependentes e se um deles é
> transcendente então isso implica o outro seja transcendente?isso me
> parece meio óbvio mas nao sei como provar
>
> --
> Israel Meireles Chrisostomo
>
> --
> Esta mensagem foi verificada pelo sistema de antivírus e
> acredita-se estar livre de perigo.

-- 
Esta mensagem foi verificada pelo sistema de antiv�rus e
 acredita-se estar livre de perigo.



Re: [obm-l] Método para participar da IOI(International Olympiad in Informatics)

2020-07-07 Por tôpico regis barros
 
Olá Gustavo
Eu recomendaria que você estudasse os banco de questões e procurasse livros de 
problemas de matemática para olimpíadas. E ver as revistas de matemática pois 
algumas tem problemas para os pessoas resolver.
RegisEm terça-feira, 7 de julho de 2020 13:14:30 BRT, Gustavo Bruno 
 escreveu:  
 
 Caro Senhor(a),
Eu sei que você deve estar muito ocupado e que recebe muitos emails, portanto 
isso deve levar apenas sessenta segundos de leitura.
Eu obtive 2 medalhas de bronze na OBMEP nos últimos 2 anos, ambas de nível 2. 
Trabalhei como programador freelancer para uma empresa canadense e, como 
consequência, possuo boas habilidades de programação e Inglês.
Gostaria de saber qual seria o melhor método para estudar e conseguir uma 
medalha de ouro na OBM e, posteriormente, entrar na seleção para a IOI. 
Acredito que estudar atráves dos bancos de questões e, periodicamente, leitura 
de material complementar seja um bom método, mas gostaria de saber a sua 
opinião.
Eu entendo se você estiver muito ocupado para responder, mas até mesmo uma 
resposta de uma ou duas linhas me faria muito feliz,
Atenciosamente,
Gustavo
--
Esta mensagem foi verificada pelo sistema de antiv�rus e 
 acredita-se estar livre de perigo.  
-- 
Esta mensagem foi verificada pelo sistema de antiv�rus e
 acredita-se estar livre de perigo.



[obm-l] Re: [obm-l] Método para participar da IOI(International Olympiad in Informatics)

2020-07-07 Por tôpico Christian Bernard
Não existe um método certo que vai levar do ponto x ao ponto y nesse
assunto e, apesar das olimpiadas terem uma interseção em alguns assuntos, é
uma boa ideia focar em apenas uma inicialmente. No mais, alguns links que
podem lhe ajudar bastante:

OBI:
www.usaco.org
codeforces.com
github.com/bqi343/USACO

OBM:
artofproblemsolving.com


On Tue, Jul 7, 2020, 11:31 Gustavo Bruno  wrote:

> Caro Senhor(a),
>
> Eu sei que você deve estar muito ocupado e que recebe muitos emails,
> portanto isso deve levar apenas sessenta segundos de leitura.
>
> Eu obtive 2 medalhas de bronze na OBMEP nos últimos 2 anos, ambas de nível
> 2. Trabalhei como programador freelancer para uma empresa canadense e, como
> consequência, possuo boas habilidades de programação e Inglês.
>
> Gostaria de saber qual seria o melhor método para estudar e conseguir uma
> medalha de ouro na OBM e, posteriormente, entrar na seleção para a IOI.
> Acredito que estudar atráves dos bancos de questões e, periodicamente,
> leitura de material complementar seja um bom método, mas gostaria de saber
> a sua opinião.
>
> Eu entendo se você estiver muito ocupado para responder, mas até mesmo uma
> resposta de uma ou duas linhas me faria muito feliz,
>
> Atenciosamente,
>
> Gustavo
>
> --
> Esta mensagem foi verificada pelo sistema de antivírus e
> acredita-se estar livre de perigo.

-- 
Esta mensagem foi verificada pelo sistema de antiv�rus e
 acredita-se estar livre de perigo.



Re: [obm-l] Problema simples gera um complicado?

2020-07-02 Por tôpico Professor Vanderlei Nemitz
Muito obrigado, Matheus!
Vou estudar sobre esse ponto especial!

Em qui., 2 de jul. de 2020 às 19:58, Matheus Bezerra <
matheusbezerr...@gmail.com> escreveu:

> Olá Vanderlei, boa noite. Esse é um fato conhecido, essas retas concorrem
> em um ponto chamado de Ponto de Fermat. Pesquisa por isso que você deve
> encontrar alguma prova. ;)
>
> *Matheus BL*
>
>
> Em qui., 2 de jul. de 2020 às 18:55, Professor Vanderlei Nemitz <
> vanderma...@gmail.com> escreveu:
>
>> Oi, pessoal, tudo bem?
>>
>> Resolvi um problema simples, que me fez pensar em outro, talvez
>> complicado. Bom, pelos menos são encontrei uma solução. Será que é verdade?
>> Se alguém puder ajudar a provar, caso seja, ficarei muito agradecido. Sem
>> querer "exigir" nada, afinal de contas eu não consegui, mas se puder ser
>> sem usar geometria analítica... :)
>>
>> Sobre os lados de um triângulo ABC são construídos externamente os
>> triângulos equiláteros BCD, CAE e ABF. Prove que os segmentos AD, BE e CF
>> são congruentes. (Esse é o problema simples!)
>>
>> Gostaria de provar que:
>>
>> *Sobre os lados de um triângulo ABC são construídos externamente os
>> triângulos equiláteros BCD, CAE e ABF. Prove que os segmentos AD, BE e CF
>> (ou suas retas suporte) concorrem em um mesmo ponto.*
>>
>> Muito obrigado!
>>
>> --
>> Esta mensagem foi verificada pelo sistema de antivírus e
>> acredita-se estar livre de perigo.
>
>
> --
> Esta mensagem foi verificada pelo sistema de antivírus e
> acredita-se estar livre de perigo.

-- 
Esta mensagem foi verificada pelo sistema de antiv�rus e
 acredita-se estar livre de perigo.



Re: [obm-l] Problema simples gera um complicado?

2020-07-02 Por tôpico Matheus Bezerra
Olá Vanderlei, boa noite. Esse é um fato conhecido, essas retas concorrem
em um ponto chamado de Ponto de Fermat. Pesquisa por isso que você deve
encontrar alguma prova. ;)

*Matheus BL*


Em qui., 2 de jul. de 2020 às 18:55, Professor Vanderlei Nemitz <
vanderma...@gmail.com> escreveu:

> Oi, pessoal, tudo bem?
>
> Resolvi um problema simples, que me fez pensar em outro, talvez
> complicado. Bom, pelos menos são encontrei uma solução. Será que é verdade?
> Se alguém puder ajudar a provar, caso seja, ficarei muito agradecido. Sem
> querer "exigir" nada, afinal de contas eu não consegui, mas se puder ser
> sem usar geometria analítica... :)
>
> Sobre os lados de um triângulo ABC são construídos externamente os
> triângulos equiláteros BCD, CAE e ABF. Prove que os segmentos AD, BE e CF
> são congruentes. (Esse é o problema simples!)
>
> Gostaria de provar que:
>
> *Sobre os lados de um triângulo ABC são construídos externamente os
> triângulos equiláteros BCD, CAE e ABF. Prove que os segmentos AD, BE e CF
> (ou suas retas suporte) concorrem em um mesmo ponto.*
>
> Muito obrigado!
>
> --
> Esta mensagem foi verificada pelo sistema de antivírus e
> acredita-se estar livre de perigo.

-- 
Esta mensagem foi verificada pelo sistema de antiv�rus e
 acredita-se estar livre de perigo.



[obm-l] Re: [obm-l] Polinômios - Longlists -83

2020-06-29 Por tôpico Anderson Torres
Em dom., 21 de jun. de 2020 às 20:09, Jeferson Almir
 escreveu:
>
> Amigos peço ajuda no seguinte problema( item b principalmente).
>
> Considere a expansão
> ( 1 + x + x^2 + x^3 + x^4 )^496 = a_0 + a_1x +  + a_1984x^1984
>
> a) Determine o mdc( a_3, a_8, a_13, ... , a_1983 )
>
> b) Prove que 10^340 < a_922 < 10^347
>
> No item a) eu usei raizes da unidade, mas se alguém tem alguma ideia via 
> Funções Geratrizes eu agradeceria muito.

Você vai fazer essencialmente a mesma coisa de qualquer forma: obter
uma relação entre os termos da forma a(5k+3). Não acho que dê para
fazer isso sem usar raízes da unidade. Até porque elas são
manipulações formais.

> Já o item b) tentei usar médias ou algo de termo geral de expansão 
> multinomial e não consegui nada.
>

Poxa, aqui eu acho que vou ter que dar a dica mesmo! Ache a soma dos
termos do MDC que cê quer calcular.

> --
> Esta mensagem foi verificada pelo sistema de antivírus e
> acredita-se estar livre de perigo.

-- 
Esta mensagem foi verificada pelo sistema de antiv�rus e
 acredita-se estar livre de perigo.


=
Instru��es para entrar na lista, sair da lista e usar a lista em
http://www.mat.puc-rio.br/~obmlistas/obm-l.html
=


[obm-l] Re: [obm-l] Re: [obm-l] Diferencial de uma função de uma variável

2020-06-21 Por tôpico Pacini Bores
 

Em 21/06/2020 17:36, Pacini Bores escreveu: 

> Obrigado a todos pelas respostas didáticas. 
> 
> Pacini 
> 
> Em 21/06/2020 13:43, Ralph Costa Teixeira escreveu: 
> Voce diz, aquele "dy" sozinho? 
> 
> Eu gosto de pensar assim: considere uma função f(x) diferenciável num ponto 
> a. A *linearizacão* de f(x) em x=a é dada por: 
> L(x) = f(a) + f'(a) (x-a) 
> e a ideia é que L(x) aproxima "bastante bem" f(x) ali perto de x=a (o gráfico 
> de L(x) é a reta tangente). 
> 
> Para dar contexto, escreva y=f(x) e, a partir de "a", vamos aplicar uma 
> variação Delta_x (um número real, possivelmente grande), indo para 
> x=a+Delta_x. Esta variação no domínio provoca variação na imagem de f, a 
> saber: 
> Delta_y = Delta_f = f(a+Delta_x)-f(a) 
> Analogamente, olhe para L(x) e, a partir de "a", aplique uma variação de dx 
> (um número real, possivelmente grande), indo para x=a+dx. A DIFERENCIAL DE F 
> NO PONTO A (ASSOCIADA A DX) é 
> dy = Delta_L=L(a+dx)-L(a) 
> ou seja, dy é simplesmente A VARIAÇÃO EM Y MEDIDA PELA LINEARIZAÇÃO, ou seja, 
> USANDO A RETA TANGENTE (ao invés de usar a f(x) original). 
> 
> Note que podemos escrever dy explicitamente em termos de f, pois temos aquela 
> fórmula ali em cima para L: 
> dy = L(a+dx)-L(a) = (f(a)+f'(a).dx)-(f(a)+f'(a).0) = f'(a).dx 
> Em suma: 
> dy = f'(a).dx 
> Esta última expressão é exatamente a equação da reta tangente, escrita dum 
> jeito mais curto (pois fizemos L(x)-L(a)=dy e x-a=dx)! 
> 
> Comparando: 
> -- Não há diferença prática entre "dx" e "Delta_x"; apenas por convenção, 
> quando eu estiver trabalhando com a linearização, vou escrever dx ao invés de 
> Delta_x. Voce não perde praticamente nada se pensar que dx=Delta_x. 
> -- Por outro lado, "dy" e "Delta_y" podem ser bem diferentes (em nenhum 
> momento eu disse que dx ou dy são pequenos!). Isto dito, o grande barato da 
> derivada é que, voce pode usar a aproximação Delta_y ~= dy para Delta_x = dx 
> suficientemente pequeno! Por isso que muita gente acaba pensando em dy como 
> um "Delta_y infinitesimal" (uma intuição útil, mas apenas intuição -- repito 
> que dy tem o direito de ser imenso e muito diferente de Delta_y). 
> 
> Abraço, Ralph. 
> 
> On Sun, Jun 21, 2020 at 11:22 AM Pacini Bores  wrote: 
> 
> Olá Pessoal, 
> 
> Qual é a melhor forma de se definir a diferencial de uma função de uma única 
> variável ? 
> 
> Abraços 
> 
> Pacini 
> 
> -- 
> Esta mensagem foi verificada pelo sistema de antivírus e 
> acredita-se estar livre de perigo. 
> -- 
> Esta mensagem foi verificada pelo sistema de antivrus e 
> acredita-se estar livre de perigo.

 
-- 
Esta mensagem foi verificada pelo sistema de antiv�rus e
 acredita-se estar livre de perigo.



[obm-l] Re: [obm-l] Diferencial de uma função de uma variável

2020-06-21 Por tôpico Ralph Costa Teixeira
Voce diz, aquele "dy" sozinho?

Eu gosto de pensar assim: considere uma função f(x) diferenciável num ponto
a. A *linearizacão* de f(x) em x=a é dada por:
L(x) = f(a) + f'(a) (x-a)
e a ideia é que L(x) aproxima "bastante bem" f(x) ali perto de x=a (o
gráfico de L(x) é a reta tangente).

Para dar contexto, escreva y=f(x) e, a partir de "a", vamos aplicar uma
variação Delta_x (um número real, possivelmente grande), indo para
x=a+Delta_x. Esta variação no domínio provoca variação na imagem de f, a
saber:
Delta_y = Delta_f = f(a+Delta_x)-f(a)
Analogamente, olhe para L(x) e, a partir de "a", aplique uma variação de dx
(um número real, possivelmente grande), indo para x=a+dx. A *diferencial de
f no ponto a (associada a dx) *é
dy = Delta_L=L(a+dx)-L(a)
ou seja, dy é simplesmente *a variação em y MEDIDA PELA LINEARIZAÇÃO*, ou
seja, *USANDO A RETA TANGENTE* (ao invés de usar a f(x) original).

Note que podemos escrever dy explicitamente em termos de f, pois temos
aquela fórmula ali em cima para L:
dy =  L(a+dx)-L(a) = (f(a)+f'(a).dx)-(f(a)+f'(a).0) = f'(a).dx
Em suma:
dy = f'(a).dx
Esta última expressão é exatamente a equação da reta tangente, escrita dum
jeito mais curto (pois fizemos L(x)-L(a)=dy e x-a=dx)!

Comparando:
-- Não há diferença prática entre "dx" e "Delta_x"; apenas por convenção,
quando eu estiver trabalhando com a linearização, vou escrever dx ao invés
de Delta_x. Voce não perde praticamente nada se pensar que dx=Delta_x.
-- Por outro lado, "dy" e "Delta_y" podem ser bem diferentes (em nenhum
momento eu disse que dx ou dy são pequenos!). Isto dito, o grande barato da
derivada é que, voce pode usar a aproximação Delta_y ~= dy para Delta_x =
dx suficientemente pequeno! Por isso que muita gente acaba pensando em dy
como um "Delta_y infinitesimal" (uma intuição útil, mas apenas intuição --
repito que dy tem o direito de ser imenso e muito diferente de Delta_y).

Abraço, Ralph.


On Sun, Jun 21, 2020 at 11:22 AM Pacini Bores 
wrote:

> Olá Pessoal,
>
>
>
> Qual é a melhor forma de se definir a diferencial de uma função de uma
> única variável ?
>
> Abraços
>
> Pacini
>
>
> --
> Esta mensagem foi verificada pelo sistema de antivírus e
> acredita-se estar livre de perigo.
>

-- 
Esta mensagem foi verificada pelo sistema de antiv�rus e
 acredita-se estar livre de perigo.



Re: [obm-l] Diferencial de uma função de uma variável

2020-06-21 Por tôpico Maikel Andril Marcelino
Usando a definição de derivada:


$$f'(x) = \lim_{h\to 0} \frac{f(x+h) - f(x)}{h}$$


[cid:7dd090d3-df35-4082-a775-5efb0208b3d0]


Atenciosamente,

Maikel Andril Marcelino
Assistente de Aluno - Biblioteca - Ramal: 7616
Coordenadoria de Apoio Acadêmico - COAPAC/IFRN-SPP
Instituto Federal do Rio Grande do Norte
Campus São Paulo do Potengi

(84) 9-9149-8991 (Contato)
(84) 8851-3451 (WhatsApp)

De: owner-ob...@mat.puc-rio.br  em nome de Pacini 
Bores 
Enviado: domingo, 21 de junho de 2020 11:08
Para: obm-l@mat.puc-rio.br
Assunto: [obm-l] Diferencial de uma função de uma variável


Olá Pessoal,



Qual é a melhor forma de se definir a diferencial de uma função de uma única 
variável ?

Abraços

Pacini



--
Esta mensagem foi verificada pelo sistema de antiv?rus e
acredita-se estar livre de perigo.

-- 
Esta mensagem foi verificada pelo sistema de antivírus e
 acredita-se estar livre de perigo.



[obm-l] Re: [obm-l] Re: [obm-l] polígono regular - 13 lados

2020-06-18 Por tôpico Daniel Jelin
Pra mim deu 91 também: C(13,3) - 13*C(6,2).
Acho que dá pra generalizar para polígonos regulares de 2n+1 lados: serão
C(2n+1,3) - (2n+1)*C(n,2) triângulos, que significa o total de triângulos
menos aqueles cujos vértices estão todos de uma mesma 'banda' do polígono.
abs,
Daniel


Livre
de vírus. www.avast.com
.
<#DAB4FAD8-2DD7-40BB-A1B8-4E2AA1F9FDF2>

On Thu, Jun 18, 2020 at 11:48 PM Ralph Costa Teixeira 
wrote:

> Hm... Fiz um raciocínio aqui, confiram se errei algo. Vou chamar os
> vértices de P1, P2, ..., P13.
>
> Primeiro: o enunciado tinha que deixar mais claro como contar
> triângulos... Por exemplo, triângulos congruentes em si contam apenas uma
> vez? P1P2P6 conta igual a P2P3P7? Normalmente, eu diria que eles são
> **distintos**, mas neste caso a resposta seria muito mais que 36 (e seria
> um múltiplo de 13, pois, para cada triângulo válido, teríamos suas 13
> rotações também, que seriam distintas).
>
> ---///---
>
> Mas vamos supor que o enunciado considera triângulos congruentes como um
> único triângulo. Neste caso, o triângulo ABC (suponha A-B-C no sentido
> anti-horário) fica completamente determinado pelos comprimentos dos 3 arcos
> AB, AC e BC no círculo circunscrito (e vice-versa: dados os 3 arcos, em
> qualquer ordem, eles determinam os comprimentos dos lados, e portanto
> determinariam um único triângulo). Escrevendo os arcos como AB=x.2pi/13,
> BC=y.2pi/13 e CA=z.2pi/13, um triângulo ABC corresponde exatamente a uma
> tripla ***desordenada*** de inteiros positivos {x,y,z} satisfazendo
> x+y+z=13.
>
> Para que o circuncentro esteja no interior do triângulo, basta que ele
> seja acutângulo, ou seja, x,y,z<=6. Então agora temos um problema
> combinatório:
>
> "Determinar o número de soluções inteiras de x+y+z=13 satisfazendo
> 1<=x,y,z<=6, onde a ordem das variáveis não importa."
> Fazendo a=6-x, b=6-y e c=6-z, o problema vira
> "Determinar o número de soluções inteiras distintas de a+b+c=5
> satisfazendo 0<=a,b,c<=5 (sem ordem)"
> Opa, assim o "<=5" fica desnecessário, pois a+b+c=5 e a,b,c>=0 implicam
> a,b,c<=5! Então agora é (quase) um problema clássico daqueles com bolinhas
> e barrinhas para separar bolinhas... "Quase" porque dissemos que a ordem
> não importa! Como os números sao pequenos, melhor fazer logo no braço...
> Suponha s.p.d.g que a>=b>=c, e teste a=5, depois a=4... e liste os casos:
> {a,b,c}={{5,0,0},{4,1,0},{3,2,0},{3,1,1},{2,2,1}}
> Ou seja, sao apenas 5 triangulos:
> {x,y,z} = {1,6,6},{2,5,6},{3,4,6},{3,5,5},{4,4,5}
>
> ---///---
>
> Agora, se você quiser a minha interpretação original onde cada posição de
> cada vértice importa... Bom, basta notar que:
> a) Cada uma das triplas (1,6,6), (3,5,5) e (4,4,5) gera 13 triângulos
> (tome um triângulo desse tipo e rode sucessivamente de ângulo 2pi/13)
> b) Cada uma das triplas (2,5,6) e (3,4,6) gera 2x13=26 triângulos... Isto
> ocorre aqui pois (2,5,6) gera um triângulo "distinto" de (2,6,5) (e um não
> pode ser obtido do outro por rotações, pois estas mantêm a ordem circular
> dos números).
> (Note que esta "duplicação" não ocorria em (a) por conta dos números
> repetidos! Por exemplo, se você tentasse montar um triângulo (6,6,1),
> digamos, P1-P7-P13, ele seria uma das rotações do (1,6,6) que eu jah tinha
> contado (a saber: P13-P1-P7). Ou seja, temos que contar todas as
> permutações de cada tripla, dividindo por 3 por conta das
> 3 permutações circulares que não geram nada de novo.)
> Então com a minha interpretação original a resposta seria (3x1+2x2) x 13 =
> 91? Errei algo?
>
> Abraço, Ralph.
>
>
>
>
> On Thu, Jun 18, 2020 at 9:31 PM Vitório Batista Lima da Silva <
> vitorio.si...@trf1.jus.br> wrote:
>
>> 3 vértices distintos de um polígono regular de 13 lados formam um
>> triângulo. Quantos desses triângulos contém o centro do círculo
>> circunscrito ao polígono?
>>
>> A resposta é 36???
>>
>> At.te,
>>
>> Vitório
>>
>> --
>> Esta mensagem foi verificada pelo sistema de antivírus e
>> acredita-se estar livre de perigo.
>>
>
> --
> Esta mensagem foi verificada pelo sistema de antivírus e
> acredita-se estar livre de perigo.



Livre
de vírus. www.avast.com
.
<#DAB4FAD8-2DD7-40BB-A1B8-4E2AA1F9FDF2>

-- 
Esta mensagem foi verificada pelo sistema de antiv�rus e
 acredita-se estar livre de perigo.



[obm-l] Re: [obm-l] polígono regular - 13 lados

2020-06-18 Por tôpico Ralph Costa Teixeira
Hm... Fiz um raciocínio aqui, confiram se errei algo. Vou chamar os
vértices de P1, P2, ..., P13.

Primeiro: o enunciado tinha que deixar mais claro como contar triângulos...
Por exemplo, triângulos congruentes em si contam apenas uma vez? P1P2P6
conta igual a P2P3P7? Normalmente, eu diria que eles são **distintos**, mas
neste caso a resposta seria muito mais que 36 (e seria um múltiplo de 13,
pois, para cada triângulo válido, teríamos suas 13 rotações também, que
seriam distintas).

---///---

Mas vamos supor que o enunciado considera triângulos congruentes como um
único triângulo. Neste caso, o triângulo ABC (suponha A-B-C no sentido
anti-horário) fica completamente determinado pelos comprimentos dos 3 arcos
AB, AC e BC no círculo circunscrito (e vice-versa: dados os 3 arcos, em
qualquer ordem, eles determinam os comprimentos dos lados, e portanto
determinariam um único triângulo). Escrevendo os arcos como AB=x.2pi/13,
BC=y.2pi/13 e CA=z.2pi/13, um triângulo ABC corresponde exatamente a uma
tripla ***desordenada*** de inteiros positivos {x,y,z} satisfazendo
x+y+z=13.

Para que o circuncentro esteja no interior do triângulo, basta que ele seja
acutângulo, ou seja, x,y,z<=6. Então agora temos um problema combinatório:

"Determinar o número de soluções inteiras de x+y+z=13 satisfazendo
1<=x,y,z<=6, onde a ordem das variáveis não importa."
Fazendo a=6-x, b=6-y e c=6-z, o problema vira
"Determinar o número de soluções inteiras distintas de a+b+c=5 satisfazendo
0<=a,b,c<=5 (sem ordem)"
Opa, assim o "<=5" fica desnecessário, pois a+b+c=5 e a,b,c>=0 implicam
a,b,c<=5! Então agora é (quase) um problema clássico daqueles com bolinhas
e barrinhas para separar bolinhas... "Quase" porque dissemos que a ordem
não importa! Como os números sao pequenos, melhor fazer logo no braço...
Suponha s.p.d.g que a>=b>=c, e teste a=5, depois a=4... e liste os casos:
{a,b,c}={{5,0,0},{4,1,0},{3,2,0},{3,1,1},{2,2,1}}
Ou seja, sao apenas 5 triangulos:
{x,y,z} = {1,6,6},{2,5,6},{3,4,6},{3,5,5},{4,4,5}

---///---

Agora, se você quiser a minha interpretação original onde cada posição de
cada vértice importa... Bom, basta notar que:
a) Cada uma das triplas (1,6,6), (3,5,5) e (4,4,5) gera 13 triângulos (tome
um triângulo desse tipo e rode sucessivamente de ângulo 2pi/13)
b) Cada uma das triplas (2,5,6) e (3,4,6) gera 2x13=26 triângulos... Isto
ocorre aqui pois (2,5,6) gera um triângulo "distinto" de (2,6,5) (e um não
pode ser obtido do outro por rotações, pois estas mantêm a ordem circular
dos números).
(Note que esta "duplicação" não ocorria em (a) por conta dos números
repetidos! Por exemplo, se você tentasse montar um triângulo (6,6,1),
digamos, P1-P7-P13, ele seria uma das rotações do (1,6,6) que eu jah tinha
contado (a saber: P13-P1-P7). Ou seja, temos que contar todas as
permutações de cada tripla, dividindo por 3 por conta das
3 permutações circulares que não geram nada de novo.)
Então com a minha interpretação original a resposta seria (3x1+2x2) x 13 =
91? Errei algo?

Abraço, Ralph.




On Thu, Jun 18, 2020 at 9:31 PM Vitório Batista Lima da Silva <
vitorio.si...@trf1.jus.br> wrote:

> 3 vértices distintos de um polígono regular de 13 lados formam um
> triângulo. Quantos desses triângulos contém o centro do círculo
> circunscrito ao polígono?
>
> A resposta é 36???
>
> At.te,
>
> Vitório
>
> --
> Esta mensagem foi verificada pelo sistema de antivírus e
> acredita-se estar livre de perigo.
>

-- 
Esta mensagem foi verificada pelo sistema de antiv�rus e
 acredita-se estar livre de perigo.



[obm-l] Re: [obm-l] Re: [obm-l] Números complexos e equações

2020-06-18 Por tôpico Prof. Douglas Oliveira
Opa mestre Claudio, muito obrigado, gostei da solução.

Douglas Oliveira

Em qua, 17 de jun de 2020 17:00, Claudio Buffara 
escreveu:

> Aquele 1+i sugere que se forme uma equação em z, onde z = (1+i)/raiz(2) *
> x, ou seja, cujas raízes sejam as da equação original giradas de 45 graus
> no sentido anti-horário e sem coeficientes complexos.
> z = (1+i)/raiz(2) * x ==> x = (1-i)/raiz(2) * z
> Assim, x^4 + 4(1+i)x + 1 = 0 ==> -z^4 + 4*raiz(2)*z + 1 = 0
> Consideremos f(z) = z^4 - 4*raiz(2)*z - 1  (multiplicar os coeficientes
> por -1 não altera as raízes).
> f(-1) = 4*raiz(2) > 0
> f(0) = -1 < 0
> f(raiz(2)) = -5 < 0
> f(2) =15 - 8*raiz(2) > 0 ==> f tem (pelo menos) duas raízes reais: uma
> entre -1 e 0 e outra entre raiz(2) e 2.
> Mas f'(z) = 4z^3 - 4*raiz(2) ==> f'(z) < 0 para z < 2^(1/6) (logo, para z
> < 0) e f'(z) > 0 para z > 2^(1/6) (logo, para z > raiz(2)), de modo que
> estas são as únicas raízes reais de f.
> Se duas das raízes da equação original, ao serem giradas de 45 graus no
> sentido anti-horário, se tornam reais, então aquelas raízes estavam na reta
> Im(z) = -Re(z).
> Além disso, como, após giradas, uma se tornou negativa e a outra positiva,
> isso significa que a primeira está no 2o quadrante e a segunda no 4o
> quadrante.
>
> Dadas as magnitudes das raízes giradas (a primeira entre -1 e 0 e a
> segunda maior do que raiz(2)), também concluímos que a soma delas está no
> 4o quadrante, ou seja, é da forma p*(1-i), com p > 0.
> Além disso, o produto delas é da forma (1/q)*i, com q > 0.
>
> Chame as outras duas raízes da equação original de a e b.
> Então, como a soma das raízes é zero, vale a+b = -p(1-i) = p(-1+i): um
> ponto do 2o quadrante sobre a reta Im(z) = -Re(z)   (1)
> Como o produto das raízes é 1, vale a*b = -q*i, um ponto do eixo
> imaginário negativo
> A localização do produto a*b implica que a/|a| e b/|b| são números
> complexos (de módulo unitário) e simétricos em relação à reta Im(z) =
> -Re(z)   (2)
> (1) e (2) implicam que a e b têm o mesmo módulo R
>
> (2) também implica que, sobre a e b:
> OU ambos pertencem ao 2o quadrante
> OU um deles pertence ao 1o e o outro ao 3o quadrante
> OU ambos pertencem ao 4o quadrante.
>
> De cara dá pra eliminar a última alternativa, já que isso implicaria que
> a+b pertence ao 4o quadrante, o que não é o caso.
>
> Resta eliminar a 1a alternativa.
> Assim, suponhamos que a e b pertencem ao 2o quadrante.
>
> Neste caso, a+b = p(-1+i) ==> |a+b| = p*raiz(2) > R*raiz(2) ==> 2*p^2 >
> 2*R^2
> E também, de qualquer jeito, ab = -qi ==> q = R^2
>
> Da equação, também sabemos que ab + ac + ad + bc + bd + cd = 0 ==>
> ab + cd + (a+b)(c+d) = 0 ==>
> -q*i + (1/q)*i + p(-1+i)*p*(1-i) = 0 ==>
> 1/q - q + 2p^2 = 0
> 1/q - q + 2R^2 < 0 ==>
> 1/R^2 - R^2 + 2*R^2 < 1/R^2 + < 0 ==> contradição ==> a e b não pertencem
> ao 2o quadrante.
>
> Logo, temos que concluir que, sobre as outras duas raízes, que uma
> pertence ao 1o e a outra ao 3o quadrante.
>
> []s,
> Claudio.
>
>
> On Wed, Jun 17, 2020 at 9:01 AM Prof. Douglas Oliveira <
> profdouglaso.del...@gmail.com> wrote:
>
>> Olá, gostaria de uma ajuda para localizar as raízes da
>> equação x^4+4(1+i)x+1=0, saber em qual quadrante estão, joguei no MAPLE e
>> percebi que existe uma em cada quadrante.
>>
>> Mas não consigo achar uma saída.
>>
>> Obrigado.
>> Douglas Oliveira
>>
>> --
>> Esta mensagem foi verificada pelo sistema de antivírus e
>> acredita-se estar livre de perigo.
>
>
> --
> Esta mensagem foi verificada pelo sistema de antivírus e
> acredita-se estar livre de perigo.

-- 
Esta mensagem foi verificada pelo sistema de antiv�rus e
 acredita-se estar livre de perigo.



[obm-l] Re: [obm-l] Números complexos e equações

2020-06-17 Por tôpico Claudio Buffara
Aquele 1+i sugere que se forme uma equação em z, onde z = (1+i)/raiz(2) *
x, ou seja, cujas raízes sejam as da equação original giradas de 45 graus
no sentido anti-horário e sem coeficientes complexos.
z = (1+i)/raiz(2) * x ==> x = (1-i)/raiz(2) * z
Assim, x^4 + 4(1+i)x + 1 = 0 ==> -z^4 + 4*raiz(2)*z + 1 = 0
Consideremos f(z) = z^4 - 4*raiz(2)*z - 1  (multiplicar os coeficientes por
-1 não altera as raízes).
f(-1) = 4*raiz(2) > 0
f(0) = -1 < 0
f(raiz(2)) = -5 < 0
f(2) =15 - 8*raiz(2) > 0 ==> f tem (pelo menos) duas raízes reais: uma
entre -1 e 0 e outra entre raiz(2) e 2.
Mas f'(z) = 4z^3 - 4*raiz(2) ==> f'(z) < 0 para z < 2^(1/6) (logo, para z <
0) e f'(z) > 0 para z > 2^(1/6) (logo, para z > raiz(2)), de modo que estas
são as únicas raízes reais de f.
Se duas das raízes da equação original, ao serem giradas de 45 graus no
sentido anti-horário, se tornam reais, então aquelas raízes estavam na reta
Im(z) = -Re(z).
Além disso, como, após giradas, uma se tornou negativa e a outra positiva,
isso significa que a primeira está no 2o quadrante e a segunda no 4o
quadrante.

Dadas as magnitudes das raízes giradas (a primeira entre -1 e 0 e a segunda
maior do que raiz(2)), também concluímos que a soma delas está no 4o
quadrante, ou seja, é da forma p*(1-i), com p > 0.
Além disso, o produto delas é da forma (1/q)*i, com q > 0.

Chame as outras duas raízes da equação original de a e b.
Então, como a soma das raízes é zero, vale a+b = -p(1-i) = p(-1+i): um
ponto do 2o quadrante sobre a reta Im(z) = -Re(z)   (1)
Como o produto das raízes é 1, vale a*b = -q*i, um ponto do eixo imaginário
negativo
A localização do produto a*b implica que a/|a| e b/|b| são números
complexos (de módulo unitário) e simétricos em relação à reta Im(z) =
-Re(z)   (2)
(1) e (2) implicam que a e b têm o mesmo módulo R

(2) também implica que, sobre a e b:
OU ambos pertencem ao 2o quadrante
OU um deles pertence ao 1o e o outro ao 3o quadrante
OU ambos pertencem ao 4o quadrante.

De cara dá pra eliminar a última alternativa, já que isso implicaria que
a+b pertence ao 4o quadrante, o que não é o caso.

Resta eliminar a 1a alternativa.
Assim, suponhamos que a e b pertencem ao 2o quadrante.

Neste caso, a+b = p(-1+i) ==> |a+b| = p*raiz(2) > R*raiz(2) ==> 2*p^2 >
2*R^2
E também, de qualquer jeito, ab = -qi ==> q = R^2

Da equação, também sabemos que ab + ac + ad + bc + bd + cd = 0 ==>
ab + cd + (a+b)(c+d) = 0 ==>
-q*i + (1/q)*i + p(-1+i)*p*(1-i) = 0 ==>
1/q - q + 2p^2 = 0
1/q - q + 2R^2 < 0 ==>
1/R^2 - R^2 + 2*R^2 < 1/R^2 + < 0 ==> contradição ==> a e b não pertencem
ao 2o quadrante.

Logo, temos que concluir que, sobre as outras duas raízes, que uma pertence
ao 1o e a outra ao 3o quadrante.

[]s,
Claudio.


On Wed, Jun 17, 2020 at 9:01 AM Prof. Douglas Oliveira <
profdouglaso.del...@gmail.com> wrote:

> Olá, gostaria de uma ajuda para localizar as raízes da
> equação x^4+4(1+i)x+1=0, saber em qual quadrante estão, joguei no MAPLE e
> percebi que existe uma em cada quadrante.
>
> Mas não consigo achar uma saída.
>
> Obrigado.
> Douglas Oliveira
>
> --
> Esta mensagem foi verificada pelo sistema de antivírus e
> acredita-se estar livre de perigo.

-- 
Esta mensagem foi verificada pelo sistema de antiv�rus e
 acredita-se estar livre de perigo.



[obm-l] Re: [obm-l] Re: [obm-l] Re: [obm-l] Conjuntos não enumeráveis

2020-06-15 Por tôpico Anderson Torres
Em seg., 15 de jun. de 2020 às 23:31, Israel Meireles Chrisostomo
 escreveu:
>
> usa a bijeção da tangente no intervalo 0 a pi sobre 2

Sim, e o que isso implica? Que a tangente mapeia esse intervalo nos
reais, logo ambos terão o mesmo tamanho - mas onde você demonstrou que
um desses não é enumerável? No máximo você demonstrou que um certo
conjunto tem bijeção com um subconjunto de si mesmo - que é meio que
uma definição de infinito.

>
> Em seg., 15 de jun. de 2020 às 21:38, Anderson Torres 
>  escreveu:
>>
>> Não entendi a última parte.
>>
>> Em dom., 14 de jun. de 2020 Ã s 18:24, Israel Meireles Chrisostomo
>>  escreveu:
>> >
>> >
>> > https://www.overleaf.com/read/cwxhsnctfxcf
>> > Nesse link eu demostro trigonmetricamente que o conjunto dos irracionais 
>> > é não enumerável.
>> > --
>> > Israel Meireles Chrisostomo
>> >
>> > --
>> > Esta mensagem foi verificada pelo sistema de antivírus e
>> > acredita-se estar livre de perigo.
>>
>> --
>> Esta mensagem foi verificada pelo sistema de antivírus e
>>  acredita-se estar livre de perigo.
>>
>>
>> =
>> Instruções para entrar na lista, sair da lista e usar a lista em
>> http://www.mat.puc-rio.br/~obmlistas/obm-l.html
>> =
>
>
>
> --
> Israel Meireles Chrisostomo
>
> --
> Esta mensagem foi verificada pelo sistema de antivírus e
> acredita-se estar livre de perigo.

-- 
Esta mensagem foi verificada pelo sistema de antiv�rus e
 acredita-se estar livre de perigo.


=
Instru��es para entrar na lista, sair da lista e usar a lista em
http://www.mat.puc-rio.br/~obmlistas/obm-l.html
=


[obm-l] Re: [obm-l] Re: [obm-l] Conjuntos não enumeráveis

2020-06-15 Por tôpico Israel Meireles Chrisostomo
usa a bijeção da tangente no intervalo 0 a pi sobre 2

Em seg., 15 de jun. de 2020 às 21:38, Anderson Torres <
torres.anderson...@gmail.com> escreveu:

> Não entendi a última parte.
>
> Em dom., 14 de jun. de 2020 Ã s 18:24, Israel Meireles Chrisostomo
>  escreveu:
> >
> >
> > https://www.overleaf.com/read/cwxhsnctfxcf
> > Nesse link eu demostro trigonmetricamente que o conjunto dos irracionais
> é não enumerável.
> > --
> > Israel Meireles Chrisostomo
> >
> > --
> > Esta mensagem foi verificada pelo sistema de antivírus e
> > acredita-se estar livre de perigo.
>
> --
> Esta mensagem foi verificada pelo sistema de antivírus e
>  acredita-se estar livre de perigo.
>
>
> =
> Instruções para entrar na lista, sair da lista e usar a lista em
> http://www.mat.puc-rio.br/~obmlistas/obm-l.html
> =
>


-- 
Israel Meireles Chrisostomo

-- 
Esta mensagem foi verificada pelo sistema de antiv�rus e
 acredita-se estar livre de perigo.



Re: [obm-l] Normas

2020-06-15 Por tôpico João Pedro de Abreu Marciano
Dado M>1. Definimos f(x) = 0 se 1/M0 tal que | f |_infinito <= B*|
f |_1 para todo f. Ou seja, as normas não são equivalentes.

Espero ter ajudado,
João Pedro Marciano.

Em seg., 15 de jun. de 2020 às 22:46, Pedro Júnior <
pedromatematic...@gmail.com> escreveu:

> [image: image.png]
> Alguém pode me ajudar nesse problema?
>
> --
>
> Pedro Jerônimo S. de O. Júnior
>
> Professor de Matemática
>
> João Pessoa – PB
>
> --
> Esta mensagem foi verificada pelo sistema de antivírus e
> acredita-se estar livre de perigo.

-- 
Esta mensagem foi verificada pelo sistema de antiv�rus e
 acredita-se estar livre de perigo.



[obm-l] Re: [obm-l] Conjuntos não enumeráveis

2020-06-15 Por tôpico Anderson Torres
Não entendi a última parte.

Em dom., 14 de jun. de 2020 às 18:24, Israel Meireles Chrisostomo
 escreveu:
>
>
> https://www.overleaf.com/read/cwxhsnctfxcf
> Nesse link eu demostro trigonmetricamente que o conjunto dos irracionais é 
> não enumerável.
> --
> Israel Meireles Chrisostomo
>
> --
> Esta mensagem foi verificada pelo sistema de antivírus e
> acredita-se estar livre de perigo.

-- 
Esta mensagem foi verificada pelo sistema de antiv�rus e
 acredita-se estar livre de perigo.


=
Instru��es para entrar na lista, sair da lista e usar a lista em
http://www.mat.puc-rio.br/~obmlistas/obm-l.html
=


Re: [obm-l] construção geométrica

2020-06-10 Por tôpico Claudio Buffara
Se o triângulo for equilátero, qualquer ponto do arco AB serve.

Enviado do meu iPhone

> Em 10 de jun de 2020, à(s) 17:24, Luís Lopes  escreveu:
> 

-- 
Esta mensagem foi verificada pelo sistema de antiv�rus e
 acredita-se estar livre de perigo.


=
Instru��es para entrar na lista, sair da lista e usar a lista em
http://www.mat.puc-rio.br/~obmlistas/obm-l.html
=


Re: [obm-l] site enem matemática

2020-06-05 Por tôpico Claudio Buffara
Pra mim, a melhor forma de se preparar é baixar as provas passadas do site do 
INEP e resolver as questões. Se vc resolver as provas dos últimos 5 ou 6 anos, 
estará bem preparado.

Se empacar em alguma questão, poste a dúvida aqui que alguém poderá responder 
(apesar deste ser um grupo de olimpíadas, pelo menos em tese).

O YouTube deve ter vídeos com soluções de todas as questões dos últimos anos.

E se você quiser a teoria também há sites pra isso. O maior deles é o 
Descomplica.

Abs

> Em 5 de jun de 2020, à(s) 09:41, carlos h Souza  
> escreveu:
> 
> 
> Bom dia,
> 
> Alguém sabe de algum site para treinamento de questões do ENEM, só de 
> matemática e raciocínio lógico ???
> 
> Obrigado
> 
> -- 
> Esta mensagem foi verificada pelo sistema de antivírus e 
> acredita-se estar livre de perigo.

-- 
Esta mensagem foi verificada pelo sistema de antiv�rus e
 acredita-se estar livre de perigo.


=
Instru��es para entrar na lista, sair da lista e usar a lista em
http://www.mat.puc-rio.br/~obmlistas/obm-l.html
=


[obm-l] Re: [obm-l] Encontrar K mínimo

2020-05-23 Por tôpico Ralph Costa Teixeira
Hm: "qualquer uma das 2021 possui voo direto", assim mesmo? Tenho ideias,
mas tem que completar.

Vou dizer que uma marcação "cuida" de uma cidade X quando existe alguma
cidade marcada com voo direto para X. Observe que, interpretando ao pé da
letra o enunciado, X não necessariamente cuida de X!

LEMA 1: Para um grafo do tipo "filete", Y1-Y2-...-YN conectadas em linha,
N>=3, o número mínimo f(N) de cidades marcadas satisfaz f(N)<=2N/3. Mais:
se N não é divisível por 3, podemos "economizar mais uma marcação" para
obter f(N)<2N/3.

Prova:
-- Para N=3, marque Y1Y2, portanto f(3)<=2.
-- Para N=4, marque Y2Y3, portanto f(4)<=2<8/3.
-- Para N=5, marque Y2Y3Y4, portanto f(5)<=3<10/3.
-- Para N=6, marque Y1Y2Y4Y5, portanto f(6)<=4.
-- Para N=7, marque Y2Y3Y5Y6 portanto f(7)<=4<14/3.
-- Para N=8, marque Y2Y3Y6Y7, portanto f(8)<=4<16/3.
...
Em geral, para N>=6, a marcação depende do resto de N na divisão por 3:
i) Se N=3k, divida as cidades em grupos de 3 e marque as 2 primeiras de
cada grupo: note que cada 2 cidades "cuidam" do seu grupo de 3, portanto
f(N)<=2N/3.
ii) Se N=3k+1, separe Y1, e divida **as outras** em grupos de 3; novamente,
marque as 2 primeiras de cada grupo (note que Y2 cuida de Y1!):
f(N)<=2(N-1)/3<2N/3.
iii) Se N=3k+2, separe as 5 primeiras e divida o resto em grupos de 3;
marque Y2Y3 e as 2 primeiras de cada grupo (observe que Y6 cuida de Y5).
Assim f(N)<=2(N-5)/3+2<2N/3.

(Obs: de fato, para N grande, o ideal se parece mais com f(N)~~N/2: divida
em grupos de 4, escolha as duas cidade do meio em cada grupo.)

---///---

Isso me faz desconfiar que o pior caso seria algo do tipo 2N/3, se tivermos
filetes de tamanho 3 ou 6. Assim fica fácil construir um exemplo onde
~~2N/3 seria necessário:

---///---

LEMA 2: k>=(2/3)*2021-1, ou seja, k>=1347.

Prova. Considere o caso onde as cidades são A, B1, B2, ..., B673, C1, C2,
..., C673, D1, D2, ..., D673, Z. Conecte apenas assim:
-- A conecta com Z;
-- A conecta com Bi para todo i;
-- Bi conecta com Ci para todo i;
-- Ci conecta com Di para todo i.
Em suma, uma cidade "central" A, donde saem 673 filetes de comprimento 3 (e
mais um filetezinho para conectar com a cidade que sobrou).

Para cuidar de Di, temos que marcar Ci. Para cuidar de Ci, temos que marcar
Bi ou Di (sendo esperto, escolhemos pelo menos um Bi para cuidar de A);
para cuidar de Z temos que marcar A.

Em suma, neste caso, a marcação deve ter pelo menos 673*2+1=1347 cidades.
---///---

Agora o que eu queria fazer era dividir o grafo em vários "filetes"
*disjuntos*, e em cada filete fazer uma marcação com 2/3 dos vértices
daquele filete, cuidando daquele filete. Juntando tudo, teríamos 2/3 dos
vértices marcados; como pelo menos um filete teria numero de vértices não
divisível por 3, eu "economizaria" uma marcação nele, chegando aos 1347.

O problema seria garantir que esses filetes tem comprimento >=3 para poder
usar o 2N/3... Se ficarmos com filetes de tamanho 1 ou 2, f(2)=2 estraga
tudo (para o de comprimento 1, também teríamos problemas). Então minha
ideia não funciona assim direto. Mas talvez ajude a pensar, ou achar um
pior caso que seja pior que a minha "estrela" ali em cima...

Abraço, Ralph.

On Sat, May 23, 2020 at 11:46 AM Jeferson Almir 
wrote:

> Amigos peço ajuda nesse problema, ou até algum resultado de grafos que
> resolva.
>
> Terra Brasilis  possui 2021 cidades, e existem voos de ida e volta entre
> algumas dessas  cidades de maneira que é possível chegar a qualquer outra
> através de voos finitos. Encontre o menor inteiro positivo k tal que,
> independente da configuração dos voos, é possível escolher k cidades de
> modo que qualquer uma das 2021 cidades possui voo direto para alguma das
> cidades marcadas.
>
> --
> Esta mensagem foi verificada pelo sistema de antivírus e
> acredita-se estar livre de perigo.

-- 
Esta mensagem foi verificada pelo sistema de antiv�rus e
 acredita-se estar livre de perigo.



[obm-l] Re: [obm-l] Re: [obm-l] Encontrar K mínimo

2020-05-23 Por tôpico Jeferson Almir
Voos finitos = é sempre possível chegar com uma certa quantidade de voos.
 Os casos iniciais que fiz me pareceu uma conjectura muito “ óbvia “, mas
não tenho certeza.
* não existe Voo  de B para B

Em sáb, 23 de mai de 2020 às 12:54, Bernardo Freitas Paulo da Costa <
bernardo...@gmail.com> escreveu:

> On Sat, May 23, 2020 at 11:46 AM Jeferson Almir
>  wrote:
> >
> > Amigos peço ajuda nesse problema, ou até algum resultado de grafos que
> resolva.
> >
> > Terra Brasilis  possui 2021 cidades, e existem voos de ida e volta entre
> algumas dessas  cidades de maneira que é possível chegar a qualquer outra
> através de voos finitos. Encontre o menor inteiro positivo k tal que,
> independente da configuração dos voos, é possível escolher k cidades de
> modo que qualquer uma das 2021 cidades possui voo direto para alguma das
> cidades marcadas.
>
> Não entendi o que quer dizer "vôos finitos"...  Tem um enunciado mais
> preciso, ou é isso aí?
>
> E, para tentar ajudar: o que acontece se forem 2 cidades? 3?  Aliás,
> uma pergunta, se o caso de 2 cidades for A - B, não tem "vôos de B
> para B", então você teria que escolher as cidades (A,B) ?  Ou também
> está faltando esse detalhe no enunciado?
>
>
> Abraços,
> --
> Bernardo Freitas Paulo da Costa
>
> --
> Esta mensagem foi verificada pelo sistema de antivírus e
>  acredita-se estar livre de perigo.
>
>
> =
> Instru�ões para entrar na lista, sair da lista e usar a lista em
> http://www.mat.puc-rio.br/~obmlistas/obm-l.html
> =
>

-- 
Esta mensagem foi verificada pelo sistema de antiv�rus e
 acredita-se estar livre de perigo.



[obm-l] Re: [obm-l] Encontrar K mínimo

2020-05-23 Por tôpico Bernardo Freitas Paulo da Costa
On Sat, May 23, 2020 at 11:46 AM Jeferson Almir
 wrote:
>
> Amigos peço ajuda nesse problema, ou até algum resultado de grafos que 
> resolva.
>
> Terra Brasilis  possui 2021 cidades, e existem voos de ida e volta entre 
> algumas dessas  cidades de maneira que é possível chegar a qualquer outra 
> através de voos finitos. Encontre o menor inteiro positivo k tal que, 
> independente da configuração dos voos, é possível escolher k cidades de modo 
> que qualquer uma das 2021 cidades possui voo direto para alguma das cidades 
> marcadas.

Não entendi o que quer dizer "vôos finitos"...  Tem um enunciado mais
preciso, ou é isso aí?

E, para tentar ajudar: o que acontece se forem 2 cidades? 3?  Aliás,
uma pergunta, se o caso de 2 cidades for A - B, não tem "vôos de B
para B", então você teria que escolher as cidades (A,B) ?  Ou também
está faltando esse detalhe no enunciado?


Abraços,
-- 
Bernardo Freitas Paulo da Costa

-- 
Esta mensagem foi verificada pelo sistema de antiv�rus e
 acredita-se estar livre de perigo.


=
Instru��es para entrar na lista, sair da lista e usar a lista em
http://www.mat.puc-rio.br/~obmlistas/obm-l.html
=


Re: [obm-l] Teoria dos numeros

2020-05-22 Por tôpico Anderson Torres
Em ter., 19 de mai. de 2020 às 15:52, Israel Meireles Chrisostomo
 escreveu:
>
> Olá pessoal.Ultimamente tenho pensado em como provar que a tangente de um 
> arco racional diferente de zero é sempre irracional.

Cê diz que se r é racional então tan(r) é irracional (exceto se r=0)?

Acho que dá para ser mais arrojado e provar logo a transcendência.
Afinal, qualquer racional pode ser multiplicado e dividido até dar 1,
e as funções tangente de múltiplo arco são racionais no arco primeiro,
e se tan 1 é transcedente, acabou.

Eu consegui chegar no seguinte: Se r é real diferente  de zero e s é
inteiro diferente de zero, então ou tan(r-1/2s)  ou tan(r) é
irracional.
> Daí então eu tomo um r racional,  então ou tan(r-1/2s)  ou tan(r) é 
> irracional, se tan(r) é irracional então está provado, se por um outro lado 
> tan(r-1/2s)  é irracional então faça r= r'+1/2s e daí tem-se tan(r') é 
> irracional.O que mostra que a tangente de todo arco racional diferente de 
> zero é irracional.
> Está correto esse meu raciocínio?
> Partindo de que "se r é real diferente  de zero e s é inteiro diferente de 
> zero, então ou tan(r-1/2s)  ou tan(r) é irracional "  como posso provar isso ?
> --
> Israel Meireles Chrisostomo
>
> --
> Esta mensagem foi verificada pelo sistema de antivírus e
> acredita-se estar livre de perigo.

-- 
Esta mensagem foi verificada pelo sistema de antiv�rus e
 acredita-se estar livre de perigo.


=
Instru��es para entrar na lista, sair da lista e usar a lista em
http://www.mat.puc-rio.br/~obmlistas/obm-l.html
=


[obm-l] Re: [obm-l] Re: [obm-l] Olimpíada de Matemática Online?

2020-05-18 Por tôpico Caio Costa
gqmo.org

Em seg., 18 de mai. de 2020 às 18:33, Caio Costa 
escreveu:

> Teve a GQMO esse mês. gqmo.org.br
>
> Em seg., 18 de mai. de 2020 às 12:14, Victor Pompêo 
> escreveu:
>
>> Eu conheço a Purple Comet:
>> https://purplecomet.org/?action=information/summary
>>
>> --
>> Victor
>>
>>
>> On Mon, 18 May 2020 at 11:52, Anderson Torres <
>> torres.anderson...@gmail.com> wrote:
>>
>>> Não lembro onde vi, acho que foi no AOPS/Mathlinks, mas existem
>>> iniciativas de olimpíadas de matemática feitas online?
>>>
>>> --
>>> Esta mensagem foi verificada pelo sistema de antivírus e
>>>  acredita-se estar livre de perigo.
>>>
>>>
>>> =
>>> Instruções para entrar na lista, sair da lista e usar a lista em
>>> http://www.mat.puc-rio.br/~obmlistas/obm-l.html
>>> =
>>>
>>
>> --
>> Esta mensagem foi verificada pelo sistema de antivírus e
>> acredita-se estar livre de perigo.
>
>

-- 
Esta mensagem foi verificada pelo sistema de antiv�rus e
 acredita-se estar livre de perigo.



[obm-l] Re: [obm-l] Re: [obm-l] Olimpíada de Matemática Online?

2020-05-18 Por tôpico Caio Costa
Teve a GQMO esse mês. gqmo.org.br

Em seg., 18 de mai. de 2020 às 12:14, Victor Pompêo 
escreveu:

> Eu conheço a Purple Comet:
> https://purplecomet.org/?action=information/summary
>
> --
> Victor
>
>
> On Mon, 18 May 2020 at 11:52, Anderson Torres <
> torres.anderson...@gmail.com> wrote:
>
>> Não lembro onde vi, acho que foi no AOPS/Mathlinks, mas existem
>> iniciativas de olimpíadas de matemática feitas online?
>>
>> --
>> Esta mensagem foi verificada pelo sistema de antivírus e
>>  acredita-se estar livre de perigo.
>>
>>
>> =
>> Instruções para entrar na lista, sair da lista e usar a lista em
>> http://www.mat.puc-rio.br/~obmlistas/obm-l.html
>> =
>>
>
> --
> Esta mensagem foi verificada pelo sistema de antivírus e
> acredita-se estar livre de perigo.

-- 
Esta mensagem foi verificada pelo sistema de antiv�rus e
 acredita-se estar livre de perigo.



[obm-l] Re: [obm-l] Olimpíada de Matemática Online?

2020-05-18 Por tôpico Victor Pompêo
Eu conheço a Purple Comet:
https://purplecomet.org/?action=information/summary

--
Victor


On Mon, 18 May 2020 at 11:52, Anderson Torres 
wrote:

> Não lembro onde vi, acho que foi no AOPS/Mathlinks, mas existem
> iniciativas de olimpíadas de matemática feitas online?
>
> --
> Esta mensagem foi verificada pelo sistema de antivírus e
>  acredita-se estar livre de perigo.
>
>
> =
> Instruções para entrar na lista, sair da lista e usar a lista em
> http://www.mat.puc-rio.br/~obmlistas/obm-l.html
> =
>

-- 
Esta mensagem foi verificada pelo sistema de antiv�rus e
 acredita-se estar livre de perigo.



[obm-l] Re: [obm-l] Re: [obm-l] Re: [obm-l] Re: [obm-l] Decrescimento de Funções Exponenciais

2020-05-13 Por tôpico Luiz Antonio Rodrigues
Olá, Ralph!
Tudo bem?
Muito obrigado!
Vou acessar os links!
Abraço!
Luiz


Em ter, 12 de mai de 2020 8:35 PM, Ralph Costa Teixeira 
escreveu:

> Bom, o assunto me parece ser "crescimento/decrescimento assintótico"...
> Não consigo pensar num texto para recomendar, mas olhe aqui:
> https://en.wikipedia.org/wiki/Big_O_notation
> E, em especial:
> https://en.wikipedia.org/wiki/Big_O_notation#Little-o_notation
>
> Abraço, Ralph.
>
> On Tue, May 12, 2020 at 7:09 PM Luiz Antonio Rodrigues <
> rodrigue...@gmail.com> wrote:
>
>> Olá, Ralph!
>> Tudo bem?
>> Sim, melhorou muito!
>> Muito obrigado!
>> Então, na função (5), nós temos uma incerteza...
>> Eu não havia percebido isso...
>> Muito interessante...
>> Vou ler mais sobre o assunto...
>> Você conhece algum bom livro que trate disso com mais profundidade?
>> Abraço!
>> Luiz
>>
>>
>> Em ter, 12 de mai de 2020 3:04 PM, Ralph Costa Teixeira <
>> ralp...@gmail.com> escreveu:
>>
>>> O assunto é delicado. Primeiro, precisamos de uma boa definição de
>>> "decresce mais rápido" (a gente diz que as exponenciais decrescem rápido,
>>> mas se você ler **ao pé da letra** isso é falso! A velocidade delas vai
>>> para 0 quando t vai para infinito... ou seja, elas decrescem mito
>>> devagar!?!?). Para esclarecer, suponho que queremos usar esta aqui:
>>>
>>> DEF. f(x) decresce (para 0) mais rápido (quando x vai para +Inf) do que
>>> g(x) quando lim f(x)/g(x) =0 (quando x vai para +Inf).
>>>
>>> Agora sim, você resolve tudo:
>>>
>>> 1) lim h(x)^2/h(x) = 0, portanto h^2 decresce mais rapido que h;
>>> 2) lim g(x)^2/h(x) = lim g(x)/h(x) . g(x) = 0.0=0, portanto g^2 decresce
>>> mais rapido que h;
>>> 3) lim f(x)*g(x)/h(x) = lim f(x) * (g(x)/h(x)) =0 (com f limitada),
>>> portanto fg decresce mais rapido que h;
>>> 4) lim sqrt(h)/h = lim 1/sqrt(h) =+Inf; assim, lim h/sqrt(h) = 0, ou
>>> seja, h decresce mais rapido que sqrt(h);
>>> 5) lim sqrt(g)/h = ??? Nao da para saber. Poderia ser g(x)=1/x^n e
>>> h(x)=1/x. Tomando n<2 ou n>2 podemos obter ambos comportamentos.
>>>
>>> Melhorou?
>>>
>>> Abraço, Ralph.
>>>
>>> On Tue, May 12, 2020 at 9:52 AM Luiz Antonio Rodrigues <
>>> rodrigue...@gmail.com> wrote:
>>>
 Olá, pessoal!

 Bom dia!

 Tudo bem?

 Estou tentando resolver um problema há uns 10 dias.

 Já tentei de tudo e estou com dúvidas.

 O problema é o seguinte:

 São dadas duas funções: h(x) e g(x).

 A função g(x) tende a zero mais rápido do que h(x), quando x tende a
 infinito.

 O problema pede que as seguintes funções sejam comparadas com h(x):


1.

(h(x))^2
2.

(g(x))^2
3.

f(x)*g(x)
4.

sqrt(h(x))
5.

sqrt(g(x))


 A pergunta é: quais dessas funções decrescem mais rápido do que h(x),
 quando x tende a infinito?

 Eu usei, entre outras, as seguintes funções:


 1/ln(x)

 1/x

 1/x^5

 1/e^x


 Utilizei a regra de L’Hospital e descobri que a única função que não
 decresce mais rápido do que h(x) é a (4).

 Também utilizei softwares gráficos e confirmei o meu resultado.

 Só sei que a resposta não está correta, mas ainda não sei qual seria a
 solução.

 Não consigo entender o motivo...

 Será que preciso achar um contra-exemplo?

 Alguém pode me ajudar?

 Muito obrigado!

 Abraços!

 Luiz

 --
 Esta mensagem foi verificada pelo sistema de antivírus e
 acredita-se estar livre de perigo.
>>>
>>>
>>> --
>>> Esta mensagem foi verificada pelo sistema de antivírus e
>>> acredita-se estar livre de perigo.
>>
>>
>> --
>> Esta mensagem foi verificada pelo sistema de antivírus e
>> acredita-se estar livre de perigo.
>
>
> --
> Esta mensagem foi verificada pelo sistema de antivírus e
> acredita-se estar livre de perigo.

-- 
Esta mensagem foi verificada pelo sistema de antiv�rus e
 acredita-se estar livre de perigo.



[obm-l] Re: [obm-l] Produtório trigonométrico

2020-05-12 Por tôpico Anderson Torres
Em seg., 11 de mai. de 2020 às 18:10, Maikel Andril Marcelino <
maikel.marcel...@ifrn.edu.br> escreveu:

> O e-mail está desativado? Não recebi/recebo meu último e-mail.
>
>
> Eu recebi tudo aqui

> Atenciosamente,
>
> *Maikel Andril Marcelino*
>
> *Assistente de Aluno *
> *Coordenadoria de Apoio Acadêmico - COAPAC/IFRN-SPP*
> *Instituto Federal do Rio Grande do Norte*
> *Campus São Paulo do Potengi*
>
>
> *(84) 9-9149-8991 (Contato) *
>
> *(84) 8851-3451 (WhatsApp) *
> --
> *De:* owner-ob...@mat.puc-rio.br  em nome de
> Vanderlei Nemitz 
> *Enviado:* quinta-feira, 7 de maio de 2020 15:07
> *Para:* OBM
> *Assunto:* [obm-l] Produtório trigonométrico
>
> Boa tarde!
> Alguém tem uma ideia para o seguinte produto?
> Tentei diversas transformações, mas sem sucesso.
>
> A reposta é 1.
>
> Produtório para k variando de 1 a n de (1 + 2.cos[(2pi.3^k)/(3^n + 1)]).
>
> Espero que tenha ficado clara a escrita.
>
> Muito obrigado!
>
>
> 
>  Livre
> de vírus. www.avast.com
> .
> <#m_8673821851113519950_DAB4FAD8-2DD7-40BB-A1B8-4E2AA1F9FDF2>
>
> --
> Esta mensagem foi verificada pelo sistema de antiv�rus e
> acredita-se estar livre de perigo.
>
> --
> Esta mensagem foi verificada pelo sistema de antivírus e
> acredita-se estar livre de perigo.
>

-- 
Esta mensagem foi verificada pelo sistema de antiv�rus e
 acredita-se estar livre de perigo.



[obm-l] Re: [obm-l] Re: [obm-l] Re: [obm-l] Decrescimento de Funções Exponenciais

2020-05-12 Por tôpico Ralph Costa Teixeira
Bom, o assunto me parece ser "crescimento/decrescimento assintótico"...
Não consigo pensar num texto para recomendar, mas olhe aqui:
https://en.wikipedia.org/wiki/Big_O_notation
E, em especial:
https://en.wikipedia.org/wiki/Big_O_notation#Little-o_notation

Abraço, Ralph.

On Tue, May 12, 2020 at 7:09 PM Luiz Antonio Rodrigues <
rodrigue...@gmail.com> wrote:

> Olá, Ralph!
> Tudo bem?
> Sim, melhorou muito!
> Muito obrigado!
> Então, na função (5), nós temos uma incerteza...
> Eu não havia percebido isso...
> Muito interessante...
> Vou ler mais sobre o assunto...
> Você conhece algum bom livro que trate disso com mais profundidade?
> Abraço!
> Luiz
>
>
> Em ter, 12 de mai de 2020 3:04 PM, Ralph Costa Teixeira 
> escreveu:
>
>> O assunto é delicado. Primeiro, precisamos de uma boa definição de
>> "decresce mais rápido" (a gente diz que as exponenciais decrescem rápido,
>> mas se você ler **ao pé da letra** isso é falso! A velocidade delas vai
>> para 0 quando t vai para infinito... ou seja, elas decrescem mito
>> devagar!?!?). Para esclarecer, suponho que queremos usar esta aqui:
>>
>> DEF. f(x) decresce (para 0) mais rápido (quando x vai para +Inf) do que
>> g(x) quando lim f(x)/g(x) =0 (quando x vai para +Inf).
>>
>> Agora sim, você resolve tudo:
>>
>> 1) lim h(x)^2/h(x) = 0, portanto h^2 decresce mais rapido que h;
>> 2) lim g(x)^2/h(x) = lim g(x)/h(x) . g(x) = 0.0=0, portanto g^2 decresce
>> mais rapido que h;
>> 3) lim f(x)*g(x)/h(x) = lim f(x) * (g(x)/h(x)) =0 (com f limitada),
>> portanto fg decresce mais rapido que h;
>> 4) lim sqrt(h)/h = lim 1/sqrt(h) =+Inf; assim, lim h/sqrt(h) = 0, ou
>> seja, h decresce mais rapido que sqrt(h);
>> 5) lim sqrt(g)/h = ??? Nao da para saber. Poderia ser g(x)=1/x^n e
>> h(x)=1/x. Tomando n<2 ou n>2 podemos obter ambos comportamentos.
>>
>> Melhorou?
>>
>> Abraço, Ralph.
>>
>> On Tue, May 12, 2020 at 9:52 AM Luiz Antonio Rodrigues <
>> rodrigue...@gmail.com> wrote:
>>
>>> Olá, pessoal!
>>>
>>> Bom dia!
>>>
>>> Tudo bem?
>>>
>>> Estou tentando resolver um problema há uns 10 dias.
>>>
>>> Já tentei de tudo e estou com dúvidas.
>>>
>>> O problema é o seguinte:
>>>
>>> São dadas duas funções: h(x) e g(x).
>>>
>>> A função g(x) tende a zero mais rápido do que h(x), quando x tende a
>>> infinito.
>>>
>>> O problema pede que as seguintes funções sejam comparadas com h(x):
>>>
>>>
>>>1.
>>>
>>>(h(x))^2
>>>2.
>>>
>>>(g(x))^2
>>>3.
>>>
>>>f(x)*g(x)
>>>4.
>>>
>>>sqrt(h(x))
>>>5.
>>>
>>>sqrt(g(x))
>>>
>>>
>>> A pergunta é: quais dessas funções decrescem mais rápido do que h(x),
>>> quando x tende a infinito?
>>>
>>> Eu usei, entre outras, as seguintes funções:
>>>
>>>
>>> 1/ln(x)
>>>
>>> 1/x
>>>
>>> 1/x^5
>>>
>>> 1/e^x
>>>
>>>
>>> Utilizei a regra de L’Hospital e descobri que a única função que não
>>> decresce mais rápido do que h(x) é a (4).
>>>
>>> Também utilizei softwares gráficos e confirmei o meu resultado.
>>>
>>> Só sei que a resposta não está correta, mas ainda não sei qual seria a
>>> solução.
>>>
>>> Não consigo entender o motivo...
>>>
>>> Será que preciso achar um contra-exemplo?
>>>
>>> Alguém pode me ajudar?
>>>
>>> Muito obrigado!
>>>
>>> Abraços!
>>>
>>> Luiz
>>>
>>> --
>>> Esta mensagem foi verificada pelo sistema de antivírus e
>>> acredita-se estar livre de perigo.
>>
>>
>> --
>> Esta mensagem foi verificada pelo sistema de antivírus e
>> acredita-se estar livre de perigo.
>
>
> --
> Esta mensagem foi verificada pelo sistema de antivírus e
> acredita-se estar livre de perigo.

-- 
Esta mensagem foi verificada pelo sistema de antiv�rus e
 acredita-se estar livre de perigo.



[obm-l] Re: [obm-l] Re: [obm-l] Decrescimento de Funções Exponenciais

2020-05-12 Por tôpico Luiz Antonio Rodrigues
Olá, Ralph!
Tudo bem?
Sim, melhorou muito!
Muito obrigado!
Então, na função (5), nós temos uma incerteza...
Eu não havia percebido isso...
Muito interessante...
Vou ler mais sobre o assunto...
Você conhece algum bom livro que trate disso com mais profundidade?
Abraço!
Luiz


Em ter, 12 de mai de 2020 3:04 PM, Ralph Costa Teixeira 
escreveu:

> O assunto é delicado. Primeiro, precisamos de uma boa definição de
> "decresce mais rápido" (a gente diz que as exponenciais decrescem rápido,
> mas se você ler **ao pé da letra** isso é falso! A velocidade delas vai
> para 0 quando t vai para infinito... ou seja, elas decrescem mito
> devagar!?!?). Para esclarecer, suponho que queremos usar esta aqui:
>
> DEF. f(x) decresce (para 0) mais rápido (quando x vai para +Inf) do que
> g(x) quando lim f(x)/g(x) =0 (quando x vai para +Inf).
>
> Agora sim, você resolve tudo:
>
> 1) lim h(x)^2/h(x) = 0, portanto h^2 decresce mais rapido que h;
> 2) lim g(x)^2/h(x) = lim g(x)/h(x) . g(x) = 0.0=0, portanto g^2 decresce
> mais rapido que h;
> 3) lim f(x)*g(x)/h(x) = lim f(x) * (g(x)/h(x)) =0 (com f limitada),
> portanto fg decresce mais rapido que h;
> 4) lim sqrt(h)/h = lim 1/sqrt(h) =+Inf; assim, lim h/sqrt(h) = 0, ou seja,
> h decresce mais rapido que sqrt(h);
> 5) lim sqrt(g)/h = ??? Nao da para saber. Poderia ser g(x)=1/x^n e
> h(x)=1/x. Tomando n<2 ou n>2 podemos obter ambos comportamentos.
>
> Melhorou?
>
> Abraço, Ralph.
>
> On Tue, May 12, 2020 at 9:52 AM Luiz Antonio Rodrigues <
> rodrigue...@gmail.com> wrote:
>
>> Olá, pessoal!
>>
>> Bom dia!
>>
>> Tudo bem?
>>
>> Estou tentando resolver um problema há uns 10 dias.
>>
>> Já tentei de tudo e estou com dúvidas.
>>
>> O problema é o seguinte:
>>
>> São dadas duas funções: h(x) e g(x).
>>
>> A função g(x) tende a zero mais rápido do que h(x), quando x tende a
>> infinito.
>>
>> O problema pede que as seguintes funções sejam comparadas com h(x):
>>
>>
>>1.
>>
>>(h(x))^2
>>2.
>>
>>(g(x))^2
>>3.
>>
>>f(x)*g(x)
>>4.
>>
>>sqrt(h(x))
>>5.
>>
>>sqrt(g(x))
>>
>>
>> A pergunta é: quais dessas funções decrescem mais rápido do que h(x),
>> quando x tende a infinito?
>>
>> Eu usei, entre outras, as seguintes funções:
>>
>>
>> 1/ln(x)
>>
>> 1/x
>>
>> 1/x^5
>>
>> 1/e^x
>>
>>
>> Utilizei a regra de L’Hospital e descobri que a única função que não
>> decresce mais rápido do que h(x) é a (4).
>>
>> Também utilizei softwares gráficos e confirmei o meu resultado.
>>
>> Só sei que a resposta não está correta, mas ainda não sei qual seria a
>> solução.
>>
>> Não consigo entender o motivo...
>>
>> Será que preciso achar um contra-exemplo?
>>
>> Alguém pode me ajudar?
>>
>> Muito obrigado!
>>
>> Abraços!
>>
>> Luiz
>>
>> --
>> Esta mensagem foi verificada pelo sistema de antivírus e
>> acredita-se estar livre de perigo.
>
>
> --
> Esta mensagem foi verificada pelo sistema de antivírus e
> acredita-se estar livre de perigo.

-- 
Esta mensagem foi verificada pelo sistema de antiv�rus e
 acredita-se estar livre de perigo.



[obm-l] Re: [obm-l] Decrescimento de Funções Exponenciais

2020-05-12 Por tôpico Ralph Costa Teixeira
P.S.: Na (3) se ele nao falou que f eh limitada, a resposta passa a ser NAO
SEI.

On Tue, May 12, 2020 at 2:52 PM Ralph Costa Teixeira 
wrote:

> O assunto é delicado. Primeiro, precisamos de uma boa definição de
> "decresce mais rápido" (a gente diz que as exponenciais decrescem rápido,
> mas se você ler **ao pé da letra** isso é falso! A velocidade delas vai
> para 0 quando t vai para infinito... ou seja, elas decrescem mito
> devagar!?!?). Para esclarecer, suponho que queremos usar esta aqui:
>
> DEF. f(x) decresce (para 0) mais rápido (quando x vai para +Inf) do que
> g(x) quando lim f(x)/g(x) =0 (quando x vai para +Inf).
>
> Agora sim, você resolve tudo:
>
> 1) lim h(x)^2/h(x) = 0, portanto h^2 decresce mais rapido que h;
> 2) lim g(x)^2/h(x) = lim g(x)/h(x) . g(x) = 0.0=0, portanto g^2 decresce
> mais rapido que h;
> 3) lim f(x)*g(x)/h(x) = lim f(x) * (g(x)/h(x)) =0 (com f limitada),
> portanto fg decresce mais rapido que h;
> 4) lim sqrt(h)/h = lim 1/sqrt(h) =+Inf; assim, lim h/sqrt(h) = 0, ou seja,
> h decresce mais rapido que sqrt(h);
> 5) lim sqrt(g)/h = ??? Nao da para saber. Poderia ser g(x)=1/x^n e
> h(x)=1/x. Tomando n<2 ou n>2 podemos obter ambos comportamentos.
>
> Melhorou?
>
> Abraço, Ralph.
>
> On Tue, May 12, 2020 at 9:52 AM Luiz Antonio Rodrigues <
> rodrigue...@gmail.com> wrote:
>
>> Olá, pessoal!
>>
>> Bom dia!
>>
>> Tudo bem?
>>
>> Estou tentando resolver um problema há uns 10 dias.
>>
>> Já tentei de tudo e estou com dúvidas.
>>
>> O problema é o seguinte:
>>
>> São dadas duas funções: h(x) e g(x).
>>
>> A função g(x) tende a zero mais rápido do que h(x), quando x tende a
>> infinito.
>>
>> O problema pede que as seguintes funções sejam comparadas com h(x):
>>
>>
>>1.
>>
>>(h(x))^2
>>2.
>>
>>(g(x))^2
>>3.
>>
>>f(x)*g(x)
>>4.
>>
>>sqrt(h(x))
>>5.
>>
>>sqrt(g(x))
>>
>>
>> A pergunta é: quais dessas funções decrescem mais rápido do que h(x),
>> quando x tende a infinito?
>>
>> Eu usei, entre outras, as seguintes funções:
>>
>>
>> 1/ln(x)
>>
>> 1/x
>>
>> 1/x^5
>>
>> 1/e^x
>>
>>
>> Utilizei a regra de L’Hospital e descobri que a única função que não
>> decresce mais rápido do que h(x) é a (4).
>>
>> Também utilizei softwares gráficos e confirmei o meu resultado.
>>
>> Só sei que a resposta não está correta, mas ainda não sei qual seria a
>> solução.
>>
>> Não consigo entender o motivo...
>>
>> Será que preciso achar um contra-exemplo?
>>
>> Alguém pode me ajudar?
>>
>> Muito obrigado!
>>
>> Abraços!
>>
>> Luiz
>>
>> --
>> Esta mensagem foi verificada pelo sistema de antivírus e
>> acredita-se estar livre de perigo.
>
>

-- 
Esta mensagem foi verificada pelo sistema de antiv�rus e
 acredita-se estar livre de perigo.



[obm-l] Re: [obm-l] Decrescimento de Funções Exponenciais

2020-05-12 Por tôpico Ralph Costa Teixeira
O assunto é delicado. Primeiro, precisamos de uma boa definição de
"decresce mais rápido" (a gente diz que as exponenciais decrescem rápido,
mas se você ler **ao pé da letra** isso é falso! A velocidade delas vai
para 0 quando t vai para infinito... ou seja, elas decrescem mito
devagar!?!?). Para esclarecer, suponho que queremos usar esta aqui:

DEF. f(x) decresce (para 0) mais rápido (quando x vai para +Inf) do que
g(x) quando lim f(x)/g(x) =0 (quando x vai para +Inf).

Agora sim, você resolve tudo:

1) lim h(x)^2/h(x) = 0, portanto h^2 decresce mais rapido que h;
2) lim g(x)^2/h(x) = lim g(x)/h(x) . g(x) = 0.0=0, portanto g^2 decresce
mais rapido que h;
3) lim f(x)*g(x)/h(x) = lim f(x) * (g(x)/h(x)) =0 (com f limitada),
portanto fg decresce mais rapido que h;
4) lim sqrt(h)/h = lim 1/sqrt(h) =+Inf; assim, lim h/sqrt(h) = 0, ou seja,
h decresce mais rapido que sqrt(h);
5) lim sqrt(g)/h = ??? Nao da para saber. Poderia ser g(x)=1/x^n e
h(x)=1/x. Tomando n<2 ou n>2 podemos obter ambos comportamentos.

Melhorou?

Abraço, Ralph.

On Tue, May 12, 2020 at 9:52 AM Luiz Antonio Rodrigues <
rodrigue...@gmail.com> wrote:

> Olá, pessoal!
>
> Bom dia!
>
> Tudo bem?
>
> Estou tentando resolver um problema há uns 10 dias.
>
> Já tentei de tudo e estou com dúvidas.
>
> O problema é o seguinte:
>
> São dadas duas funções: h(x) e g(x).
>
> A função g(x) tende a zero mais rápido do que h(x), quando x tende a
> infinito.
>
> O problema pede que as seguintes funções sejam comparadas com h(x):
>
>
>1.
>
>(h(x))^2
>2.
>
>(g(x))^2
>3.
>
>f(x)*g(x)
>4.
>
>sqrt(h(x))
>5.
>
>sqrt(g(x))
>
>
> A pergunta é: quais dessas funções decrescem mais rápido do que h(x),
> quando x tende a infinito?
>
> Eu usei, entre outras, as seguintes funções:
>
>
> 1/ln(x)
>
> 1/x
>
> 1/x^5
>
> 1/e^x
>
>
> Utilizei a regra de L’Hospital e descobri que a única função que não
> decresce mais rápido do que h(x) é a (4).
>
> Também utilizei softwares gráficos e confirmei o meu resultado.
>
> Só sei que a resposta não está correta, mas ainda não sei qual seria a
> solução.
>
> Não consigo entender o motivo...
>
> Será que preciso achar um contra-exemplo?
>
> Alguém pode me ajudar?
>
> Muito obrigado!
>
> Abraços!
>
> Luiz
>
> --
> Esta mensagem foi verificada pelo sistema de antivírus e
> acredita-se estar livre de perigo.

-- 
Esta mensagem foi verificada pelo sistema de antiv�rus e
 acredita-se estar livre de perigo.



[obm-l] Re: [obm-l] Re: [obm-l] Decrescimento de Funções Exponenciais

2020-05-12 Por tôpico Luiz Antonio Rodrigues
Olá, Pedro!
Tudo bem?
Muito obrigado por sua resposta.
Funcionou!
O problema estava na função (5).
Mas eu estive pensando no que acontece com esta função.
É como se ela coincidisse, quando x tende a infinito, com a função original
(h(x))?
Isto é muito interessante...



Em ter, 12 de mai de 2020 12:09 PM, Pedro Angelo 
escreveu:

> Sobre o item 5, o que acontece se h(x)=x^(-1) e g(x)=x^(-1.1) ?
>
> Le mar. 12 mai 2020 à 09:52, Luiz Antonio Rodrigues
>  a écrit :
> >
> > Olá, pessoal!
> >
> > Bom dia!
> >
> > Tudo bem?
> >
> > Estou tentando resolver um problema há uns 10 dias.
> >
> > Já tentei de tudo e estou com dúvidas.
> >
> > O problema é o seguinte:
> >
> >
> > São dadas duas funções: h(x) e g(x).
> >
> > A função g(x) tende a zero mais rápido do que h(x), quando x tende a
> infinito.
> >
> > O problema pede que as seguintes funções sejam comparadas com h(x):
> >
> >
> > (h(x))^2
> >
> > (g(x))^2
> >
> > f(x)*g(x)
> >
> > sqrt(h(x))
> >
> > sqrt(g(x))
> >
> >
> > A pergunta é: quais dessas funções decrescem mais rápido do que h(x),
> quando x tende a infinito?
> >
> > Eu usei, entre outras, as seguintes funções:
> >
> >
> > 1/ln(x)
> >
> > 1/x
> >
> > 1/x^5
> >
> > 1/e^x
> >
> >
> > Utilizei a regra de L’Hospital e descobri que a única função que não
> decresce mais rápido do que h(x) é a (4).
> >
> > Também utilizei softwares gráficos e confirmei o meu resultado.
> >
> > Só sei que a resposta não está correta, mas ainda não sei qual seria a
> solução.
> >
> > Não consigo entender o motivo...
> >
> > Será que preciso achar um contra-exemplo?
> >
> > Alguém pode me ajudar?
> >
> > Muito obrigado!
> >
> > Abraços!
> >
> > Luiz
> >
> >
> > --
> > Esta mensagem foi verificada pelo sistema de antivírus e
> > acredita-se estar livre de perigo.
>
> --
> Esta mensagem foi verificada pelo sistema de antivírus e
>  acredita-se estar livre de perigo.
>
>
> =
> Instru�ões para entrar na lista, sair da lista e usar a lista em
> http://www.mat.puc-rio.br/~obmlistas/obm-l.html
> =
>

-- 
Esta mensagem foi verificada pelo sistema de antiv�rus e
 acredita-se estar livre de perigo.



[obm-l] Re: [obm-l] Decrescimento de Funções Exponenciais

2020-05-12 Por tôpico Pedro Angelo
Sobre o item 5, o que acontece se h(x)=x^(-1) e g(x)=x^(-1.1) ?

Le mar. 12 mai 2020 à 09:52, Luiz Antonio Rodrigues
 a écrit :
>
> Olá, pessoal!
>
> Bom dia!
>
> Tudo bem?
>
> Estou tentando resolver um problema há uns 10 dias.
>
> Já tentei de tudo e estou com dúvidas.
>
> O problema é o seguinte:
>
>
> São dadas duas funções: h(x) e g(x).
>
> A função g(x) tende a zero mais rápido do que h(x), quando x tende a infinito.
>
> O problema pede que as seguintes funções sejam comparadas com h(x):
>
>
> (h(x))^2
>
> (g(x))^2
>
> f(x)*g(x)
>
> sqrt(h(x))
>
> sqrt(g(x))
>
>
> A pergunta é: quais dessas funções decrescem mais rápido do que h(x), quando 
> x tende a infinito?
>
> Eu usei, entre outras, as seguintes funções:
>
>
> 1/ln(x)
>
> 1/x
>
> 1/x^5
>
> 1/e^x
>
>
> Utilizei a regra de L’Hospital e descobri que a única função que não decresce 
> mais rápido do que h(x) é a (4).
>
> Também utilizei softwares gráficos e confirmei o meu resultado.
>
> Só sei que a resposta não está correta, mas ainda não sei qual seria a 
> solução.
>
> Não consigo entender o motivo...
>
> Será que preciso achar um contra-exemplo?
>
> Alguém pode me ajudar?
>
> Muito obrigado!
>
> Abraços!
>
> Luiz
>
>
> --
> Esta mensagem foi verificada pelo sistema de antivírus e
> acredita-se estar livre de perigo.

-- 
Esta mensagem foi verificada pelo sistema de antiv�rus e
 acredita-se estar livre de perigo.


=
Instru��es para entrar na lista, sair da lista e usar a lista em
http://www.mat.puc-rio.br/~obmlistas/obm-l.html
=


<    1   2   3   4   5   6   7   8   9   10   >